You are on page 1of 180

1

.
2

TABLE OF CONTENTS

Lucero vda. De Sindayen vs. Insular Life Assurance Co., 62 Phil. 9 (1935) 5

Enriquez vs. Sun Life Assur. Co. of Canada, 41 Phil. 209 (1920) 8

The Insular Life Assur. Co. vs. Ebrado, 80 SCRA 18 (1977) 10

Heirs of Maramag vs. Maramag (2009) 13

Sun Insurance Office, Ltd. Vs. Court of Appeals, 195 SCRA 193 (1991) 16

Fortune Medicare vs. David Robert U. Amorin, G.R. No. 195872, March 12, 2014 19
DBP Pool Accredited Insurance Companies vs. Radio Mindanao Network 480
SCRA 314 (2006) 22

Philippine Health Care Provider Inc. vs. Comm of Internal Revenue, 600 SCRA 413
(2009) 24

Phil. Refining Co. vs. Palomar, 148 SCRA 313 27

Zenith Insurance Corp. vs. Court of Appeals, 185 SCRA 398 29

Blue Cross Health Care vs. Olivares, 544 SCRA 580 (2008) 32

Rizal Surety & Insurance Company vs. Court of Appeals, 336 SCRA 12 (2000 35

General Corporation of the Philippines vs. Union Insurance Society of Canton,


G.R. No. L-2684, September 14, 1950 38

Lorenzo Shipping vs. Chubb and Sons, G.R. No. 147724, June 8, 2004 43

Geagonia vs. Court of Appeals, 241 SCRA 152 [1995] 45

Lalican vs. Insular Life Insurance Co. Limited, 597 SCRA 159 (2009) 48

Gaisano Cagayan Inc. vs. Insurance Company of North America (G.R. No. 147839,
June 8, 2006) 51

Ong Lim Sing Jr. vs. FGB Leasing Finance Corp. 524 SCRA 333 (2007) 53

Cha vs. Court of Appeals, 277 SCRA 690 (1997) 55

San Miguel Brewery vs. Law Union & Rock Ins. Co., 40 Phil. 674 [1920] 57

Malayan Insurance Company, Inc. vs. PAP Co., Ltd. (2013), G.R. No. 200784,
August 7, 2013 59

Saturnino vs. Phil. American Life Insurance Co., 7 SCRA 316 (1963) 64
3

Philamcare Health Systems, Inc., vs. Court Of Appeals and Julita Trinos, G.R. No.
125678, March 18, 2002 67

Great Pacific Life Assn. Co. vs. Court of Appeals, 89 SCRA 543 (1979) 70

Sunlife Assur. Co. of Canada vs. Court of Appeals, 245 SCRA 268 (1995) 73

Ng Zee vs. Asian Crusader Life Assurance Corp. 122 SCRA 461 [1983] 75

Argente Vs. West Coast Life Insurance Co, 51 Phil 275 78

Tan vs Court of Appeals, 174 SCRA 403 (1989) 81

Manila Bankers Life Ins. Vs. Aban, 702 SCRA 417 83

Coquia vs. Fieldmen’s Insurance Co., Inc., G.R. No. L-23276, Nov. 29, 1968 85

Bonifacio Bros. Inc. vs Mora, G.R. No. L-20853, May 29, 196 87

Rizal Commercial Banking Corporation vs. Court of Appeals, G.R. Nos. 128833,
April 20, 1998 89

Sun Insurance Office Ltd. vs. Court of Appeals, G.R. No. 89741, March 13, 1991 92
Malayan Insurance Co., Inc. vs. Cruz-Arnaldo, G.R. No. 67835, October 12, 1987 95
American Home Assurance Co. vs. Tantoco Enterprise Inc. 366 SCRA 740 (2001)
97

Gen. Insurance v. Ng Hua, G.R. No. L-14373, January 30, 1960 99

Prudential Guarantee and Assurance, Inc. vs. Trans-Asia Shipping Lines, Inc.
G.R. No. 151890, June 20, 2006 101

Delsan Transport Lines, Inc. v. The Hon. Court of Appeals, G.R. No. 127897,
November 15, 2001 103

American Home Assurance v. Chua, 309 SCRA 250 (1999) 106

Makati Tuscany Condominium vs. Court of Appeals, G.R. No. 95546, Nov. 6, 1992
107

UCPB General Insurance vs. Masagana Telamart, G.R. No. 137172, April 4, 2001
(decision on the M/R) 111

Phil. Phoenix vs Woodworks, 20 SCRA 1270 (1967) 113

Phil. Phoenix vs. Woodworks, 92 SCRA 419 (1979) 115

Grepalife vs. CA, 184 SCRA 501 (1990) 118

Heirs of I. Coscolluela vs. Rico General Insurance Corp. 179 SCRA 511 (1989) 120
4

Country Bankers Insurance Corp. vs. Lianga Bay and Multi-Purpose Cooperative,
Inc. 374 SCRA 653 123

DBP Pool of Accredited Insurance Companies vs. Radio Mindanao Network, Inc.
480 SCRA 314 (2006) 125

FGU Insurance Corporation vs. Court of Appeals, 454 SCRA 337 (2005) 127

Bonifacio Bros. vs. Mora, 20 SCRA 261 (1969) 129

Allied Banking Corp. vs. Lim Sio Wan, 549 SCRA 504 (2008) 132

Stokes vs. Malayan Insurance Co., Inc. 127 SCRA 766 (1984) 134

United Merchants Corporation vs. Country Bankers Insurance Corporation. 676


SCRA 382 (2012) 137

Santa Ana vs. Commercial Union Assurance Company, Ltd., 55 Phil. 329. 140

Guingon vs. Del Monte, 20 SCRA 1043 143

Go Tiaco vs. Union Insurance, 40 Phil. 40 (1919) 145

White Gold Marine Services vs. Pioneer Insurance, 464 SCRA 448 (2005) 148

Cathay Insurance Co. vs. Court of Appeals, 151 SCRA 710 (1987) 151

Choa Tiek Seng vs. Court of Appeals, 183 SCRA 223 [1990] 153

Filipino Merchants Insurance Co., vs. Court of Appeals, 179 SCRA 638 (1989) 157

Caltex [Phil.] Inc. vs. Sulpicio Lines, Inc., 315 SCRA 709 (1999) 160

San Miguel Corporation vs. Heirs of S. Inguito, 384 SCRA 87 [2002] 162

Roque vs. Intermediate Appellate Court 139 SCRA 596 (1985) 165

Phil. American General Insurance Co. vs. Court of Appeals, 273 SCRA 262 (1997)
168

Pan Malayan Insurance Co. vs. Court of Appeals, 201 SCRA 382 (1991) 170

Aboitiz Shipping Corp. vs. Court of Appeals, 569 SCRA 294 (2008) 174

Oriental Assurance Corporation, vs. Court of Appeals and Panama Saw Mill Co.,
Inc., G.R. No. 94052, August 09, 1991 176

Vector Shipping Corporation v. American Home Assurance Company (G.R. No.


159213) 179
5

Case Title Lucero vda. De Sindayen vs. Insular Life Assurance Co.,
62 Phil. 9 (1935)

Topic Consent, delivery

Digested By ABONAL, Maria Lou Isabel P.

Characters and Their Roles

Petitioner: Fortunata Lucero Viuda de Sendayen (wife of the deceased Arturo


Sindayen)

Respondent: Insular Life Assurance Co., Ltd.

FACTS:

Arturo Sindayen (ARTURO), a linotype operator in the Bureau of Printing at Manila, and
his wife Fortunata Lucero Sindayen (LUCERO) went to Tarlac to spend Christmas with
his aunt, Felicidad Estrada (ESTRADA).

On December 26, 1932, ARTURO applied for an insurance policy with Insular Life
Assistance Co., Ltd. (INSULAR). He submitted a written application to INSULAR’s agent,
Cristobal Mendoza (MENDOZA), for a policy of insurance on his life in the sum of Php
1000. ARTURO paid MENDOZA a portion of the premium, amounting to Php 15. They
both agreed that the policy be delivered to ESTRADA and she will complete the payment.
ARTURO left Php 26.06 to ESTRADA and she paid it to MENDOZA upon delivery of the
policy for the first annual premium of Php 40.06.

The application which ARTURO signed in Tarlac contained among others the following
provisions:

1. That if this application is accepted and a policy issued in my favor, I bind myself to
accept the same and to pay at least the first year's premium thereon in the City of Manila.
6

2. That the said policy shall not take effect until the first premium has been paid and the
policy has been delivered to and accepted by me, while I am in good health.

3. That the agent taking this application has no authority to make, modify or discharge
contracts, or to waive any of the Company's right or requirements.

On January 1, 1933, ARTURO was examined by a doctor and concluded he was healthy.
He was also only 29 years old. On January 11 of the same year, INSULAR accepted the
risk and issued the policy ARTURO applied for, dated Dec. 1, 1932 and mailed it to
MENDOZA for delivery.

But then, on January 12, 1933, ARTURO got sick and he was found to be suffering from
acute nephritis and uremia. He was not treated for his illness.

MENDOZA delivered the policy to ESTRADA on January 18. MENDOZA inquired about
ARTURO's health and ESTRADA replied that she believed he was healthy as he had no
information that he was sick. On January 19, ARTURO died. Only on the next day,
January 20, did MENDOZA find out about the death of ARTURO. He then called
ESTRADA to return the policy. She returned it but MENDOZA did not offer to return the
premium already paid.

The next month, February 4, 1933, INSULAR obtained from the widow of ARTURO,
LUCERO, her signature to a legal document entitled “ACCORD, SATISFACTION, AND
RELEASE” which stipulated: “Whereby in consideration of the sum of Php 40.06 paid to
her by a check of the company, she “assigns, releases and forever discharges said Insular
Life Assurance Co., Ltd., its successors and assigns, of all claims, obligation in or
indebtedness which she, as such beneficiary ever had or now has, hereafter, shall, or
may have, for, upon, or by reason of said policy of life insurance numbered 47710 upon
the life of said Arturo Sindayen, the latter now deceased, or arising therefrom or
connected therewith in any manner.” The check was never cashed but it was returned to
the company. And so, this action was brought to enforce payment of the policy.

ISSUE:

1. Whether or not there was delivery despite the fact that the policy was not delivered
to and accepted by the insured (ARTURO) in person

2. Whether or not ARTURO’s bad health at the time of the delivery of the application
rendered the policy void.

RULING:

1. YES. The delivery was made.


7

The Supreme Court emphasized that delivery to the insured in person is not necessary.
Delivery may be made by mail or to a duly constituted agent. INSULAR cited no
authorities to support its proposition and none need be cited to refute it.

2. NO. The policy remains enforceable.

The act of delivery of the policy in absence of fraud or other ground for rescission
consummates the insurance. INSULAR having decided that all the conditions precedent
to the taking effect of the policy had been complied with and having accepted the premium
and delivered the policy thereafter to the insured, it is now estopped to assert that it never
intended that the policy should take effect. Also, MENDOZA was authorized by the
company to make the delivery of the policy when he received the payment of the first
premium and he was satisfied that the insured, ARTURO, was in good health.
MENDOZA’s decision is just as binding on the company as if the decision has been made
by its board of directors. MENDOZA and even ESTRADA had no knowledge at that time
that the insured, ARTURO, was already sick. As such there was no fraud or even collusion
between MENDOZA and ESTRADA that could serve as a ground for the policy to be
voided.

As such, the Supreme Court holds that INSULAR assumed the risk covered the policy
ARTURO applied for when the policy was delivered and the premium paid for.
8

Case Title Enriquez vs. Sun Life Assur. Co. of Canada, 41 Phil. 209
(1920)

Topic Perfection of Insurance Contract

Digested By BARROSO, Jon Carlow L.

Characters and Their Roles

Petitioner: RAFAEL ENRIQUEZ, as administrator of the estate of the late Joaquin Ma.
Herrer,

Respondent: SUN LIFE ASSURANCE COMPANY OF CANADA,

FACTS:

On Sept. 24 1917, Joaquin Ma. Herrer made an application to SunLife through its office
in Manila for life annuity. Two (2) days later, he paid the sum of P6,000 to the company’s
manager in its Manila office and was given a receipt.

On Nov. 26, 1917, the head office in Montreal, Canada gave notice of acceptance by
cable wire to Manila. On the same date, the Manila office prepared a letter notifying Herrer
that his application had been accepted and this was placed in the ordinary channels of
transmission, but as far as known was never actually mailed and never received by
Herrer. Herrer died on Dec. 20, 1917. Enriquez, as administrator of Herrer’s estate,
brought this action to recover the P6,000 paid by the deceased.

ISSUE:

Whether or not the insurance contract was perfected.

RULING:
9

NO. The contract for life annuity was NOT perfected because it had NOT been proved
satisfactorily that the acceptance of the application ever came to the knowledge of the
applicant. An acceptance of an offer of insurance NOT actually or constructively
communicated to the proposer does NOT make a contract of insurance, as the locus
poenitentiae is ended when an acceptance has passed beyond the control of the party.

There being no perfected contract, the policy cannot bind Herrer. Art. 1262 provides that
acceptance by letter does not bind the person making the offer except from the time it
came to his knowledge. The pertinent fact is that according to the provisional receipt, the
insurance company had to: 1) conduct a medical examination; 2) had to obtain the head
office's approval; and 3) somehow communicate such approval. It is true that the letter
notifying acceptance was deposited in the post office, but the fact of notification is a
rebuttable presumption and the facts clearly show that Herrer never received the notice
of the acceptance before his death.
10

Case Title The Insular Life Assur. Co. vs. Ebrado, 80 SCRA 18
(1977)

Topic Disqualified beneficiary

Digested By BOISER, Virgilio IV A.

Characters and Their Roles

Petitioner: THE INSULAR LIFE ASSURANCE COMPANY, LTD.

Respondent: CARPONIA T. EBRADO

FACTS:

On September 1, 1968, Buenaventura Cristor Ebrado was issued by The Insurance Life
Assurance Co., Ltd., on a whole-life for P5,882.00 with a rider for Accidental Death for
the same amount. He designated Carponia T. Ebrado, his common-law wife, as the
revocable beneficiary in his policy. He referred to her as his wife in the policy.

On October 21, 1969, He died as a result of an accident when he was hit by a failing
branch of a tree. As the policy was in force, the insurance company was liable to pay the
coverage in the total amount of P11,745.73, representing the face value of the policy in
the amount of P5,882.00 plus the additional benefits for accidental death also in the
amount of P5,882.00 and the refund of P18.00 paid for the premium due November, 1969,
minus the unpaid premiums and interest thereon due for January and February, 1969, in
the sum of P36.27.

Carponia T. Ebrado filed a claim for the proceeds of the Policy as the designated
beneficiary therein, although she admits that she and the insured Buenaventura C.
Ebrado were merely living as husband and wife without the benefit of marriage. Pascuala
T. Ebrado, also filed a claim to the insurance company, this time claiming to be the legal
wife of Buenaventura. She asserts that she has a better right over the proceeds than
Carponia who is a common-law wife.

ISSUE:
11

Whether or not a common-law wife named as beneficiary in the life insurance policy of a
legally married man may claim the proceeds thereof in case of death of the latter.

RULING:

NO.

According to the Supreme Court, the New Civil Code states: “The contract of insurance
is governed by special laws. Matters not expressly provided for in such special laws shall
be regulated by this Code.” When not otherwise specifically provided for by the Insurance
Law, the contract of life insurance is governed by the general rules of the civil law
regulating contracts. And under Article 2012 of the same Code, “any person who is
forbidden from receiving any donation under Article 739 cannot be named beneficiary of
a life insurance policy by the person who cannot make a donation to him.” Common-law
spouses are, definitely, barred from receiving donations from each other. Article 739 of
the new Civil Code provides:

“The following donations shall be void:

“1. Those made between persons who were guilty of adultery or concubinage at the time
of donation;

“2. Those made between persons found guilty of the same criminal offense, in
consideration thereof;

“3. Those made to a public officer or his wife, descendants or ascendants by reason of
his office.

“In the case referred to in No. 1, the action for declaration of nullity may be brought by the
spouse of the donor or donee; and the guilt of the donee may be proved by
preponderance of evidence in the same action.”

In essence, a life insurance policy is no different from a civil donation insofar as the
beneficiary is concerned. Both are founded upon the same consideration: liberality. A
beneficiary is like a donee, because from the premiums of the policy which the insured
pays out of liberality, the beneficiary will receive the proceeds or profits of said insurance.
As a consequence, the proscription in Article 739 of the new Civil Code should equally
operate in life insurance contracts. The mandate of Article 2012 cannot be laid aside: any
person who cannot receive a donation cannot be named as beneficiary in the life
insurance policy of the person who cannot make the donation. Under American law, a
policy of life insurance is considered as a testament and in construing it, the courts will,
so far as possible treat it as a will and determine the effect of a clause designating the
beneficiary by rules under which wills are interpreted.
12

Policy considerations and dictates of morality rightly justify the institution of a barrier
between common-law spouses in regard to property relations since such relationship
ultimately encroaches upon the nuptial and filial rights of the legitimate family. There is
every reason to hold that the bar on donations between legitimate spouses and those
between illegitimate ones should be enforced in life insurance policies since the same are
based on similar considerations. As above pointed out, a beneficiary in a life insurance
policy is no different from a donee. Both the recipients of pure beneficence. So long as
marriage remains the threshold of family laws, reason and morality dictate that the
impediments imposed upon married couple should likewise be imposed upon extra-
marital relationship. If a legitimate relationship is circumscribed by these legal disabilities,
with more reason should an illicit relationship be restricted by these disabilities.

Further, the Court said that a conviction for adultery or concubinage does not need to be
exacted before the disabilities mentioned in Article 739 may effectuate. More specifically,
with regard to the disability on “persons who were guilty of adultery or concubinage at the
time of the donation,” Article 739 itself provides:

“In the case referred to in No. 1, the action for declaration of nullity may be brought by the
spouse of the donor or donee; and the guilt of the donee may be proved by
preponderance of evidence in the same action.”

The underscored clause neatly conveys that no criminal conviction for the disqualifying
offense is a condition precedent. In fact, it cannot even be gleaned from the aforequoted
provision that a criminal prosecution is needed. On the contrary, the law plainly states
that the guilt of the party may be proved “in the same action” for declaration of nullity of
donation. And, it would be sufficient if evidence preponderates upon the guilt of the
consort for the offense indicated. The quantum of proof in criminal cases is not demanded.
13

Case Title Heirs of Maramag vs. Maramag (2009)

Topic Rules on Succession not applicable

Digested By BUSTAMANTE, Anne Georgette A.

Characters and Their Roles

Petitioner: Vicenta and her children ( legitimate family)

Respondent: Eva de Guzman Maramag and her children (Odessa, Karl Brian, and
Trisha Angelie) illegitimate family

FACTS:

Vicenta Maramag and her children (legitimate family of Loreto Maramag) filed an action
for revocation and/or reduction of insurance proceeds for being void and/or inofficious
against Eva De Guzman Maramag et al. (Loreto's illegitimate family)

Eva was a concubine of Loreto and a suspect in the killing of the latter, thus, she is
disqualified to receive any proceeds from his insurance policies from Insular Life
Assurance Company, Ltd. (Insular) and Great Pacific Life Assurance Corporation
(Grepalife).

Further, the illegitimate children of Loreto by Eva (namely: Odessa, Karl Brian, and Trisha
Angelie) were entitled only to one-half of the legitime of the legitimate children, thus, the
proceeds released to Odessa and those to be released to Karl Brian and Trisha Angelie
(both minors) were inofficious and should be reduced in accordance with the Civil Code
provisions on inofficious donations.

Insular admitted that Loreto misrepresented Eva as his legitimate wife and Odessa, Karl
Brian, and Trisha Angelie as his legitimate children. Insular claimed that when it
ascertained that Eva was not the legal wife of Loreto, it disqualified her as a beneficiary
14

and divided the proceeds among Odessa, Karl Brian, and Trisha Angelie, as the
remaining designated beneficiaries.

Insular further claimed that it was bound to honor the insurance policies designating the
children of Loreto with Eva as beneficiaries pursuant to Section 53 of the Insurance Code.

Vicenta et al insist that the designation of a beneficiary is an act of liberality or a donation


and, therefore, subject to the provisions of Articles 752 and 772 of the Civil Code.

Thus, Vicenta prayed that the share of Eva and portions of the shares of Loreto's
illegitimate children should be awarded to them, being the legitimate heirs of Loreto
entitled to their respective legitimes.

ISSUE:

Whether or not compulsory heirs not being named as beneficiaries in the insurance
policies are entitled to the proceeds.

RULING:

NO. The legitimate family are third parties to the insurance contracts with Insular and
Grepalife and, thus, are not entitled to the proceeds thereof. Accordingly, Insular and
Grepalife have no legal obligation to turn over the insurance proceeds to the compulsory
heirs.

Article 2011 of the Civil Code expressly provides that insurance contracts shall be
governed by special laws, i.e., the Insurance Code. Section 53 of the Insurance Code
states

SECTION 53. The insurance proceeds shall be applied exclusively to the proper interest
of the person in whose name or for whose benefit it is made unless otherwise specified
in the policy.

Pursuant thereto, it is obvious that the only persons entitled to claim the insurance
proceeds are either the insured, if still alive; or the beneficiary, if the insured is already
deceased, upon the maturation of the policy. The exception to this rule is a situation where
the insurance contract was intended to benefit third persons who are not parties to the
same in the form of favorable stipulations or indemnity.
15

The revocation of Eva as a beneficiary in one policy and her disqualification as such in
another are of no moment considering that the designation of the illegitimate children as
beneficiaries in Loreto's insurance policies remain valid. Because no legal proscription
exists in naming as beneficiaries the children of illicit relationships by the insured, the
shares of Eva in the insurance proceeds, whether forfeited by the court in view of the
prohibition on donations under Article 739 of the Civil Code or by the insurers themselves
for reasons based on the insurance contracts, must be awarded to the said illegitimate
children, the designated beneficiaries, to the exclusion of petitioners.

It is only in cases where the insured has not designated any beneficiary, or when the
designated beneficiary is disqualified by law to receive the proceeds, that the insurance
policy proceeds shall redound to the benefit of the estate of the insured.
16

Case Title Sun Insurance Office, Ltd. Vs. Court of Appeals, 195
SCRA 193 (1991)

Topic Insurance, Prescriptive Period

Digested By CAGAPE, Joanna Paula M.

Characters and Their Roles

Petitioner: Sun Insurance, insurer

Respondent: Emilio Tan

FACTS:

Emilio Tan took a P300,000.00 property insurance policy to cover his interest in the
electrical supply store of his brother housed in a building in Iloilo City on August 15, 1983.
After 4 days, the house was burned. Tan filed his claim for fire loss with Sun Insurance
Office on August 20, 1983 but Sun Insurance Office denied the claim on February 20,
1984. Tan sought reconsideration in April 1984 but the same was rejected. Tan kept on
seeking for reconsideration but Sun Insurance advised Tan’s counsel that the denial of
Tan’s claim remained unchanged (September 3, 1985.)

On November 20, 1985, Tan filed Civil Case No. 16817 with the Regional Trial Court of
Iloilo, Branch 27. Sun Insurance Company filed a motion to dismiss on the alleged ground
that the action had already prescribed. Said motion was denied in an order dated
November 3, 1987; and petitioner's motion for reconsideration was also denied in an order
dated January 14, 1988.

ISSUES:

1. Whether or not the filing of a motion for reconsideration interrupts the twelve (12)
months prescriptive period to contest the denial of the Insurance claim
17

2. Whether or not the rejection of the claim shall be deemed final only if it contains
words to the effect that the denial is final

RULING:

1. No. The answer to the first issue is in the negative.

While it is a cardinal principle of insurance law that a policy or contract of insurance is to


be construed liberally in favor of the insured and strictly against the insurer company, yet,
contracts of insurance, like other contracts, are to be construed according to the sense
and meaning of the terms which the parties themselves have used. If such terms are clear
and unambiguous, they must be taken and understood in their plain, ordinary and popular
sense (Pacific Banking Corp. v. Court of Appeals, 168 SCRA 1 [1988]).

Condition 27 of the Insurance Policy, which is the subject of the conflicting contentions of
the parties, reads:

27. Action or suit clause — If a claim be made and rejected and an action or suit
be not commenced either in the Insurance Commission or in any court of
competent jurisdiction within twelve (12) months from receipt of notice of such
rejection, or in case of arbitration taking place as provided herein, within twelve
(12) months after due notice of the award made by the arbitrator or arbitrators or
umpire, then the claim shall for all purposes be deemed to have been abandoned
and shall not thereafter be recoverable hereunder.

As the terms are very clear and free from any doubt or ambiguity whatsoever, it must be
taken and understood in its plain, ordinary and popular sense pursuant to the above-cited
principle laid down by this Court.

It is apparent that Section 27 of the insurance policy was stipulated pursuant to Section
63 of the Insurance Code, which states that:

Sec. 63. A condition, stipulation or agreement in any policy of insurance, limiting


the time for commencing an action thereunder to a period of less than one year
from the time when the cause of action accrues, is void.

WHEN DOES CAUSE OF ACTION ACCRUE?

In support of private respondent's view, two rulings of this Court have been cited, namely,
the case of Eagle Star Insurance Co. vs. Chia Yu (96 Phil. 696 (1955]), where the Court
held:
18

The right of the insured to the payment of his loss accrues from the happening of
the loss. However, the cause of action in an insurance contract does not accrue
until the insured's claim is finally rejected by the insurer. This is because before
such final rejection there is no real necessity for bringing suit.

and the case of ACCFA vs. Alpha Insurance & Surety Co., Inc. (24 SCRA 151 [1968],
holding that:

Since "cause of action" requires as essential elements not only a legal right of the
plaintiff and a correlated obligation of the defendant in violation of the said legal
right, the cause of action does not accrue until the party obligated (surety) refuses,
expressly or impliedly, to comply with its duty (in this case to pay the amount of the
bond).

Indisputably, the above-cited pronouncements of this Court may be taken to mean


that the insured's cause of action or his right to file a claim either in the Insurance
Commission or in a court of competent jurisdiction commences from the time of
the denial of his claim by the Insurer, either expressly or impliedly.

2. But as pointed out by the petitioner insurance company, the rejection referred to should
be construed as the rejection, in the first instance, for if what is being referred to is a
reiterated rejection conveyed in a resolution of a petition for reconsideration, such should
have been expressly stipulated.

While in the Eagle Star case (96 Phil. 701), this Court uses the phrase "final rejection",
the same cannot be taken to mean the rejection of a petition for reconsideration as
insisted by respondents. Such was clearly not the meaning contemplated by this Court.
The Insurance policy in said case provides that the insured should file his claim, first, with
the carrier and then with the insurer. The "final rejection" being referred to in said case is
the rejection by the insurance company.
19

Case Title

Fortune Medicare Inc. vs. David Robert U. Amorin, G.R.


No. 195872, March 12, 2014

Topic Rules of Construction

Digested By CASANOVA, Jenny Mae I.

Characters and Their Roles

Petitioner: Fortune Medicare, Inc.

Respondent: David Robert U. Amorin

Facts:

David Robert U. Amorin (Amorin) was a cardholder/member of Fortune Medicare, Inc.


(Fortune Care), a corporation engaged in providing health maintenance services to its
members. The terms of Amorin’s medical coverage were provided in a Corporate Health
Program Contract (Health Care Contract) which was executed on January 6, 2000 by
Fortune Care and the House of Representatives, where Amorin was a permanent
employee.

Amorin underwent an emergency surgery, specifically appendectomy, at the St. Francis


Medical Center in Honolulu, Hawaii. The said surgery caused him to incur professional
and hospitalization expenses of US$7,242.35 and US$1,777.79, respectively. He
attempted to recover from Fortune Care the full amount thereof upon his return to Manila,
but the company merely approved a reimbursement of P12,151.36, an amount that was
based on the average cost of appendectomy if the procedure were performed in an
accredited hospital in Metro Manila.

Consequently, Amorin received under protest the approved amount, but asked for its
adjustment to cover the total amount of professional fees which he had paid, and eighty
percent (80%) of the approved standard charges based on “American standard,”
considering that the emergency procedure occurred in the U.S.A. However, Fortune Care
denied his request, prompting the latter to file a complaint for breach of contract with
damages with the Regional Trial Court.
20

Issue: Whether or not Fortune Care is liable to pay the total amount of professional fees
which Amorin had paid, and eighty percent (80%) of the approved standard charges
based on the “American standard”.

Ruling:

Yes. The Court agrees with the CA’s finding that Fortune Care’s liability to Amorin under
the subject Health Care Contract should be based on the expenses for hospital and
professional fees which he actually incurred, and should not be limited by the amount that
he would have incurred had his emergency treatment been performed in an accredited
hospital in the Philippines.

The Court emphasizes that for purposes of determining the liability of a health care
provider to its members, jurisprudence holds that a health care agreement is in the nature
of nonlife insurance, which is primarily a contract of indemnity. Once the member incurs
hospital, medical or any other expense arising from sickness, injury or other stipulated
contingent, the health care provider must pay for the same to the extent agreed upon
under the contract.

In the instant case, the extent of Fortune Care’s liability to Amorin under the attendant
circumstances was governed by Section 3(B), Article V of the subject Health Care
Contract, considering that the appendectomy which the member had to undergo qualified
as an emergency care, but the treatment was performed at St. Francis Medical Center in
Honolulu, Hawaii, U.S.A., a non-accredited hospital.

B. EMERGENCY CARE IN NON-ACCREDITED HOSPITAL

xxx

However, if the emergency confinement occurs in foreign territory, Fortune Care


will be obligated to reimburse or pay eighty (80%) percent of the approved
standard charges which shall cover the hospitalization costs and professional fees.

As may be gleaned from the Health Care Contract, the parties thereto contemplated the
possibility of emergency care in a foreign country. As the contract recognized Fortune
Care’s liability for emergency treatments even in foreign territories, it expressly limited its
liability only insofar as the percentage of hospitalization and professional fees that must
be paid or reimbursed was concerned, pegged at a mere 80% of the approved standard
charges. Contrary to Fortune Care’s argument, from nowhere in the Health Care Contract
could it be reasonably deduced that these “standard charges” referred to the “Philippine
standard,” or that cost which would have been incurred if the medical services were
performed in an accredited hospital situated in the Philippines.
21

Therefore, the Court held that the amount payable by Fortune Care should not be limited
to the cost of treatment in the Philippines, as to do so would result in the clear
disadvantage of its member. In the event that the premium and other charges in the
Health Care Contract were merely computed on assumption and risk under Philippine
cost and, that the American cost standard or any foreign country’s cost was never
considered, such limitations should have been distinctly specified and clearly reflected in
the extent of coverage which the company voluntarily assumed.
22

Case Title
DBP Pool Accredited Insurance Companies vs. Radio
Mindanao Network, Inc. 480 SCRA 314 (2006)

Topic Excepted Risk

Digested By CONDA, Ana Ultima G.

Parties:

● DBP Pool of Accredited Insurance Co. – Petitioner/Insurer


o Provident is also an insurer of RMN

● Radio Mindanao Network [RMN] – Respondent/Insured

Facts:

1. RMN owns several broadcasting stations all over the country. DBP Pool of
Accredited Insurance covered RMN’s transmitter, furniture, fixture and other
transmitter facilities for the amount of P5,883,650.00 under a Fire Insurance Policy.
[Provident, another insurer, covered RMN’s transmitter equipment and generating
set for the amount of P13,550,000.00]
2. Sometime in July 1988, the radio station of RMN in Bacolod City was razed by fire
causing damage in the amount of P1,044,040.00.
3. RMN sought recovery under the two insurance policies, but the claims were denied
on the ground the cause of loss was excepted risk as provided in the contract.

“This insurance does not cover any loss or damage occasioned by or through or
in consequence, directly or indirectly, of any of the following consequences,
namely: war, invasion, mutiny, riot, rebellion, revolution among others.”

4. Contention of DBP Pool Insurance: The insurance companies maintained that


the evidence showed that the fire was caused by members of the Communist Party
of the Philippines/New People’s Army (CPP/NPA) that is why the claims for
recovery was denied;

Issue: WON Radio Mindanao Network is entitled to the fire insurance claims

Discussion: Yes.
23

The SC ruled that DBP Pool failed to prove that the loss was caused by an excepted risk.
An insurance contract, being a contract of adhesion, should be so interpreted as to carry
out the purpose for which the parties entered into the contract which is to insure against
risks of loss or damage to the goods. Limitations of liability should be regarded with
extreme jealousy and must be construed in such a way as to preclude the insurer from
noncompliance with its obligations.

While a provision in the insurance contract states that it is the duty or burden of proof of
the insured to show that the loss or damage is covered by the policy, the Court held that
the burden of proof still rests upon DBP Pool Insurance to prove that the damage or loss
was caused by an excepted risk in order to escape any liability under the contract.

Particularly, in insurance cases, where a risk is excepted by the terms of a policy which
in sures against other perils or hazards, loss from such a risk constitutes a defense which
the insurer may urge, since it has not assumed that risk, and from this it follows that an
insurer seeking to defeat a claim because of an exception or limitation in the policy has
the burden of proving that the loss comes within the purview of the exception or limitation
set up. If a proof is made of a loss apparently within a contract of insurance, the burden
is upon the insurer to prove that the loss arose from a cause of loss which is excepted or
for which it is not liable, or from a cause which limits its liability. Consequently, it is
sufficient for RMN to prove the fact of damage or loss.

In this case, the evidence on record did not sufficiently establish the identity of the
author of the damage – that it was indeed NPA members nor was it an act of rebellion,
insurrection or usurped power. Hence, DBP failed to discharge their responsibility to
present adequate proof that the loss or damage was due to a risk excluded.
24

Case Title

Philippine Health Care Provider Inc. vs. Comm of


Internal Revenue, 600 SCRA 413 (2009)

Topic Health care agree

Digested By DOLOR, Rhodilee Jean A.

Characters and Their Roles

Petitioner: Philippine Health Care Provider (PHCR), a Health Maintenance


Organization (HMO)

Respondent: Commissioner of Internal Revenue (demanding payment of


deficiency tax from petitioner)

FACTS:

1. Petitioner Philippine Health Care Provider is a domestic corporation whose


primary purpose is:

"[t]o establish, maintain, conduct and operate a prepaid group practice health care
delivery system or a health maintenance organization to take care of the sick and
disabled persons enrolled in the health care plan and to provide for the
administrative, legal, and financial responsibilities of the organization."

2. Individuals enrolled in its health care programs pay an annual membership fee and
are entitled to various preventive, diagnostic and curative medical services
provided by its duly licensed physicians, specialists and other professional
technical staff participating in the group practice health delivery system at a
hospital or clinic owned, operated or accredited by it.
25

3. Respondent commissioner of internal revenue (CIR) demanded that the Philippine


Health Care Providers, Inc. pay the deficiency [documentary stamp tax (DST)]
assessment imposed on petitioner’s health care agreement with the members of
its health care program. CIR argues that petitioner PHCR’s health care agreement
was a contract of insurance subject to DST under Section 185 of the 1997 Tax
Code.

4. Petitioner argues that the DST under Section 185 of the National Internal Revenue
of 1997 is imposed only on a company engaged in the business of fidelity bonds
and other insurance policies. Petitioner, as an HMO, is a service provider, not an
insurance company.

ISSUES:

1. Whether or not PHCR as a Health Maintenance Organization (HMO) is engaged


in the business of insurance
2. Whether or not the health care agreement is an insurance contract

RULING:

No.

Applying the "principal object and purpose test," there is significant American case law
supporting the argument that a corporation (such as an HMO, whether or not organized
for profit), whose main object is to provide the members of a group with health services,
is not engaged in the insurance business.

Lastly, it is significant that the petitioner, as an HMO, is not part of the insurance industry.
This is evident from the fact that it is not supervised by the Insurance Commission but by
the Department of Health.

1. Whether or not the health care agreement is an insurance contract

No. It is not an insurance contract within the context of our Insurance Code.

Section 2 (1) of the Insurance Code defines a contract of insurance as an agreement


whereby one undertakes for a consideration to indemnify another against loss, damage
26

or liability arising from an unknown or contingent event. An insurance contract exists


where the following elements concur:

1. The insured has an insurable interest;


2. The insured is subject to a risk of loss by the happening of the designed peril;
3. The insurer assumes the risk;
4. Such assumption of risk is part of a general scheme to distribute actual losses
among a large group of persons bearing a similar risk and
5. In consideration of the insurer’s promise, the insured pays a premium.

Do the agreements between PHCR and its members possess all these elements?

1. Even if a contract contains all the elements of an insurance contract, if its primary
purpose is the rendering of service, it is not a contract of insurance.

2. There is nothing in petitioner's agreements that gives rise to a monetary liability on the
part of the member to any third party-provider of medical services which might in turn
necessitate indemnification from petitioner. The terms "indemnify" or "indemnity"
presuppose that a liability or claim has already been incurred. There is no indemnity
precisely because the member merely avails of medical services to be paid or already
paid in advance at a pre-agreed price under the agreements.

3. According to the agreement, a member can take advantage of the bulk of the benefits
anytime, e.g. laboratory services, x-ray, routine annual physical examination and
consultations, vaccine administration as well as family planning counseling, even in the
absence of any peril, loss or damage on his or her part.

4. In case of emergency, petitioner is obliged to reimburse the member who receives


care from a non-participating physician or hospital. The assumption of the expense by
petitioner is not confined to the happening of a contingency but includes incidents even
in the absence of illness or injury.

5. Petitioner’s objective is to provide medical services at reduced cost, not to distribute


risk like an insurer.
27

Case Title
Phil. Refining Co. vs. Palomar, 148 SCRA 313

Topic Nature of Lottery

Digested By DOMINGO, Angel Jean N.

Characters and Their Roles

Petitioner: PHILIPPINE REFINING CO., INC.

Respondent: HON. ENRICO PALOMAR

FACTS:

This is an appeal from the decision of the Court of First Instance of Manila in Civil Case
No. 72498, 1 entitled "Philippine Refining Company v. Hon. Enrico Palomar, finding that
plaintiff-appellee's promotion schemes ("Breeze Easy Money" and "CAMIA Lucky-
Key Hunt") were not in the nature of a lottery and enjoining appellant from issuing
a "fraud order" on the aforementioned schemes of appellee

It appears that the Philippine Refining Company, herein appellee, resorted to two
schemes to promote the sale of its products: Breeze Easy Money and CAMIA Lucky-Key
Hunt, both of which envisioned the giving away for free of certain prizes (without
additional consideration) for the purchase of Breeze soap and CAMIA cooking oil.

In other words, the participants would get the exact value of the price for the goods plus
the chance of winning in the scheme. No one would be required to pay more than the
usual price of the products.

ISSUE:

Whether or not the promotional schemes employed were in the nature of a lottery.
28

RULING:

NO, The Honorable Supreme Court ruled that a plan whereby prizes can be
obtained without any additional consideration (when a product is purchased) is not
a lottery. It is thus clear that the schemes in the case at bar are not lotteries. The
allegation that the prohibition by the Postmaster General should have first been appealed
to the Department Secretary concerned in view of the doctrine denominated as "the
exhaustion of administrative remedies" has no application here because one recognized
exception to the doctrine is when the issue raised is purely a legal one. In view of the
foregoing, the Court RESOLVED to DISMISS the appeal and to AFFIRM the assailed
decision of the Court of First Instance.

Additional Info:

Lottery means any scheme for the disposal or distribution of property by chance among
persons who have paid or promised to pay any valuable consideration for the chance of
obtaining property, or portion of it, or for any share or any interest in property, upon any
agreement, understanding, or expectation that it is to be distributed or disposed of by lot
or chance, whether called a lottery, raffle, or gift enterprise, or by whatever name it is
known.

The three essential elements of a lottery are: First, consideration; second, prize; and
third, chance.
29

Case Title
Zenith Insurance Corp. vs. Court of Appeals, 185 SCRA
398

Topic Award of Moral and Exemplary Damages and


Deductible Franchises in Motor Vehicle Insurance
Claims

Digested By: ABONAL, Maria Lou Isabel P.

Characters and Their Roles

Petitioner: Zenith Insurance Corp.

Respondent: Court of Appeals (Private Respondent: Lawrence Fernandez)

FACTS:

Lawrence Fernandez insured his car for “own damage” under private care policy with
Zenith Insurance Corporation. The car figured an accident and suffered actual damages
in the amount of P3,640.00. After allegedly being given a run around by Zenith for 2
months, Fernandez filed a complaint with the RTC Cebu for a sum of money and damages
resulting from the refusal of Zenith to pay the amount claimed. Aside from actual damages
and interests, Fernandez also prayed for moral damages of P10,000.00, exemplary
damages of P5,000.00, attorney’s fees of P3,000.00 and litigation expenses of P3,000.00.
The RTC ruled in favor of Fernandez and awarded the amount prayed for as actual
damages but it incurred interest at the rate of twice the prevailing interest rates. Attorney’s
fees and litigation expenses were also awarded in the same amount as that prayed for
by Fernandez. However, the RTC increased the awards for moral and exemplary
damages, from P10,000.00 to P20,000.00 and from P5,000.00 to P20,000.00
respectively. The CA affirmed the RTC’s decision in toto. In its Petition, Zenith argues
that the CA has no legal basis in awarding the moral damages, exemplary damages and
attorney’s fees in an amount more than that prayed for in the complaint.

ISSUE: 1. Whether or not the award of moral damages, exemplary damages and
attorney’s fees was proper.
30

2. Whether or not the deductible franchise and 20% depreciation on the parts
are allowed in motor vehicle insurance claims.

RULING: 1. Moral and Exemplary Damages - NO.

Attorney’s Fees - YES.

As to moral and exemplary the award was improper while as to the attorney’s fees it was
proper.

Under Section 244 of the Insurance Code, in case of unreasonable delay in the payment
of the proceeds of an insurance policy, the damages that may be awarded are: 1)
attorney’s fees; 2) other expenses incurred by the insured person by reason of such
unreasonable denial or withholding of payment; 3) interest at twice the ceiling prescribed
by the Monetary Board of the amount of the claim due the injured; and 4) the amount of
the claim.

As regards the award of moral and exemplary damages, the rules under the Civil Code
of the Philippines shall govern. In awarding moral damages in case of breach of contract,
there must be a showing that the breach was wanton and deliberately injurious or the one
responsible acted fraudulently or in bad faith. In the instant case, Fernandez was given a
“run-around” for 2 months, which is the basis for the award of the damages granted under
the Insurance Code for unreasonable delay in the payment of claim.

However, the act of Zenith of delaying payment for two months cannot be considered as
so wanton or malevolent to justify an award of P20,000.00 as moral damages, given that
the actual damage on the car was only P3,460.00. The real reason was that the parties
could not come to an agreement as regards the amount of the actual damage on the car,
hence, the P10,000.00 prayed for by Fernandez is equitable. As to the exemplary
damages, the insurance company had not acted in wanton, oppressive or malevolent
manner, hence, award cannot be given. On one hand, the amount of P5,000.00 awarded
as attorney’s fees is justified considering that there were other petitions filed and
defended by Fernandez in connection with this case. Lastly, the actual damages incurred
by Fernandez in the amount of P3,640.00 plus interest of twice the ceiling prescribed by
the Monetary Board computed from the time of submission of proof of loss had been
established before the trial court and affirmed by the appellate court. Furthermore,

2. NO.

Deduction of deductible franchise and 20% depreciation on parts are not allowed in motor
vehicle insurance claims.—As regards the actual damages incurred by private
31

respondent, the amount of P3,640.00 had been established before the trial court and
affirmed by the appellate court. Respondent appellate court correctly ruled that the
deductions of P250.00 and P274.00 as deductible franchise and 20% depreciation on
parts, respectively claimed by petitioners as agreed upon in the contract, had no basis.
Furthermore, the policy does not mention any deductible franchise.

Therefore, the award of moral damages is reduced to P10,000.00 and the award of
exemplary damages is hereby deleted. The awards due to private respondent Fernandez
are as follows: P3,640.00 as actual claim plus interest of twice the ceiling prescribed by
the Monetary Board computed from the time of submission of proof of loss; P10,000.00
as moral damages; P5,000.00 as attorney’s fees; P3,000.00 as litigation expenses; and
Costs.
32

Case Title
Blue Cross Health Care vs. Olivares, 544 SCRA 580
(2008)

Topic Rules of Construction, Limitations on Liability:


Construed strictly against the insurer;

Digested By BARROSO, Jon Carlow L.

Characters and Their Roles

Petitioner: Blue Cross Health Care

Respondent: Neomi Olivares

FACTS:

Neomi Olivares applied for a health care program with Blue Cross for the amount of
12,000 pesos. 38 days after she applied, she suffered from a stroke. Ailments due to “pre-
existing conditions” were excluded from the coverage. She was confined in Medical City
and discharged with a bill of Php 34,000. Blue Cross refused to pay unless she had her
physician’s certification that she was suffering from a pre-existing condition. When Blue
Cross still refused to pay, she filed suit in the MTC. The health care company rebutted by
saying that the physician didn’t disclose the condition due to the patient’s invocation of
the doctor-client privilege. The MTC dismissed for a lack of cause of action because the
physician didn’t disclose the condition. In the RTC, the spouses were awarded the amount
of the hospital bills plus 60,000 in damages. This was under the ratio that the burden to
prove that Neomi had a pre-existing condition was under Blue Cross. The CA denied the
motion for reconsideration of the health care company.

ISSUE:

1. Whether petitioner was able to prove that respondent Neomi's stroke was caused by a
pre-existing condition and therefore was excluded from the coverage of the health care
agreement.

2. Whether it was liable for moral and exemplary damages and attorney's fees.
33

RULING:

1. No, the petitioner was not able to prove that the respondent Neomi’s stroke was caused
by a pre-existing condition. In the case of “Philamcare Health Systems, Inc. v. CA” it was
held that a health care agreement is in the nature of a non-life insurance. It is an
established rule in insurance contracts that when their terms contain limitations on liability,
they should be construed strictly against the insurer. These are contracts of adhesion the
terms of which must be interpreted and enforced stringently against the insurer which
prepared the contract. This doctrine is equally applicable to health care agreements.”

The agreement defined a pre-existing condition as:

“a disability which existed before the commencement date of membership whose natural
history can be clinically determined, whether or not the Member was aware of such illness
or condition. Such conditions also include disabilities existing prior to reinstatement date
in the case of lapse of an Agreement.”

“Under this provision, disabilities which existed before the commencement of the
agreement are excluded from its coverage if they become manifest within one year from
its effectivity.”

Petitioners still averred that the non-disclosure of the pre-existing condition made a
presumption in its favor. Respondents still maintained that the petitioner had the duty to
prove its accusation.

Petitioner never presented evidence to prove its presumption that the Doctor’s report
would work against Neomi. They only perceived that the invocation of the privilege made
the report adverse to Neomi and such was a disreputable presumption. They should have
made an independent assessment of Neomi’s condition when it failed to obtain the report.
They shouldn’t have waited for the attending physician’s report to come out.

Section 3 (e), Rule 131 of the Rules of Court states:

Under the rules of court, Rule 131, Sec. 3.

Disputable presumptions. ― The following presumptions are satisfactory if


uncontradicted, but may be contradicted and overcome by other evidence:

(e) That evidence willfully suppressed would be adverse if produced.

The exception on presenting evidence applies when the suppression is an exercise of a


privilege.
34

Hence, Neomi had the privilege not to present the Doctor’s report under the doctor-client
privilege.

2. The court quoted the CA and RTC decision stating that “ the refusal of petitioner to pay
respondent Neomi's bills smacks of bad faith, as its refusal [was] merely based on its own
perception that a stroke is a pre-existing condition.” Also, there was factual bases in the
RTC and CA for the award of the damages.
35

Case Title

Rizal Surety & Insurance Company vs. Court of Appeals,


336 SCRA 12 (2000

Topic Construction/Interpretation

Digested By: BUSTAMANTE, Anne Georgette A.

Characters and Their Roles

Petitioner: Transworld Knitting Mills, Inc

Respondent: Rizal Surety and Insurance Co

Doctrine: Terms in an insurance policy, which are ambiguous, equivocal or uncertain are
to be construed strictly and most strongly against the insurer

FACTS:Rizal Surety & Insurance Company (Rizal Insurance) issued Fire Insurance
Policy in favor of Transworld Knitting Mills, Inc. (Transworld), initially for One Million
(P1,000,000.00) Pesos and eventually increased to One Million Five Hundred Thousand
(P1,500,000.00) Pesos, covering the period from August 14, 1980 to March 13, 1981.

Pertinent portions of subject policy on the buildings insured, and location thereof, read:

‘On stocks of finished and/or unfinished products, raw materials and supplies of every
kind and description, the properties of the Insureds and/or held by them in trust, on
commission or on joint account with others and/or for which they (sic) responsible in case
of loss whilst contained and/or stored during the currency of this Policy in the
premises occupied by them forming part of the buildings situate (sic) within own
Compound at MAGDALO STREET, BARRIO UGONG, PASIG, METRO MANILA,
PHILIPPINES, BLOCK NO. 601.

Fire broke out in the compound of Transworld, razing the middle portion of its four-span
building and partly gutting the left and right sections thereof. A two-storey building (behind
said four-span building) where fun and amusement machines and spare parts were
stored, was also destroyed by the fire.
36

Rizal Insurance contended that the fire insurance policy litigated upon protected only the
contents of the main building (four-span), and did not include those stored in the two-
storey annex building.

Transworld theorized that the so called “annex” was not an annex but was actually an
integral part of the four-span building and therefore, the goods and items stored therein
were covered by the same fire insurance policy.

ISSUE:

Issue: Whether or not the fire insurance policy litigated upon protected only the contents
of the main building (four-span), and did not include those stored in the two-storey annex
building

RULING:

Ruling: No, Annex is included in the policy.

Resolution of the issue posited hinges on the proper interpretation of the stipulation in
subject fire insurance policy regarding its coverage. Settled is the rule that Terms in an
insurance policy, which are ambiguous, equivocal or uncertain are to be construed strictly
and most strongly against the insurer.

It can be gleaned unerringly that the fire insurance policy in question did not limit its
coverage to what was stored in the four-span building. Two requirements must concur in
order that the said fun and amusement machines and spare parts would be deemed
protected by the fire insurance policy under scrutiny, to wit:

"First, said properties must be contained and/or stored in the areas occupied by
Transworld and second, said areas must form part of the building described in the policy
xxx"

The two-storey building involved, a permanent structure which adjoins and


intercommunicates with the "first right span of the lofty storey building", formed part
thereof.

Verily, the "annex " was not an annex building but an integral and inseparable part of the
four-span building described in the policy and consequently, the machines and spare
parts stored therein were covered by the fire insurance in dispute.

Also, considering that the two-storey building aforementioned was already existing when
subject fire insurance policy contract was entered into, petitioner should have specifically
excluded the said two-storey building from the coverage of the fire insurance if minded to
37

exclude the same but if did not, and instead, went on to provide that such fire insurance
policy covers the products, raw materials and supplies stored within the premises of
respondent Transworld which was an integral part of the four-span building occupied by
Transworld, knowing fully well the existence of such building adjoining and
intercommunicating with the right section of the four-span building.

Indeed, the stipulation as to the coverage of the fire insurance policy under controversy
has created a doubt regarding the portions of the building insured thereby. Article 1377
of the New Civil Code provides:

"Art.1377. The interpretation of obscure words or stipulations in a contract shall


not favor the party who caused the obscurity"

Conformably, it stands to reason that the doubt should be resolved against the petitioner,
Rizal

Surety Insurance Company, whose lawyer or managers drafted the fire insurance policy
contract under scrutiny. Citing the aforecited provision of law in point, the Court in
Landicho vs. Government Service Insurance System, ruled:

"This is particularly true as regards insurance policies, in respect of which it is settled that
the 'terms in an insurance policy, which are ambiguous, equivocal, or uncertain x x x are
to be construed strictly and most strongly against the insurer, and liberally in favor of the
insured so as to effect the dominant purpose of indemnity or payment to the insured,
especially where forfeiture is involved' and the reason for this is that the 'insured usually
has no voice in the selection or arrangement of the words employed and that the language
of the contract is selected with great care and deliberation by experts and legal advisers
employed by, and acting exclusively in the interest of, the insurance company.
38

Case Title
General Corporation of the Philippines vs. Union
Insurance Society of Canton, G.R. No. L-2684,
September 14, 1950

Topic Marine Insurance

Digested By CAGAPE, Joanna Paula M.

Characters and Their Roles

Petitioner: General Corporation of the Philippines- domestic corp

Respondents: Union Insurance Insurance Society of Canton- foreign corp, settling


agent of Fireman’s Fund Insurance Co. in the Philippines

FACTS:

The parties:

General Corporation of the Philippines and the Mayon Investment Co. are domestic
corporations duly organized and existing by virtue of the laws of the Philippines, with
principal offices in the City of Manila. The Union Insurance Society of Canton, Ltd. is a
foreign insurance corporation, duly authorized to do business in the Philippines, with head
office in the City of Hongkong, China, and a branch office in Manila. The Fireman’s Fund
Insurance Co. is a foreign insurance corporation duly organized and existing under the
laws of the State of California, U. S. A. It has been duly registered with the Insurance
Commissioner of the Bureau of Commerce as such insurance company since November
7, 1946, and authorized to do business in the Philippines since that date.

The Union Insurance Society of Canton, Ltd. has been acting as settling agent of and
settling insurance claims against the Fireman’s Fund Insurance Co. even before the last
world war and continued as such at least up to November 7, 1946.
39

The controversy

Upon arrival of the merchandise in Manila the consignee or purchaser would appear to
have failed to meet the terms of the sale . Hence, pursuant to the agreement between the
shippers and the General Corporation, the shipping papers, including the twelve marine
insurance policies were surrendered to the General Corporation of the Philippines. The
merchandise released to them, the latter claiming that they had paid to the bank the full
invoice price. Some of the merchandise were lost and others damaged while in transit
and inasmuch as the policies were made payable to the order of the assured in Manila,
the plaintiffs filed the corresponding claims with the defendant Union Insurance Society
of Canton, Ltd. in Manila acting as settling agent of its co-defendant Fireman’s Fund
Insurance Co.

The General Corporation of the Philippines and the Mayon Investment Co. as plaintiffs
sued the Union Insurance Society of Canton, Ltd. and the Fireman’s Fund Insurance Co.
for the payment of twelve marine insurance policies in the sum of P57,137.60. Said
policies were issued by the Fireman’s Fund Insurance Co. for merchandise shipped from
the United States to the Philippines in 1945, in the name of Western Canvas Products
Company and/ or Rovan Trading Company, doing business in Seattle, Washington, U. S.
A.

Proceedings in the USA

All the claim papers with the exception of insurance policy No. 70448/6 (Exhibit E-2) for
$2,902.36 were forwarded to defendant Fireman’s Fund Insurance Co. at Seattle,
Washington, following instructions from the said company, and the claims there approved
by the insurance company. However, the claims were there adjudicated by the Superior
Court of the State of Washington for King County against the plaintiffs in the present case
and in favor of other claimants. As regards the claim based on insurance policy No.
70448/6, Exhibit E-2, involved in the present appeal, inasmuch as it was filed a little late,
it was not forwarded to the United States and so was never passed upon by the Fireman’s
Fund Insurance Co. at Seattle. It was not also approved or disapproved by the Union
Insurance Society of Canton, Ltd. in Manila.

Proceedings at the RTC

After hearing, said court found and held that as regards the eleven marine insurance
policies which have been the subject of interpleader in the Superior Court in the State of
Washington for King County and decided by said court against the herein plaintiffs, said
decision constituted res adjudicata binding upon the plaintiffs herein. The trial court
absolved the defendant Union Insurance Society of Canton, Ltd. from the complaint but
condemned the Fireman’s Fund Insurance Co. to pay the plaintiffs the sum of $2,000 or
40

its equivalent in Philippine currency, with legal interest from and including September 12,
1946, on the claim based on the marine insurance policy No. 70448/6, Exhibit E-2.

How the case reached the Supreme Court:

The plaintiffs General Corporation of the Philippines and Mayon Investment Co. appealed
from that part of the decision referring to the eleven marine insurance policies. Said
appeal is now docketed in the Supreme Court as G. R. No. L-2303. The Fireman’s Fund
Insurance Co. appealed from the decision in so far as it was sentenced to pay $2,000 to
the plaintiffs. Because of the amount involved the appeal was sent to the Court of
Appeals. However, being a companion case of G. R. No. L-2303, at the instance of
appellant, the case was finally elevated to the Supreme Court which gave it due course
by its resolution of December 9, 1948, and docketed here as G. R. No. L-2684. This is
the case on appeal now under consideration.

The appellant contends that the trial court erred in holding that it acquired jurisdiction over
appellant Fireman’s Fund Insurance Co. and in rendering judgment against it in the sum
of $2,000.

ISSUE:

Whether or not the Fireman’s Fund Insurance is doing insurance business in the
Philippines

RULING:

Yes. Fireman’s Fund Insurance Co. is doing insurance business.

It is a rule generally accepted that one single or isolated business transaction does not
constitute "doing business" within the meaning of the law, and that transactions which are
occasional, incidental and casual, not of a character to indicate a purpose to engage in
business do not constitute the doing or engaging in business contemplated by law. In
order that a foreign corporation may be regarded as doing business within a State, there
must be continuity of conduct and intention to establish a continuous business, such as
the appointment of a local agent, and not one of a temporary character. (Thompson on
41

Corporations, Vol. 8, 3d edition, pp. 844-847 and Fisher’s Philippine Law of Stock
Corporation, p. 415.)

The Honorable Supreme Court ruled that the Fireman’s Fund Insurance Co., is doing
insurance business in the Philippines. Specifically, the Fireman’s Fund Insurance Co
does the following:

1. Issues marine insurance policies covering different shipments that are


made payable in Manila.

2. Fireman’s Fund Insurance Co. appointed and kept a settling agent in the
Philippines

3. The issued policies are collectible by the consignees in Manila or such other
persons or entities who meet the terms by paying the amounts of the invoices

4. According to the evidence, since before the war, the Fireman’s Fund
Insurance Co. would appear to have engaged in this kind of business and had
employed its co-defendant Union Insurance Society of Canton, Ltd. as its settling
agent, although sometime in 1946, between July and August of that year, appellant
had its own employee from its head office in America, one John L. Stewart, acting
as its settling agent here.

5. On November 7, 1946, less than two months after service of summons, it


applied for, obtained a license and was authorized to regularly do business in the
Philippines.

As already stated, section 14, Rule 7 of the Rules of Court makes no distinction as to
corporations with or without authority to do business in the Philippines. The test is whether
a foreign corporation was actually doing business here. Otherwise, a foreign corporation
illegally doing business here because of its refusal or neglect to obtain the corresponding
license and authority to do business may successfully though unfairly plead such neglect
or illegal act so as to avoid service and thereby impugn the jurisdiction of the local courts.

In conclusion, a foreign corporation actually doing business in this jurisdiction, with or


without license or authority to do so, is amenable to process and the jurisdiction of local
courts. If such foreign corporation has a license to do business, then summons to it will
42

be served on the agent designated by it for the purpose, or otherwise in accordance with
the provisions of the Corporation Law. Where such foreign corporation actually doing
business here has not applied for license to do so and has not designated an agent to
receive summons, then service of summons on it will be made pursuant to the provisions
of the Rules of Court, particularly Rule 7, section 14 thereof. We further hold that where
a foreign insurance corporation engages in regular marine insurance business here by
issuing marine insurance policies abroad to cover foreign shipments to the Philippines,
said policies being made payable here, and said insurance company appoints and keeps
an agent here to receive and settle claims flowing from said policies, then said foreign
corporation will be regarded as doing business here in contemplation of law.
43

Case Title
Lorenzo Shipping vs. Chubb and Sons, G.R. No.
147724, June 8, 2004

Topic Marine Insurance

Digested By: CASANOVA, Jenny Mae I.

Characters and Their Roles

Petitioner: Lorenzo Shipping Corp

Respondent: Chubb and Sons, Inc.

FACTS:

Mayer Steel Pipe Corporation of Binondo, Manila, loaded 581 bundles of ERW black steel
pipes on board the vessel M/V Lorcon IV, owned by petitioner Lorenzo Shipping, for
shipment to Davao City. Petitioner Lorenzo Shipping issued a clean bill of lading for the
account of the consignee, Sumitomo Corporation which in turn, insured the goods with
respondent Chubb and Sons, Inc.

Subsequently, the M/V Lorcon IV arrived at the Sasa Wharf in Davao City. When
Respondent Transmarine Carriers received the subject shipment it found out that the
steel pipes were submerged in seawater in the hatch of the said vessel. As a result, the
consignee Sumitomo then hired the services of R.J. Del Pan Surveyors to inspect the
shipment and they discovered that the subject shipment was no longer in good condition,
as in fact, the pipes were found with rust formation. Moreover, the surveyor observed that
the cargo hold of the M/V Lorcon IV was flooded with seawater, and the tank top was
“rusty, thinning, and with several holes at different places.

While the cargo was being shipped by respondent Gearbulk from Davao City to the
U.S.A., consignee Sumitomo sent a letter of intent to petitioner Lorenzo Shipping to inform
the latter that it will be filing a claim based on the damaged cargo once such damage had
been assessed. Due to the cargo’s heavily rusted condition, the consignee Sumitomo
rejected the damaged steel pipes and declared them unfit for the purpose they were
44

intended. It then filed a marine insurance claim with respondent Chubb and Sons, Inc.
which the latter settled in the amount of US$104,151.00. Subsequently, respondent
Chubb and Sons, Inc. filed a complaint for collection of a sum of money against
respondents Lorenzo Shipping, Gearbulk, and Transmarine. Hence, this petition.

ISSUE:

Whether or not Lorenzo Shipping can be held liable for the payment of damages for its
negligence in carrying the cargo.

RULING:

Yes. Lorenzo Shipping can be held liable for the payment of damages for its negligence
in carrying the cargo.

The Court held that petitioner Lorenzo Shipping was negligent in its care and custody of
the consignee’s goods. The steel pipes, subject of this case, were in good condition when
they were loaded at the port of origin (Manila) on board petitioner Lorenzo Shipping’s M/V
Lorcon IV en route to Davao City. As evidence, Petitioner Lorenzo Shipping issued clean
bills of lading covering the subject shipment. A bill of lading, aside from being a contract
and a receipt is also a symbol of the goods covered by it. A bill of lading which has no
notation of any defect or damage in the goods is called a “clean bill of lading.” A clean bill
of lading constitutes prima facie evidence of the receipt by the carrier of the goods as
therein described.

In the present case, M/V Lorcon IV of petitioner Lorenzo Shipping received the steel pipes
in good order and condition, evidenced by the clean bills of lading it issued. When the
cargo was unloaded from petitioner Lorenzo Shipping’s vessel at the Sasa Wharf in
Davao City, the steel pipes were rusted all over. Furthermore, R.J. Del Pan Surveyors
found that the cargo hold of the M/V Lorcon IV was flooded with seawater, and the tank
top was rusty, thinning and perforated, thereby exposing the cargo to sea water. There
can be no other conclusion than that the cargo was damaged while on board the vessel
of petitioner Lorenzo Shipping, and that the damage was due to the latter’s negligence.

Therefore, the Court held that the negligence of petitioner was sufficiently established
since it failed to keep its vessel in seaworthy condition and it is liable to pay the insurance
company.
45

Case Title
Geagonia vs. Court of Appeals, 241 SCRA 152 [1995]

Topic Over - insurance

Digested By DOMINGO, Angel Jean N.

Characters and Their Roles

Petitioner: ARMANDO GEAGONIA

Respondent: COURT OF APPEALS and COUNTRY BANKERS INSURANCE


CORPORATION

Facts:

Geagonia, owner of a store, obtained from Country Bankers fire insurance policy for
P100,000.00. The 1 year policy covered the stock trading of dry goods.

The policy contained the following condition:

3. The insured shall give notice to the Company of any insurance or


insurances already affected, or which may subsequently be effected,
covering any of the property or properties consisting of stocks in trade,
goods in process and/or inventories only hereby insured, and unless such
notice be given and the particulars of such insurance or insurances be
stated therein or endorsed in this policy pursuant to Section 50 of the
Insurance Code, by or on behalf of the Company before the occurrence of
any loss or damage, all benefits under this policy shall be deemed
forfeited, provided however, that this condition shall not apply when the
total insurance or insurances in force at the time of the loss or damage is
not more than P200,000.00.
46

The petitioners’ stocks were destroyed by fire. He then filed a claim which was
subsequently denied because the petitioner’s stocks were covered by two
other fire insurance policies for Php 200,000 issued by PFIC. The basis of
the private respondent’s denial was the petitioner’s alleged violation of Condition
3 of the policy.

Geagonia then filed a complaint against the private respondent in the Insurance
Commission for the recovery of P100,000.00 under fire insurance policy and
damages.

He claimed that he knew the existence of the other two policies. But, he
said that he had no knowledge of the provision in the private respondent’s
policy requiring him to inform it of the prior policies and this requirement
was not mentioned to him by the private respondent’s agent.

The Insurance Commission found that the petitioner did not violate Condition
3 as he had no knowledge of the existence of the two fire insurance policies
obtained from the PFIC; that it was Cebu Tesing Textiles w/c procured the
PFIC policies w/o informing him or securing his consent; and that Cebu
Tesing Textile, as his creditor, had insurable interest on the stocks.

The Insurance Commission then ordered the respondent company to pay the
complainant the sum of P100,000.00 with interest and attorney’s fees.
CA reversed the decision of the Insurance Commission because it found that
the petitioner knew of the existence of the two other policies issued by the PFIC.

Issue:

1. WON the petitioner had not disclosed the two insurance policies when he obtained the
fire insurance and thereby violated Condition 3 of the policy.

2. WON he is prohibited from recovering

Ruling:

1. YES, the petitioner did not disclose


2. No, he is not prohibited from recovering

Rationale:

The court agreed with the CA that the petitioner knew of the prior policies issued by the
PFIC. His letter of 18 January 1991 to the private respondent conclusively proves this
47

knowledge. His testimony to the contrary before the Insurance Commissioner and which
the latter relied upon cannot prevail over a written admission made ante litem motam. It
was, indeed, incredible that he did not know about the prior policies since these
policies were not new or original.

Stated differently, provisions, conditions or exceptions in policies which tend to work a


forfeiture of insurance policies should be construed most strictly against those for whose
benefits they are inserted, and most favorably toward those against whom they are
intended to operate.

With these principles in mind, Condition 3 of the subject policy is not totally free from
ambiguity and must be meticulously analyzed. Such analysis leads us to conclude that
(a) the prohibition applies only to double insurance, and (b) the nullity of the policy
shall only be to the extent exceeding P200,000.00 of the total policies obtained.

Furthermore, by stating within Condition 3 itself that such condition shall not apply if the
total insurance in force at the time of loss does not exceed P200,000.00, the private
respondent was amenable to assume a co-insurer’s liability up to a loss not
exceeding P200,000.00.

What it had in mind was to discourage over-insurance. Indeed, the rationale behind
the incorporation of “other insurance” clause in fire policies is to prevent over-
insurance and thus avert the perpetration of fraud. When a property owner obtains
insurance policies from two or more insurers in a total amount that exceeds the property’s
value, the insured may have an inducement to destroy the property for the purpose of
collecting the insurance. The public as well as the insurer is interested in preventing a
situation in which a fire would be profitable to the insured.
48

Case Title
Lalican vs. Insular Life Insurance Co. Limited, 597
SCRA 159 (2009)

Topic Insurable Interest; Reinstatement

Digested By: GALEA, Samantha Racel Jeanne R.

Parties:

● Petitioner: Violeta Lalican (Widow of Insured)


● Respondent: Insular Life Insurance Co. Limited (Insurer)

Facts:

During his lifetime, Eulogio Lalican applied for an insurance policy with Insular, to which
Policy No. 9011992 containing a 20-Year Endowment Variable Income Package Flexi
Plan worth P500,000.00, with two riders valued at P500,000.00 each, was issued. Eulogio
was to pay the premiums on a quarterly basis, with a grace period of 31 days for the
payment of each premium subsequent to the first. While Eulogio paid for July and October
1997, he failed to pay January 1998 even after the lapse of the grace period of 31 days.
Therefore, Policy No. 9011992 lapsed and became null and void. In September 1998,
Eulogio submitted an Application for Reinstatement and included payments for the
overdue interest on the premium assessed since January 1998, as well as the premiums
for April and July. All these were submitted to Malaluan, the agent. However, it was only
Malaluan’s husband who received the same because the agent was away on a business
errand. Eulogio died that same day of cardio-respiratory arrest secondary to electrocution.
Malaluan, without knowing of Eulogio’s death, forwarded the application and payments to
Insular’s regional office in San Fernando. However, Insular no longer acted upon it
because it was informed that Eulogio had already died.

Violata, the widow of Eulogio, filed a claim for death benefits against Insular Life
Assurance Company Ltd. (Insular). RTC dismissed Violeta’s claim and denied Petitioner’s
Notice of Appeal by way of Rule 41, and declared its decision final.

Issue:
49

WON Eulogio still had insurable interest in his own life when he reinstated the subject
Policy just before he passed away on 17 September 1998

Ruling:

No. An insurable interest is one of the most basic and essential requirements in an
insurance contract. In general, an insurable interest is that interest which a person is
deemed to have in the subject matter insured, where he has a relation or connection with
or concern in it, such that the person will derive pecuniary benefit or advantage from the
preservation of the subject matter insured and will suffer pecuniary loss or damage from
its destruction, termination, or injury by the happening of the event insured against. The
existence of an insurable interest gives a person the legal right to insure the subject matter
of the policy of insurance. Section 10 of the Insurance Code indeed provides that every
person has an insurable interest in his own life. Section 19 of the same code also states
that an interest in the life or health of a person insured must exist when the insurance
takes effect, but need not exist thereafter or when the loss occurs.

In the present case, the Honorable Supreme Court ruled that Policy No. 9011992 had
already lapsed and become void on 24 February 1998, upon the expiration of the 31-day
grace period for payment of the premium, which fell due on 24 January 1998, without any
payment having been made. In fact, Eulogio's filing of his first Application for
Reinstatement with Insular Life, through Malaluan, on 26 May 1998, constitutes an
admission that Policy No. 9011992 had lapsed by then. Insular Life did not act on
Eulogio's first Application for Reinstatement, since the amount Eulogio simultaneously
deposited was sufficient to cover only the P8,062.00 overdue premium for 24 January
1998, but not the P322.48 overdue interests thereon.

To reinstate a policy means to restore the same to premium-paying status after it has
been permitted to lapse. Both the Policy Contract and the Application for Reinstatement
provide for specific conditions for the reinstatement of a lapsed policy.

The Policy Contract between Eulogio and Insular Life identified the following conditions
for reinstatement should the policy lapse:

10. REINSTATEMENT

You may reinstate this policy at any time within three years after it lapsed if the following
conditions are met: (1) the policy has not been surrendered for its cash value or the period
of extension as a term insurance has not expired; (2) evidence of insurability satisfactory
to [Insular Life] is furnished; (3) overdue premiums are paid with compound interest at a
rate not exceeding that which would have been applicable to said premium and
indebtedness in the policy years prior to reinstatement; and (4) indebtedness which
existed at the time of lapsation is paid or renewed.

Additional conditions for reinstatement of a lapsed policy were stated in the Application
for Reinstatement which Eulogio signed and submitted, to wit: I/We agree that said Policy
shall not be considered reinstated until this application is approved by the Company
50

during my/our lifetime and good health and until all other Company requirements for the
reinstatement of said Policy are fully satisfied.

I/We further agree that any payment made or to be made in connection with this
application shall be considered as deposit only and shall not bind the Company until this
application is finally approved by the Company during my/our lifetime and good health. If
this application is disapproved, I/We also agree to accept the refund of all payments made
in connection herewith, without interest, and to surrender the receipts for such payment.

In the instant case, Eulogio's death rendered impossible full compliance with the
conditions for reinstatement of Policy. Citing Andres v. The Crown Life Insurance
Company, the Honorable Supreme Court discussed that, The stipulation in a life
insurance policy giving the insured the privilege to reinstate it upon written application
does not give the insured absolute right to such reinstatement by the mere filing of an
application. The insurer has the right to deny the reinstatement if it is not satisfied as to
the insurability of the insured and if the latter does not pay all overdue premium and all
other indebtedness to the insurer. After the death of the insured the insurance Company
cannot be compelled to entertain an application for reinstatement of the policy because
the conditions precedent to reinstatement can no longer be determined and satisfied.

Therefore, since the conditions for reinstatement under the Policy Contract and
Application for Reinstatement were written in clear and simple language, which could not
admit of any meaning or interpretation other than those that they so obviously embody,
Eulogio's death, just hours after filing his Application for Reinstatement and depositing his
payment for overdue premiums and interests with Malaluan, does not constitute a special
circumstance to already consider Policy reinstated. Said circumstance cannot override
the clear and express provisions of the Policy Contract and Application for Reinstatement,
and operate to remove the prerogative of Insular Life thereunder to approve or disapprove
the Application for Reinstatement.
51

Case Title
Gaisano Cagayan Inc. vs. Insurance Company of North
America (G.R. No. 147839, June 8, 2006)

Topic Insurable Interest and Right of Subrogation

Digested By: IRAN, Jayson O.

Parties:

Characters and Their Roles

Petitioner: Gaisano Cagayan Inc

Respondent: Insurance Company of North America

FACTS:

Petitioner is a customer and dealer of the products of Intercapitol Marketing Corporation


(IMC) IMC and Levi Strauss Phils. Inc. (LSPI). IMC and LSPI separately obtained from
respondent fire insurance policies with book debt endorsements. The insurance policies
provide for coverage on “book debts in connection with ready-made clothing materials
which have been sold or delivered to various customers and dealers of the Insured
anywhere in the Philippines.

On February 25, 1991, the Gaisano Superstore Complex in Cagayan de Oro City, owned
by petitioner, was consumed by fire. Included in the items lost or destroyed in the fire
were stocks of ready-made clothing materials sold and delivered by IMC and LSPI. On
February 4, 1992, respondent filed a complaint for damages against the petitioner. It
alleges that IMC and LSPI filed with respondent their claims under their respective fire
insurance policies with book debt endorsements; that respondent paid the claims of IMC
and LSPI and, by virtue thereof, respondent was subrogated to their rights against
petitioner; that respondent made several demands for payment upon petitioner but these
went unheeded. Hence this petition.

ISSUE:

Whether or not IMC and LSPI have insurable interest and whether or not IMC and LSPI
is liable to the insurer?
52

RULING:

YES. In this case, what was insured against were the accounts of IMC and LSPI with the
petitioner which remained unpaid 45 days after the loss through fire, and not the loss or
destruction of the goods delivered. Thus, IMC and LSPI did not lose complete interest in
the goods. They have an insurable interest until full payment of the value of the delivered
goods. Unlike the civil law concept of res perit domino, where ownership is the basis for
consideration of who bears the risk of loss, in property insurance, one’s interest is not
determined by concept of title, but whether insured has substantial economic interest in
the property.

Under Article 1263 of the Civil Code, “[i]n an obligation to deliver a generic thing, the loss
or destruction of anything of the same kind does not extinguish the obligation.” If the
obligation is generic in the sense that the object thereof is designated merely by its class
or genus without any particular designation or physical segregation from all others of the
same class, the loss or destruction of anything of the same kind even without the debtor’s
fault and before he has incurred in delay will not have the effect of extinguishing the
obligation. This rule is based on the principle that the genus of a thing can never perish.
Genus nunquan perit. An obligation to pay money is generic; therefore, it is not excused
by fortuitous loss of any specific property of the debtor.

Thus, whether fire is a fortuitous event or petitioner was negligent are matters immaterial
to this case. What is relevant here is whether it has been established that the petitioner
has outstanding accounts with IMC and LSPI. Moreover, the subrogation receipt, by itself,
is sufficient to establish not only the relationship of respondent as insurer and IMC as the
insured, but also the amount paid to settle the insurance claim. The right of subrogation
accrues simply upon payment by the insurance company of the insurance claim.
Respondent’s action against petitioner is squarely sanctioned by Article 2207 of the Civil
Code which provides:

Art. 2207. If the plaintiff’s property has been insured, and he has received
indemnity from the insurance company for the injury or loss arising out of the wrong
or breach of contract complained of, the insurance company shall be subrogated
to the rights of the insured against the wrongdoer or the person who has violated
the contract.

In this case, the petitioner failed to refute respondent’s evidence. Therefore, the petitioner
is liable to the insurer.
53

Case Title
Ong Lim Sing Jr. vs. FGB Leasing Finance Corp. 524
SCRA 333 (2007)

Topic

Digested By: MACULBE, Tresha Hyacinth A.

Characters and Their Roles

Petitioner: JVL Food Products and Vicente Ong Lim Sing, Jr.

Respondent: FEB Leasing and Finance Corporation (FEB)

FACTS:

FEB Leasing and Finance Corporation (FEB) leased equipment and motor vehicles to JVL Food
Products with a monthly rental of P170,494. At the same date, Vicente Ong Lim Sing, Jr. (Lim)
executed an Individual Guaranty Agreement with FEB to guarantee the prompt and faithful
performance of the terms and conditions of the lease agreement.

However, JVL defaulted in the payment of the monthly rentals resulting to arrears of
P3,414,468.75 and refused to pay despite demands. As such, FEB filed a complaint for damages
and replevin against JVL, Lim and John Doe.

JVL and Lim admitted the existence of the lease agreement but asserted that it is in reality a sale
of equipment on installment basis, with FEB acting as the financier

The RTC ruled that the Sale is on installment and the FEB elected full payment of the obligation
so for the unreturned units and machineries the JVL and Lim are jointly and severally liable to
pay.
54

However, CA granted FEB appeal that it is a financial lease agreement under Republic Act (R.A.)
No. 8556 and ordered JVL and Lim jointly and severally to pay P3,414,468.75

ISSUE: WON JVL and Lim should jointly and severally be liable for the insured financial lease

RULING:

YES. A Contract of adhesion is as binding as any ordinary contract, hence JVL and Lim should
be jointly and severally liable.

The Lease Contract with corresponding Lease Schedules with Delivery and Acceptance
Certificates is, in point of fact, a financial lease within the purview of R.A. No. 8556.

FEB leased the subject equipment and motor vehicles to JVL in consideration of a monthly
periodic payment of P170,494.00. The periodic payment by petitioner is sufficient to amortize at
least 70% of the purchase price or acquisition cost of the said movables in accordance with the
Lease Schedules with Delivery and Acceptance Certificates.

JVL entered into the lease contract with full knowledge of its terms and conditions. Lim, as a
lessee, has an insurable interest in the equipment and motor vehicles leased. In the financial
lease agreement, FEB did not assume responsibility as to the quality, merchantability, or capacity
of the equipment. This stipulation provides that, in case of defect of any kind that will be found
by the lessee in any of the equipment, recourse should be made to the manufacturer.

The financial lessor, being a financing company, i.e., an extender of credit rather than an ordinary
equipment rental company, does not extend a warranty of the fitness of the equipment for any
particular use. Thus, the financial lessee was precisely in a position to enforce such warranty
directly against the supplier of the equipment and not against the financial lessor and JVL and
Lim should be held jointly and severally liable.
55

Case Title
Cha vs. Court of Appeals, 277 SCRA 690 (1997)

Topic Measure of Insurable Interest

Digested By: MAGUINDANAO, Da'ud A.

Characters and Their Roles

Petitioner: Spouses Cha

Respondent: CA and CKS

FACTS:

Spouses Cha leased a property with CKS as lessor. On their contract, it was provided
that Sps Cha cannot insure their own merchandise against fire except with written consent
and approval from CKS. However, Sps. Cha still took an insurance against fire on their
merchandise inside the leased property. A fire broke out, and CKS learned that Sps Cha
procured an insurance policy. CKS demanded payment from United Insurance saying
that it should pay to CKS based on its lease contract with Cha Sps. United refused to pay
CKS.

ISSUE:

Whether or not CKS has the right to claim the proceeds of the fire insurance

RULING:

No. CKS is not the beneficiary of the fire insurance. Sec. 18 of the Insurance Code
provides:

Sec. 18. No contract or policy of insurance on property shall be enforceable except for
the benefit of some person having an insurable interest in the property insured.

A non-life insurance policy such as the fire insurance policy taken by petitioner-spouses
over their merchandise is primarily a contract of indemnity. Insurable interest in the
56

property insured must exist at the time the insurance takes effect and at the time the loss
occurs

In the present case, it cannot be denied that CKS has no insurable interest in the goods
and merchandise inside the leased premises under the provisions of Section 17 of the
Insurance Code which provide:

Sec. 17. The measure of an insurable interest in property is the extent to which the insured
might be damnified by loss of injury thereof.

Therefore, respondent CKS cannot, under the Insurance Code — a special law — be
validly a beneficiary of the fire insurance policy taken by the petitioner-spouses over their
merchandise. This insurable interest over said merchandise remains with the insured, the
Cha spouses. The automatic assignment of the policy to CKS under the provision of the
lease contract previously quoted is void for being contrary to law and/or public policy.
57

Case Title
San Miguel Brewery vs. Law Union & Rock Ins. Co., 40
Phil. 674 [1920]

Topic Insurable Interest

Digested By: MAYANGAO, Ian Jermie O.

Characters and Their Roles

Plaintiff: San Miguel Brewery, Inc

Defendant: Law Union And Rock Insurance Co., and Henry Harding

FACTS:

On Jan. 12, 1918, D.P. Dunn mortgaged a parcel of land to SMB to secure a debt of
10,000.00 Mortgage contract stated that Dunn was to have the property insured at his
own expense, authorizing SMB to choose the insurers and to receive the proceeds
thereof and retain so much of the proceeds as would cover the mortgage debt.

Dunn likewise authorized SMB to take out the insurance policy for him. Brias, SMB’s
general manager, approached Law Union for insurance to the extent of 15,000.00 upon
the property. In the application, Brias stated that SMB’s interest in the property was
merely that of a mortgagee.

Law Union, not wanting to issue a policy for the entire amount, issued one for P7,500 and
procured another policy of equal amount from Filipinas Cia de Seguros. Both policies
were issued in the name of SMB only and contained no reference to any other interests
in the propty. Both policies required assignments to be approved and noted on the policy.

Premiums were paid by SMB and charged to Dunn. A year later, the policies were
renewed.

In 1917, Dunn sold the property to Henry Harding, but no assignment of the policies was
made to the latter.

Property was destroyed by fire. SMB filed an action in court to recover on the policies.
Harding was made a defendant because by virtue of the sale, he became the owner of
the property, although the policies were issued in SMB’s name.
58

SMB sought to recover the proceeds to the extent of its mortgage credit with the balance
to go to Harding.

Insurance Companies contended that they were not liable to Harding because their
liability under the policies was limited to the insurable interests of SMB only.

SMB eventually reached a settlement with the insurance companies and was paid the
balance of it’s mortgage credit. Harding was left to fend for himself. Trial court ruled
against Harding. Hence the appeal.

ISSUE:

Whether or not the insurance companies are liable to Harding for the balance of the
proceeds of the 2 policies.

RULING:

No. Under the Insurance Act, the measure of insurable interest in the property is the
extent to which the insured might be daminified by the loss or injury thereof. Also it is
provided in the Insurance Act that the insurance shall be applied exclusively to the proper
interest of the person in whose name it is made. Undoubtedly, SMB as the mortgagee of
the property, had an insurable interest therein; but it could NOT, an any event, recover
upon the two policies an amount in excess of its mortgage credit.

In Section 19 of the Insurance Act we find it stated that "a change of interest in any part
of a thing insured unaccompanied by a corresponding change of interest in the insurance,
suspends the insurance to an equivalent extent, until the interest in the thing and the
interest in the insurance are vested in the same person."

Again in section 55 it is declared that "the mere transfer of a thing insured does not
transfer the policy, but suspends it until the same person becomes the owner of both the
policy and the thing insured."

By virtue of the Insurance Act, neither Dunn nor Harding could have recovered from the
two policies. With respect to Harding, when he acquired the property, no change or
assignment of the policies had been undertaken. The policies might have been worded
differently so as to protect the owner, but this was not done.
59

Case Title
Malayan Insurance Company, Inc. vs. PAP Co., Ltd.
(2013), G.R. No. 200784, August 7, 2013

Topic Fire Insurance; Concealment and Misrepresentation

Digested By: MENDOZA, Juan Antonio A.

Characters and Their Roles

Petitioner: Malayan Insurance Company, Inc.

Respondent: PAP Co., Ltd. (Phil. Branch)

FACTS:

On May 13, 1996, Malayan issued a Fire Insurance Policy to PAP for PAP’s machineries
and equipment located at Sanyo Building.

The insurance was worth 15 million pesos and effective for one (1) year. It was procured
by PAP for Rizal Commercial Banking Corporation (RCBC), the mortgagee of the insured
equipment and machineries.

After nearly a year but prior to the expiration of the insurance coverage, PAP renewed
the policy on an “as is” basis. A renewal policy was issued by Malayan to PAP for the
period of May 13, 1997 to May 13, 1998.

On October 12, 1997, and during the subsistence of the renewal policy, the insured
machineries and equipment were totally lost by fire. PAP filed a fire insurance claim with
Malayan in the amount insured.

In a letter, Malayan denied the claim arguing that at the time of the loss, the insured
machineries and equipment were transferred by PAP to a different location from that
indicated in the policy. The insured machineries were transferred in September 1996 from
the Sanyo Building to the Pace Pacific Building.
60

PAP countered that Malayan cannot avoid liability as it was informed of the transfer by
RCBC, a party duty-bound to relay such information. However, Malayan reiterated its
denial of PAP’s claim.

PAP then filed a complaint against Malayan. The RTC ruled in favor of PAP. It ordered
Malayan to pay an indemnity for the loss under the fire insurance policy (15 million pesos
worth) as well as for attorney’s fees (500 thousand).

The RTC explained that:

1. Malayan is liable because although there was a change in the condition of the thing
insured as a result of the transfer, Malayan failed to show proof that such transfer
resulted in the increase of the risk insured against. The RTC argued that in the
absence of such proof, the contract of fire insurance is not affected per Article 169
of the Insurance Code.
2. PAP’s notice to RCBC sufficiently complied with the notice requirement under the
policy, considering that it was RCBC which procured the insurance. PAP acted in
good faith in notifying RCBC about the transfer and the latter even conducted an
inspection of the machinery in its new location.

Malayan appealed to the CA, arguing that it should not be held liable because PAP
transferred the machineries and equipment to a location different to that in the policy,
without notice and/or their consent.

The CA ruled in favor of PAP. It explained that:

1. Malayan is liable for the whole 15 million but not for the attorney’s fees.
2. Malayan failed to prove that there was a prohibition on the transfer of the insured
properties during the efficacy of the insurance policy.
3. Malayan also failed to show that its contractual consent was needed before
transferring the insured properties.
4. Despite claiming that the original and renewed policies contained provisions on
transfer limitations, Malayan never cited the specific provisions.
5. Even if there was such a provision on transfer restrictions, Malayan cannot escape
liability because the transfer was made during the effectivity of the original policy.
PAP transferred the properties sometime in September 1996. Therefore, Malayan
was aware or should have been aware of such transfer when it issued the renewal
policy. The CA opined that since an insurance policy is a contract of adhesion, any
ambiguity must be resolved against the party that prepared the contract, which, in
this case, was Malayan.
6. Malayan failed to show that the transfer increased the risk of the loss. Thus, it could
not use such transfer as an excuse to avoid liability. Although the insurance
61

proceeds were payable to RCBC, PAP could still sue Malayan to enforce its rights
on the policy because it remained a party to the insurance contract.

Malayan filed a petition for review with the SC.

Malayan argued that it cannot be held liable under the insurance contract because:

1. PAP committed concealment, misrepresentation, and breach of an affirmative


warranty under the renewal policy when it transferred the location of the insured
properties without informing Malayan.
2. The transfer affected the correct estimation of risk which should have enabled
Malayan to decide whether it was willing to assume the risk, and if so, at what rate
of premium.
3. The transfer also affected Malayan’s ability to control the risk by guarding against
the increase of the risk brought about by the change in conditions, specifically the
location.
4. PAP violated Secs. 27, 74, and 45.

"Section 27. A concealment whether intentional or unintentional entitles the injured


party to rescind a contract of insurance.”

"Section 45. If a representation is false in a material point, whether affirmative or


promissory, the injured party is entitled to rescind the contract from the time when
the representation becomes false.”

"Section 74. The violation of a material warranty, or other material provision of a


policy, on the part of either party thereto, entitles the other to rescind.”

PAP countered that:

1. There is no evidence of concealment, misrepresentation, or deception on its part


and that its claim is not fraudulent.
2. It can still sue to protect its rights and interest on the policy notwithstanding the
fact that the proceeds of the same was payable to RCBC, and that it can collect
interest at the rate of 12% per annum on the proceeds because its claim for
indemnity was unduly delayed without legal justification.

ISSUE:

1. Whether or not Malayan Insurance is liable for payment of the insurance.

RULING:
62

Malayan cannot be held liable.

The policy forbade the removal of the insured properties unless sanctioned by Malayan.
– Condition No. 9 (c) of the policy clearly and expressly provided that the removal of the
insured property to any building or place required the consent of Malayan. It added that
any transfer effected by the insured, without the insurer’s consent, would free the insurer
from any liability.

PAP failed to notify and to obtain the consent of Malayan regarding the removal of the
insured properties. – PAP tried to prove notice by showing that it relayed the fact of
transfer to RCBC, the entity which made the referral and the named beneficiary in the
policy. Malayan and RCBC might have been sister companies, but such fact did not make
one an agent of the other. The fact that RCBC referred PAP to Malayan did not clothe it
with authority to represent and bind the said insurance company. After the referral, PAP
directly dealt with Malayan.

The transfer from the Sanyo Factory to the Pace Factory increased the risk. – The transfer
to the Pace Factory exposed the properties to a hazardous environment and negatively
affected the fire rating stated in the renewal policy. The increase in tariff rate from 0.449%
to 0.657% put the subject properties at a greater risk of loss. Such increase would
necessarily entail an increase in the premium payment on the policy. PAP failed to refute
Malayan’s argument on the increased risk.

"Section 170. An alteration in the use or condition of a thing insured from that to
which it is limited by the policy made without the consent of the insurer, by means
within the control of the insured, and increasing the risks, entitles an insurer to
rescind a contract of fire insurance.”

"Section 171. An alteration in the use or condition of a thing insured from that to
which it is limited by the policy, which does not increase the risk, does not affect a
contract of fire insurance.”

Malayan was entitled to rescind the insurance contract. – Considering that the original
policy was renewed on an “as is basis,” it follows that the renewal policy carried with it the
same stipulations and limitations. The terms and conditions of the renewal policy
provided, among others, that the location of the risk insured against is at the Sanyo
Factory in PEZA. Although both factories were in PEZA, it was not the location stipulated
in the renewal policy. An international loss adjuster which investigated the fire ruled that
since the location is not the one in the policy, the policy will not respond to the loss/claim.
63

It can be argued that the transfer without notice and consent, after the renewal of the
policy, PAP clearly committed concealment, misrepresentation, and a breach of material
warranty.

"Section 26. A neglect to communicate that which a party knows and ought to
communicate, is called a concealment.”

"Section 27. A concealment whether intentional or unintentional entitles the injured


party to rescind a contract of insurance.”

Accordingly, an insurer can exercise its right to rescind an insurance contract when the
following conditions are present:

1. The policy limits the use or condition of the thing insured;


2. There is an alteration in said use or condition;
3. The alteration is without the consent of the insurer;
4. The alteration is made by means within the insured’s control; and
5. The alteration increases the risk of loss.

In the case at bar, all conditions were present.

WHEREFORE, the Decision of the CA was reversed and set aside. Malayan was
declared not liable for the loss of the insured properties.
64

Case Title
Saturnino vs. Phil. American Life Insurance Co., 7
SCRA 316 (1963)

Topic Non-medical Insurance; Concealment

Digested By: MIRAFUENTES, Charlotte P.

Characters and Their Roles

Petitioner: Heirs of Estefania Saturnino - Insurance beneficiaries

Respondent: Philippine American Life Insurance Company - Insurance Company

FACTS:

Estefania Saturnino applied for a 20-year endowment non-medical insurance. This kind
of policy dispenses with the medical examination of the applicant usually required in
ordinary life policies. However, detailed information is called for in the application
concerning the applicant's health and medical history. Here, Estefania submitted her
application on 16 November 1957 and was subsequently issued on the same day upon
payment of the premium.

On September 19, 1958, she died of pneumonia hence her heirs demanded for payment
of the face value of the policy.

However, it appears on record that two months prior to the issuance of the policy or on
September 9, 1957, Estefania was operated on for cancer, involving complete removal of
the right breast; stayed in the hospital for 8 days; and went through a surgery but could
not be considered cured since her disease was malignant in nature.

Despite these circumstances, she did not disclose it in her insurance application. On the
contrary, she stated that she did not have any disease listed in the application, included
among other things cancer; that she had not consulted any physician, undergone any
operation or suffered any injury for the past five years; and that she had never been
treated for nor did she ever have any illness or disease peculiar to her sex, particularly of
65

the breast. The application also recites that the foregoing declarations constituted "a
further basis for the issuance of the policy."

ISSUE: Whether or not the insurance policy should be rescinded due to material
representation and concealment

RULING: Yes, the insurance policy should be rescinded due to material representation
and concealment.

Section 30 of the Insurance Code provides that "materiality is to be determined not by


the event, but solely by the probable and reasonable influence of the facts upon the party
to whom the communication is due, in forming his estimate of the proposed contract, or
in making his inquiries."

In this case, there can be no dispute that the information given by Estefania in her
application for insurance was false, namely, that she had never had cancer or tumors, or
consulted any physician or undergone any operation within the preceding period of five
years.

The waiver of medical examination renders even more material the information required
of the Estefania concerning previous condition of health and diseases suffered, for such
information necessarily constitutes an important factor which PhilAm Life Insurance takes
into consideration in deciding whether to issue the policy or not. It is logical to assume
that if the insurer had been properly apprised of Estefania's medical history she would at
least have been made to undergo medical examination in order to determine her
insurability.

There was no negligence on the part of the insurer when no medical examination was
conducted precisely because Estefania had given herself a clean bill of health that she
no longer considered an actual medical checkup necessary.

There was concealment of the fact of the operation which Estefania went through. In the
first place the concealment of the fact of the operation itself was fraudulent, as there could
not have been any mistake about it, no matter what the ailment. Secondly, in order to
avoid a policy it is not necessary to show actual fraud on the part of the insured.

In this jurisdiction a concealment, whether intentional or unintentional, entitles the insurer


to rescind the contract of insurance, concealment being defined as "negligence to
communicate that which a party knows and ought to communicate" (Sections 24 & 26,
Act No. 2427).

"The basis of the rule vitiating the contract in cases of concealment is that it misleads or
deceives the insurer into accepting the risk, or accepting it at the rate of premium agreed
66

upon. The insurer, relying upon the belief that the assured will disclose every material fact
within his actual or presumed knowledge, is misled into a belief that the circumstance
withheld does not exist, and he is thereby induced to estimate the risk upon a false basis
that it does not exist."

Therefore, the insurance policy should be rescinded due to material representation and
concealment by the insured.
67

Case Title
Philamcare Health Systems, Inc., vs. Court Of Appeals
and Julita Trinos, G.R. No. 125678, March 18, 2002

Topic Insurance Contract; Concealment

Digested By: OLAER, Jezrel L.

Characters and Their Roles

Petitioner: Philamcare Health Systems, Inc.

Respondent: Julita Trinos

FACTS:

Ernani Trinos, deceased husband of respondent Julita Trinos, applied for a health care
coverage with petitioner Philamcare Health Systems, Inc. In the standard application
form, he answered no to the following question: Have you or any of your family members
ever consulted or been treated for high blood pressure, heart trouble, diabetes, cancer,
liver disease, asthma or peptic ulcer? (If Yes, give details).

The application was approved for a period of one year under which Julita’s husband was
entitled to avail of hospitalization benefits, whether ordinary or emergency, listed therein.
He was also entitled to avail of "out-patient benefits" such as annual physical
examinations, preventive health care and other out-patient services. And upon the
termination of the agreement, the same was extended for another year.

During the period of his coverage, Ernani suffered a heart attack and was confined at the
Manila Medical Center. However, her husband later on died. While her husband was in
the hospital, Julita tried to claim the benefits under the health care agreement. However,
Philamcare denied her claim saying that the Health Care Agreement was void alleging
that there was a concealment regarding Ernani’s medical history. Doctors at the MMC
allegedly discovered at the time of Ernani’s confinement that he was hypertensive,
diabetic and asthmatic, contrary to his answer in the application form.
68

Philamcare claimed that it granted benefits only when the insured is alive during the one-
year duration. It contended that there was no indemnification unlike in insurance
contracts. It supported this claim by saying that it is a health maintenance organization
covered by the DOH and not by the Insurance Commission. Lastly, it claimed that the
Incontestability clause does not apply because it requires an effectivity period of at least
two years.

ISSUE:

(1) Whether or not the health care agreement is not an insurance contract

(2) Whether or not there is concealment of material fact made by Ernani

RULING:

(1) NO. Section 2 (1) of the Insurance Code defines a contract of insurance as an
agreement whereby one undertakes for a consideration to indemnify another against loss,
damage or liability arising from an unknown or contingent event. An insurance contract
exists where the following elements concur: 1. The insured has an insurable interest; 2.
The insured is subject to a risk of loss by the happening of the designated peril; 3. The
insurer assumes the risk; 4. Such assumption of risk is part of a general scheme to
distribute actual losses among a large group of persons bearing a similar risk; and 5. In
consideration of the insurer’s promise, the insured pays a premium.

Section 3 of the Insurance Code states that any contingent or unknown event, whether
past or future, which may damnify a person having an insurable interest against him, may
be insured against. Every person has an insurable interest in the life and health of himself.
Section 10 provides:

Every person has an insurable interest in the life and health:

(1) of himself, of his spouse and of his children; xxx

In the case at bar, the insurable interest of respondent’s husband in obtaining the health
care agreement was his own health. The health care agreement was in the nature of
non-life insurance, which is primarily a contract of indemnity. Once the member
incurs hospital, medical or any other expense arising from sickness, injury or other
stipulated contingent, the health care provider must pay for the same to the extent agreed
upon under the contract.

(2) NO. The answer assailed by Philamcare was in response to the question relating to
the medical history of the applicant. This largely depends on opinion rather than fact,
especially coming from Julita’s husband who was not a medical doctor. Where matters
69

of opinion or judgment are called for, answers made in good faith and without
intent to deceive will not avoid a policy even though they are untrue.

The fraudulent intent on the part of the insured must be established to warrant rescission
of the insurance contract. Concealment as a defense for the health care provider or
insurer to avoid liability is an affirmative defense and the duty to establish such defense
by satisfactory and convincing evidence rests upon the provider or insurer. In any case,
with or without the authority to investigate, petitioner is liable for claims made under the
contract. Having assumed a responsibility under the agreement, petitioner is bound to
answer the same to the extent agreed upon. In the end, the liability of the health care
provider attaches once the member is hospitalized for the disease or injury covered by
the agreement or whenever he avails of the covered benefits which he has prepaid.

Being a contract of adhesion, the terms of an insurance contract are to be construed


strictly against the party which prepared the contract – the insurer. By reason of the
exclusive control of the insurance company over the terms and phraseology of the
insurance contract, ambiguity must be strictly interpreted against the insurer and liberally
in favor of the insured, especially to avoid forfeiture.
70

Case Title
Great Pacific Life Assn. Co. vs. Court of Appeals, 89
SCRA 543 (1979)

Topic Binding receipts; Concealment

Digested By: Olayan, Imma Coney P.

Characters and Their Roles

Petitioner: Great Pacific Life Assurance, Lapulapu D. Mondragon

Respondent: Ngo Hing

FACTS:

On March 14, 1957, Ngo Hing filed an application with Great Pacific Life Assurance
Company for a twenty-year endowment policy on the life of his one-year old daughter
Helen Go. Ngo Hing supplied the essential data which the Mondragon, the Branch
Manager of Pacific Life in Cebu wrote on the application form signed by Ngo Hing. Ngo
Hing paid the annual premium and was issued a binding deposit receipt.

On April 30, 1957, Pacific Life sent the Branch Manager Mondragon a letter disapproving
of the application because a 20-year endowment plan is not available for minors below
seven years old. The non-acceptance of the insurance plan was allegedly not
communicated to Ngo Hing.

Instead, on May 6, 1957, Mondragon wrote back Pacific Life strongly recommending the
approval of the 20-year endowment life insurance on the ground that Pacific Life is the
only insurance company not selling the 20-year endowment insurance plan to children,
pointing out that since 1954 the customers, especially the Chinese, were asking for such
coverage.

It was then that on May 28, 1957, the child Helen Go died of influenza. Ngo Hing was
denied the payment of the proceeds of the insurance.
71

ISSUE:

(1) Whether or not the binding deposit receipt constituted a temporary contract of the life
insurance

(2) Whether Ngo Hing concealed the state of health and physical condition of Helen Go,
which rendered void the binding deposit receipt

RULING:

(1) NO, the binding deposit receipt is intended to be merely a provisional or temporary
insurance contract.

It is merely an acknowledgment, on behalf of the company, that the latter’s branch office
had received from the applicant the insurance premium and had accepted the application
subject for processing by the insurance company; and that the latter will either approve
or reject the same on the basis of whether or not the applicant is “insurable on standard
rates.”

Since Pacific Life disapproved the insurance application of respondent Ngo Hing, the
binding deposit receipt in question had never become in force at any time. Upon this
premise, the binding deposit receipt is, manifestly, merely conditional and does not insure
outright.

Where an agreement is made between the applicant and the agent, no liability shall attach
until the principal approves the risk and a receipt is given by the agent. The acceptance
is merely conditional, and is subordinated to the act of the company in approving or
rejecting the application. Thus, in life insurance, a “binding slip” or “binding receipt” does
not insure by itself. Binding receipts are not contracts of insurance. They are temporary,
acknowledgement receipts.

(2) YES, Ngo Hing had deliberately concealed the state of health and physical condition
of his daughter Helen Go.

When Ngo Hing supplied the required essential data for the insurance application form,
he was fully aware that his one-year old daughter is typically a mongoloid child.

Such a congenital physical defect could never be ensconced nor disguised. Nonetheless,
Ngo Hing, in apparent bad faith, withheld the fact material to the risk to be assumed by
72

the insurance company. As an insurance agent of Pacific Life, he ought to know, as he


surely must have known, his duty and responsibility to supply such a material fact. Had
he divulged said significant fact in the insurance application form. Pacific Life would have
verified the same and would have had no choice but to disapprove the application outright.

The contract of insurance is one of perfect good faith (uberrima fides meaning good faith;
absolute and perfect candor or openness and honesty; the absence of any concealment
or deception, however slight not for the insured alone but equally so for the insurer.
Concealment is a neglect to communicate that which a party known and ought to
communicate. Whether intentional or unintentional, the concealment entitles the insurer
to rescind the contract of insurance.

Hence, no insurance contract was perfected between the parties with the noncompliance
of the conditions provided in the binding receipt, and concealment, as legally defined,
having been committed by Ngo Hing.

WHEREFORE, Pacific Life and Mondragon are absolved from civil liabilities. Pacific Life
is ordered to reimburse the annual premium paid by Ngo Hing.
73

Case Title
Sunlife Assur. Co. of Canada vs. Court of Appeals, 245
SCRA 268 (1995)

Topic Section 26 - Concealment

Digested By: RAFOLS, James Ray B.

Characters and Their Roles

Petitioner: SUNLIFE ASSURANCE COMPANY OF CANADA, Petitioner

Respondent: The Hon. COURT OF APPEALS and Spouses ROLANDO and


BERNARDA BACANI, Respondent

FACTS:

On April 15, 1986, Bacani procured a life insurance contract for himself from Sun Life. He
was issued a life insurance policy with double indemnity in case of accidental death. The
designated beneficiary was his mother, Bernarda.

On June 26, 1987, the insured died in a plane crash. Bernarda Bacani filed a claim with
Sun Life, seeking the benefits of the insurance. Sun Life conducted an investigation and
its findings prompted it to reject the claim.

Sun Life discovered that 2 weeks prior to his application, Bacani was examined and
confined at the Lung Center of the Philippines, where he was diagnosed for renal failure.
During his confinement, the deceased was subjected to urinalysis, ultrasonography and
hematology tests. He did not reveal such a fact in his application.

In its letter, Sun Life informed Bernarda that the insured did not disclose material facts
relevant to the issuance of the policy, thus rendering the contract of insurance voidable.
A check representing the total premiums paid in the amount of P10,172.00 was attached
to said letter.
74

Bernarda and her husband filed an action for specific performance against Sun Life. RTC
ruled for Bernarda holding that the facts concealed by the insured were made in good
faith and under the belief that they need not be disclosed. Moreover, it held that the health
history of the insured was immaterial since the insurance policy was "non-medical."

ISSUE:

Whether or not the beneficiary can claim despite the concealment.

RULING:

No. Section 26 of the Insurance Code is explicit in requiring a party to a contract of


insurance to communicate to the other, in good faith, all facts within his knowledge which
are material to the contract and as to which he makes no warranty, and which the other
has no means of ascertaining.

Materiality is to be determined not by the event, but solely by the probable and reasonable
influence of the facts upon the party to whom communication is due, in forming his
estimate of the disadvantages of the proposed contract or in making his inquiries (The
Insurance Code, Sec 31)

The terms of the contract are clear. The insured is specifically required to disclose to the
insurer matters relating to his health. The information which the insured failed to disclose
were material and relevant to the approval and the issuance of the insurance policy. The
matters concealed would have definitely affected the petitioner's action on his application,
either by approving it with the corresponding adjustment for a higher premium or rejecting
the same. Moreover, a disclosure may have warranted a medical examination of the
insured by the petitioner in order for it to reasonably assess the risk involved in accepting
the application.

Thus, "good faith" is no defense in concealment. The insured's failure to disclose the fact
that he was hospitalized for two weeks prior to filing his application for insurance, raises
grave doubts about his bonafides. It appears that such concealment was deliberate on
his part.
75

Case Title
Ng Zee vs. Asian Crusader Life Assurance Corp. 122
SCRA 461 [1983]

Topic Concealment

Digested By: RIZON, Justine M.

Characters and Their Roles

Petitioner, Beneficiary: Ng Gan Zee

Respondent, Insurer: Asian Crusader Life Assurance Corp.

Insured – Kwong Nam

FACTS:

On May 12, 1962, Kwong Nam applied for a 20-year endowment insurance on his life for
the sum of P20,000, with his wife Ng Gan Zee, as beneficiary. On the same day, Asian
Crusader upon receipt of the required premium, approved the application and issued the
corresponding policy.

On December 6, 1963, Kwong Nam died of cancer of the liver with metastasis. All
premiums were religiously paid at the time of his death.

Ng Gan Zee presented a claim to Asian Crusader for payment of the face value of the
policy and submitted the required proof of death.

Asian Crusader denied the claimed that the insured was guilty of misrepresentation when
the insured was examined in connection with his application for life insurance, he gave
the appellant's medical examiner false and misleading information as to his ailment and
previous operation. The alleged false statements given by Kwong Nam are as follows:

Operated on for a Tumor [mayoma] of the stomach. Claims that Tumor has been
associated with ulcer of stomach. Tumor taken out was hard and of a hen's egg
76

size. Operation was two [2] years ago in Chinese General Hospital by Dr. Yap.
Now, claims he is completely recovered.

Asian Crusader presented reports which showed that the insured was operated on for
“peptic ulcer” involving the excision of a portion of the stomach. It argued that a tumor,
“hard and of a hen’s egg size” was removed during said operation, constituted material
concealment

ISSUE:

Was Asian Crusader, because of insured aforesaid representation, misled or deceived


into entering into contract or in accepting the risk at the rate of the premium agreed upon?

RULING:

Section 27 of the Insurance Law [Act 2427] provides:

Sec. 27. Such party a contract of insurance must communicate to the other, in
good faith, all facts within his knowledge which are material to the contract, and
which the other has not the means of ascertaining, and as to which he makes no
warranty.

Thus, "concealment exists where the assured had knowledge of a fact material to
the risk, and honesty, good faith, and fair dealing requires that he should
communicate it to the assurer, but he designedly and intentionally withholds the
same."

Se. 27 requires that fraudulent intent on the part of the insured be established to entitle
the insurer to rescind the contract.

There was no fraudulent intent on the part of Kwong Nam in this case.

Kwong Nam had informed the appellant's medical examiner that the tumor for which he
was operated on was "associated with ulcer of the stomach." In the absence of evidence
that the insured had sufficient medical knowledge as to enable him to distinguish between
"peptic ulcer" and "a tumor", his statement that said tumor was "associated with ulcer of
the stomach, " should be construed as an expression made in good faith of his belief as
to the nature of his ailment and operation.
77

Even if the information communicated was imperfect from the viewpoint of a medical
expert, it was sufficient to induce Asian Crusader to make further inquiries about the
ailment and operation of the insured, pursuant to Section 32 of Insurance Law.

Section 32. The right to information of material facts may be waived either by
the terms of insurance or by neglect to make inquiries as to such facts where they
are distinctly implied in other facts of which information is communicated.

It has been held that where, upon the face of the application, a question appears to
be not answered at all or to be imperfectly answered, and the insurers issue a
policy without any further inquiry, they waive the imperfection of the answer and
render the omission to answer more fully immaterial.

If the ailment and operation of Kwong Nam was a fact material to the risk, Asian Crusader
or its medical examiner should have made further inquiries from the Chinese General
Hospital or require copies of the hospital records from the appellant before acting on the
application for insurance. The fact of the matter is that the defendant was too eager to
accept the application and receive the insured's premium.
78

Case Title
Argente Vs. West Coast Life Insurance Co, 51 Phil 275

Topic Concealment; Misrepresentation

Digested By: SUMAMBAN, Geraldine B.

Characters and Their Roles

Petitioner: Spouses Bernardo Argente and Vicenta de Ocampo - made concealments in


their application for joint insurance with West Coast Life Insurance Co.

Respondent: West Coast Life Insurance Co. - refused to pay the claim of Bernardo
Argente because the insurance was obtained through fraud and misrepresentation.

Doctrine: From the standpoint of the insurer. — A fact is material if the knowledge of it
would have a "probable and reasonable influence upon the insurer in assessing the risk
involved and in making or omitting further inquiries, and cause him either to reject the risk
or to accept it only at a higher premium rate or on different terms though that fact may not
even remotely contribute to the contingency upon which the insurer would become liable,
or in any wise affect the risk.

FACTS:

On February 9, 1925, Spouses Bernardo Argente and Vicenta de Ocampo signed an


application for joint insurance in the sum of P2,000.00 with West Coast Life Insurance
Co. Both applications, with the exception of the names and the signatures of the
applicants, were written by Jose Geronimo del Rosario, an agent for the West Coast Life
Insurance Co. But all the information contained in the applications was furnished by the
agent.

Pursuant to his application, Bernardo Argente was examined by Dr. Cesareo Sta. Ana, a
medical examiner for the West Coast Life Insurance Co. The result of such examination
was recorded in the Medical Examiner's Report, and with the exception of the signature
of Bernardo Argente, was in the hand-writing of Doctor Sta. Ana.But the information or
79

answers to the questions contained on the face of the Medical Examiner's Report were
furnished to the doctor by the applicant, Bernardo Argente.

Vicenta was also examined by Dr. Cesareo Sta. Ana at her residence in Manila. The
result of the medical examination, including, among other things, the answers given by
Vicenta de Ocampo to the questions propounded, to her by the physician, appears in the
Medical Examiner's Report.

On May 9, 1925, the spouses submitted an amended application increasing the amount
to P15,000.00. However, the policy was not delivered until July 2, 1925. Since more than
30 days had elapsed since the medical examination was conducted, the spouses were
required to file a certificate of health before the policy was delivered.

On November 18, 1925, Vicenta died of cerebral apoplexy hence Bernardo filed a claim.
However, after the insurance company conducted an investigation it was found out that
the answers given by the insured in their medical examination with regards to their health
and previous illnesses and medical attendance were untrue. Therefore, West Coast
Insurance did not pay the claim of Bernardo because the insurance was obtained through
fraud and misrepresentation.

The untruthfulness of the medical examinations were based on:


● the admission of Bernardo that he has not consulted or has been treated for any illness
or ailment within the last five years, even though he was confined in a hospital two
years ago, and
● the admission of Vicenta that her intake of beer is only in small amounts, she has not
suffered any disease of the brain or nervous system, and that she was in good health,
but it was found out that she was diagnosed with alcoholism and with psycho-neurosis.

Bernardo Argente, while readily conceding most of the facts herein narrated, yet alleges
that both he and his wife revealed to the company's physician, Doctor Sta. Ana, all the
facts concerning their previous illnesses and medical attendance, but that Doctor Sta.
Ana, presumably acting in collusion with the insurance agent, Jose Geronimo del Rosario,
failed to record them in the medical reports.

ISSUE:

Whether or not Bernardo and Vicenta made false representations in their application for
joint insurance with West Coast Life Insurance Co. making contract of insurance
rescinded
80

RULING: YES.
One ground for the rescission of a contract of insurance under the Insurance Act is "a
concealment," which in section 25 is defined as "A neglect to communicate that which a
party knows and ought to communicate."

In an action on a life insurance policy where the evidence conclusively shows that the
answers to questions 'concerning diseases were untrue, the truth or falsity of the answers
become the determining factor. If the policy was procured by fraudulent representations,
the contract of insurance apparently set forth therein was never legally existent. It can
fairly be assumed that had the true facts been disclosed by the assured, the insurance
would never have been granted

Concealment exists where the assured has knowledge of a fact material to the risk, and
honesty, good faith, and fair dealing requires that he should communicate it to the
assured, but he designedly and intentionally withholds the same. Another rule is that if
the assured undertakes to state all the circumstances affecting the risk, a full and fair
statement of all is required. The concealment must, in the absence of inquiries, be
not only material, but fraudulent, or the fact must have been intentionally withheld.
If a material fact is concealed by assured it is equivalent to a false representation
that it does not exist and that the essentials are the truth of the representations whether
they were intended to mislead and did the insurer accept them as true and act upon them
to his prejudice. So it is decided that under a stipulation voiding the policy for concealment
or misrepresentation of any material fact or if his interest is not truly stated or is other
than the sole and unconditional ownership the facts are unimportant that insured did not
intend to deceive or withhold information as to encumbrances even though no questions
were asked.

"The basis of the rule vitiating the contract in cases of concealment is that it misleads or
deceives the insurer into accepting the risk, or accepting it at the rate of premium agreed
upon. The insurer, relying upon the belief that the assured will disclose every material fact
within his actual or presumed knowledge, is misled into a belief that the circumstance
withheld does not exist, and he is thereby induced to estimate the risk upon a false basis
that it does not exist. The assurer in assuming a risk is entitled to know every material
fact of which the assured has exclusive or peculiar knowledge, as well as all material facts
which directly tend to increase the hazard or risk which are known by the assured, or
which ought to be or are presumed to be known by him. And a concealment of such facts
vitiates the policy.

In this case, Bernardo and Vicenta made concealments in their application for joint
insurance with West Coast Life Insurance Co. Hence, the contract is rescinded.
81

Case Title
Tan vs Court of Appeals, 174 SCRA 403 (1989)

Topic Section 48

Digested By: TUDIO, Gabriel L.

Characters and Their Roles

Petitioner: EMILIO TAN, JUANITO TAN, ALBERTO TAN and ARTURO TAN

Respondent: The Court of Appeals and The Philippine American Life Insurance
Company

FACTS:
Petitioners appeal from the Decision of the Insurance Commissioner dismissing herein
petitioners’ complaint against respondent Philippine American Life Insurance Company
for the recovery of the proceeds of Policy in the amount of P80,000.00.

On September 23, 1973, Tan Lee Siong, father of herein petitioners, applied for life
insurance in the amount of P80,000.00 with Philippine American Life Insurance Company.
Said application was approved and Policy No. 1082467 was issued effective November
6, 1973, with petitioners the beneficiaries thereof (Exhibit A).

On April 26, 1975, Tan Lee Siong died of hepatoma .Petitioners then filed with Philippine
American Life Insurance Company their claim for the proceeds of the life insurance policy.
However, in a letter dated September 11, 1975, Philippine American Life Insurance
Company denied petitioners’ claim and rescinded the policy by reason of the alleged
misrepresentation and concealment of material facts made by the deceased Tan Lee
Siong in his application for insurance. The premiums paid on the policy were thereupon
refunded.

Alleging that Philippine American Life Insurance Company’s refusal to pay them the
proceeds of the policy was unjustified and unreasonable, petitioners filed on November
27, 1975, a complaint against the former with the Office of the Insurance Commissioner.
82

The petitioners contend that the Philippine American Life Insurance Company no longer
had the right to rescind the contract of insurance as rescission must allegedly be done
during the lifetime of the insured within two years and prior to the commencement of
action.

ISSUE:
Whether or not the Insurer (Philippine American Life Insurance Company) is liable.

RULING:
NO. The so-called “incontestability clause” precludes the insurer from raising the
defenses of false representations or concealment of material facts insofar as health and
previous diseases are concerned if the insurance has been in force for at least two years
during the insured’s lifetime. The phrase “during the lifetime” found in Section 48 simply
means that the policy is no longer considered in force after the insured has died. The key
phrase in the second paragraph of Section 48 is “for a period of two years.”

The insurer has two years from the date of issuance of the insurance contract or of its last
reinstatement within which to contest the policy, whether or not, the insured still lives
within such period. After two years, the defenses of concealment or misrepresentation,
no matter how patent or well founded, no longer lie.

In this case, the policy was in force for a period of only 1 year and 5 months (September
23, 1973 to April 26, 1975), the insured died before the 2-year period had lapsed,
respondent company is not barred from proving that the policy is void ab initio by reason
of the insured’s fraudulent concealment or misrepresentation. Also, the evidence of the
respondent company shows that the insured was diabetic for 5 years. Because of the
concealment made by the deceased of his consultations and treatments for hypertension,
diabetes and liver disorders, respondent company was thus misled into accepting the risk
and approving his application. For as long as no adverse medical history is revealed in
the application form, an applicant for insurance is presumed to be healthy and physically
fit and no further medical investigation or examination is conducted by the respondent
company.

Thus, the insurer is not liable.


83

Case Title
Manila Bankers Life Ins. Vs. Aban, 702 SCRA 417

Topic Section 48

Digested By: ABONAL, Maria Lou Isabel P.

FACTS:

On July 3, 1993, Delia Sotero took out a life insurance policy from Manila Bankers Life
Insurance, designating respondent Cresencia P. Aban, her niece, as her beneficiary.

Petitioner issued the Insurance Policy, with a face value of P100,000.00, in Sotero’s favor
on August 30, 1993, after the requisite medical examination and payment of the insurance
premium.

On April 10, 1996,when the insurance policy had been in force for more than two years
and seven months, Sotero died. Respondent filed a claim for the insurance proceeds on
July 9, 1996. Petitioner, however, denied the claim and instead refunded the premiums
paid on the policy claiming that the policy was obtained by fraud, concealment and/or
misrepresentation. Petitioner also filed for rescission.

Respondent filed a Motion to Dismiss claiming that petitioner’s cause of action was barred
by prescription pursuant to Section 48 of the Insurance Code, which provides as follows:

“Whenever a right to rescind a contract of insurance is given to the insurer by any


provision of this chapter, such right must be exercised previous to the commencement of
an action on the contract.

After a policy of life insurance made payable on the death of the insured shall have been
in force during the lifetime of the insured for a period of two years from the date of its
issue or of its last reinstatement, the insurer cannot prove that the policy is void ab initio
or is rescindable by reason of the fraudulent concealment or misrepresentation of the
insured or his agent.”
84

ISSUE: Whether or not Manila Bankers Life Insurance Corporation can still rescind
the insurance contract

RULING:

NO. Manila Bankers Life Insurance Corporation can no longer rescind the insurance
contract.

Allegations of fraud, which are predicated on respondent’s alleged posing as Sotero and
forgery of her signature in the insurance application, are at once belied by the trial and
appellate courts’ finding that Sotero herself took out the insurance for herself. “Fraudulent
intent on the part of the insured must be established to entitle the insurer to rescind the
contract”. In the absence of proof of such fraudulent intent, no right to rescind arises.

Moreover, Section 48 of the Insurance Code provides that an insurer is given two years
– from the effectivity of a life insurance contract and while the insured is alive – to discover
or prove that the policy is void ab initio or is rescindable by reason of the fraudulent
concealment or misrepresentation of the insured or his agent. After the two-year period
lapses, or when the insured dies within the period, the insurer must make good on the
policy, even though the policy was obtained by fraud, concealment, or misrepresentation.

Section 48 regulates both the actions of the insurers and prospective takers of life
insurance. It gives insurers enough time to inquire whether the policy was obtained by
fraud, concealment, or misrepresentation; on the other hand, it forewarns scheming
individuals that their attempts at insurance fraud would be timely uncovered – thus
deterring them from venturing into such nefarious enterprise. At the same time, legitimate
policy holders are absolutely protected from unwarranted denial of their claims or delay
in the collection of insurance proceeds occasioned by allegations of fraud, concealment,
or misrepresentation by insurers, claims which may no longer be set up after the two-year
period expires as ordained under the law.

In this case, the records show that the insured died after the two-year period, hence, the
petitioner is already barred from proving that the policy is void ab initio by reason of
fraudulent concealment or misrepresentation.
85

Case Title
Coquia vs. Fieldmen’s Insurance Co., Inc., G.R. No. L-
23276, Nov. 29, 1968

Topic General Principles on Insurance; Stipulation pour


Autrui

Digested By: BARROSO, Jon Carlow L.

Characters and Their Roles

Petitioner: Carlito Coquia

Respondent: Fieldmen’s Insurance Co., Inc.

FACTS:

On December 1, 1961, Fieldmen;s Insurance Company, Inc. issued in favor of the Manila
Yellow Taxicab Co., Inc.from December 1, 1961 to December 1, 1962

A taxicab of Manila driven by Carlito Coquia, met a vehicular accident at Mangaldan,


Pangasinana and died in February 1962.

The insured filed a claim for P5,000 in which Fieldmen's replied with an offer to pay
P2,000 by way of compromise. The insured rejected it and countered with P4,000

September 18, 1962: Carlito's parents filed a complaint against the Company for
collection. The company pleaded lack of cause of action. RTC: ordered to pay the parents

ISSUE:

Whether or not there is a stipulation pour autrui that exempts the general rule that the
parents are not a party to the contract
86

RULING:

Ø Yes, there is a stipulation that the Company "will indemnify any authorized Driver who
is driving the Motor Vehicle" of the Insured and, in the event of death of said driver, the
Company shall, likewise, "indemnify his personal representatives."

Ø typical of contracts pour autrui, this character being made more manifest by the fact
that the deceased driver paid 50% of the corresponding premiums, which were deducted
from his weekly commissions

Ø expressly stipulated and declared that it shall be a condition precedent to any right of
action or suit upon this Policy that the award by such arbitrator, arbitrators or umpire of
the amount of the Company's liability hereunder if disputed shall be first obtained

Ø both parties from the inception of their dispute proceeded in entire disregard of the
provisions of the contract relating to arbitration

Ø conduct was as effective a rejection of the right to arbitrate

Ø Stipulation pour autrui: a contract or provision in a contract that confers a benefit on a


third-party beneficiary.
87

Case Title
Bonifacio Bros. Inc. vs Mora, G.R. No. L-20853, May 29,
196

Topic Privity of Contract

Digested By: BOISER, Virgilio IV A.

FACTS:

Enrique Mora mortgaged his Oldsmobile sedan car to HS Reyes Inc. with the condition
that Mora would insure the car with HS Reyes as beneficiary. The car was then insured
with State Insurance Company and the policy delivered to Mora. During the effectivity of
the insurance contract, the car figured in an accident. The company then assigned the
accident to an insurance appraiser for investigation and appraisal of the damage.

Mora without the knowledge and consent of HS Reyes, authorized Bonifacio Bros to fix
the car, using materials supplied by the Ayala Auto Parts Company. For the cost of Labor
and materials, Mora was billed P2,002.73. The bill was sent to the insurer’s appraiser.
The insurance company drew a check in the amount of the insurance proceeds and
entrusted the check to its appraiser for delivery to the proper party. The car was delivered
to Mora without the consent of HS Reyes, and without payment to Bonifacio Bros and
Ayala.

Upon the theory that the insurance proceeds should be directly paid to them, Bonifacio
and Ayala filed a complaint against Mora and the insurer with the municipal court for the
collection of P2,002.73. The insurance company filed its answer with a counterclaim for
interpleader, requiring Bonifacio and HS Reyes to interplead in order to determine who
has a better right to the proceeds.

ISSUE:

Whether or not there is privity of contract between Bonifacio and Ayala on one hand and
State Insurance on the other.
88

RULING:

NO.

It is fundamental that contracts take effect only between the parties thereto, except in
some specific instance provided by law where the contract contains some stipulation in
favor of a third person. Such stipulation is known as a stipulation pour autrui; or a provision
in favor of a third person not a party to the contract.

Under this doctrine, a third person is allowed to avail himself of a benefit granted to him
by the terms of the contract, provided that the contracting parties have clearly and
deliberately conferred a favor upon such person. Consequently, a third person NOT a
party to the contract has NO action against the parties thereto, and cannot generally
demand the enforcement of the same.

In the instant case the insurance contract does not contain any words or clauses to
disclose an intent to give any benefit to any repairmen or material men in case of repair
of the car in question. The parties to the insurance contract omitted such stipulation, which
is a circumstance that supports the said conclusion. On the other hand, the “loss payable”
clause of the insurance policy stipulates that “Loss, if any, is payable to H.S. Reyes, Inc.”
indicating that it was only the H.S. Reyes, Inc. which they intended to benefit.

A policy of insurance is a distinct and independent contract between the insured and
insurer, and third persons have no right either in a court of equity, or in a court of law, to
the proceeds of it, unless there be some contract of trust, expressed or implied, by the
insured and third person. In this case, no contract of trust, express or implied. In this case,
no contract of trust, expressed or implied exists. We, therefore, agree with the trial court
that no cause of action exists in favor of the appellants in so far as the proceeds of
insurance are concerned. The appellant’s claim, if at all, is merely equitable in nature and
must be made effective through Enrique Mora who entered into a contract with the
Bonifacio Bros Inc. This conclusion is deducible not only from the principle governing the
operation and effect of insurance contracts in general, but is clearly covered by the
express provisions of section 50 of the Insurance Act (now Sec. 53).

The policy in question has been so framed that “Loss, if any, is payable to H. S. Reyes,
Inc.” which unmistakably shows the intention of the parties.
89

Case Title
Rizal Commercial Banking Corporation vs. Court of
Appeals, G.R. Nos. 128833, April 20, 1998

Topic Insurable interest

Digested By: BUSTAMANTE, Anne Georgette A.

Characters and Their Roles

Petitioner: RCBC

Respondent: GUYO & Sons, Inc.

FACTS:

FACTS:

GOYU & Sons, Inc. (GOYU) obtained a credit facility from Rizal Commercial Banking
Corporation (RCBC). It executed a mortgage contract in favor of the bank wherein it was
expressly stipulated that GOYU will insure all the subject properties with an insurance
company approved by the bank and to endorse and deliver the policies to the bank.
GOYU, through Alchester Insurance, Agency, Inc., took insurance policies from Malayan
Insurance Company, Inc. (MICO), sister company of RCBC, and endorsed them in favor
of RCBC. Copies of the endorsements were sent and received by GOYU, RCBC and
MICO. GOYU continued to enjoy the benefits of the credit facilities extended to it by the
bank.

When GOYU’s factory buildings were gutted by fire, GOYU and RCBC filed separate
claims with MICO but were both denied because the policies were either attached or
claimed by other creditors.

GOYU then filed a complaint for specific performance and damages disowning the
endorsements or lack of authority of Alchester to prepare and issue said endorsements
in favor of RCBC.

Both RTC and CA ruled that endorsements do not bear the signature of any officer of
GOYU concluded that the endorsements favoring RCBC as defective.
90

ISSUE: Whether or not RCBC, as mortgagee, has any right over the insurance policies
taken by GOYU, the mortgagor, in case of the occurrence of loss.

HELD: YES.

It is settled that a mortgagor and a mortgagee have separate and distinct insurable
interests in the same mortgaged property, such that each one of them may insure the
same property for his own sole benefit. There is no question that GOYU could insure the
mortgaged property for its own exclusive benefit.

In the present case, although it appears that GOYU obtained the subject insurance
policies naming itself as the sole payee, the intentions of the parties as shown by their
contemporaneous acts, must be given due consideration in order to better serve the
interest of justice and equity.

It is to be noted that nine endorsement documents were prepared by Alchester in favor


of RCBC. Alchester would not have endorsed the policies to RCBC had it not been so
directed by GOYU. On equitable principles, particularly on the ground of estoppel, the
Court is constrained to rule in favor of mortgagor RCBC. RCBC, in good faith, relied upon
the endorsement documents sent to it as this was only pursuant to the stipulation in the
mortgage contracts. The Court finds such reliance to be justified under the circumstances
of the case. GOYU failed to seasonably repudiate the authority of the person or persons
who prepared such endorsements. Over and above this, GOYU continued, in the
meantime, to enjoy the benefits of the credit facilities extended to it by RCBC. After the
occurrence of the loss insured against, it was too late for GOYU to disown the
endorsements for any imagined or contrived lack of authority of Alchester to prepare and
issue said endorsements. If there had not been actually an implied ratification of said
endorsements by virtue of GOYU’s inaction in this case, GOYU is at the very least
estopped from assailing their operative effects. To permit GOYU to capitalize on its non-
confirmation of these endorsements while it continued to enjoy the benefits of the credit
facilities of RCBC which believed in good faith that there was due endorsement pursuant
to their mortgage contracts, is to countenance grave contravention of public policy, fair
dealing, good faith, and justice. Such an unjust situation, the Court cannot sanction.
Under the peculiar circumstances obtaining in this case, the Court is bound to recognize
RCBC’s right to the proceeds of the insurance policies if not for the actual endorsement
of the policies, at least on the basis of the equitable principle of estoppel.

GOYU cannot seek relief under Section 53 of the Insurance Code which provides that the
proceeds of insurance shall exclusively apply to the interest of the person in whose
name or for whose benefit it is made. The peculiarity of the circumstances obtained in
91

the instant case presents a justification to take exception to the strict application of said
provision, having been sufficiently established that it was the intention of the parties to
designate RCBC as the party for whose benefit the insurance policies were taken out.
This Court cannot over stress the fact that upon receiving its copies of the endorsement
documents prepared by Alchester, GOYU, despite the absence of its written conformity
thereto, obviously considered said endorsement to be sufficient compliance with its
obligation under the mortgage contracts since RCBC accordingly continued to extend the
benefits of its credit facilities and GOYU continued to benefit therefrom. Just as plain too
is the intention of the parties to constitute RCBC as the beneficiary of the various
insurance policies obtained by GOYU. The intention of the parties will have to be given
full force and effect in this particular case. The insurance proceeds may, therefore, be
exclusively applied to RCBC, which under the factual circumstances of the case, is truly
the person or entity for whose benefit the policies were clearly intended. Moreover, the
law’s evident intention to protect the interests of the mortgagee upon the mortgaged
property is expressed in Article 2127 of the Civil Code.

Lastly, the proceeds of the insurance policies endorsed to RCBC cannot be attached by
GOYU’s other creditors up to the extent of the GOYU’s outstanding obligation in RCBC’s
favor. To the extent of GOYU’s outstanding obligation with RCBC, allother insurance
policies are, therefore, to be released from attachment, garnishment, and levy by the
other creditors of GOYU. It is basic and fundamental that the first mortgagee has superior
rights over junior mortgagees or attaching creditors.
92

Case Title
Sun Insurance Office Ltd. vs. Court of Appeals, G.R.
No. 89741, March 13, 1991

Topic Prescription, Fire Insurance

Digested By: CAGAPE, Joanna Paula M.

Characters and Their Roles

Petitioner: Sun Insurance: insurer

Respondent: Emilio Tan

FACTS:

Emilio Tan took a P300,000.00 property insurance policy to cover his interest in the
electrical supply store of his brother housed in a building in Iloilo City on August 15, 1983.
After 4 days, the house was burned. Tan filed his claim for fire loss with Sun Insurance
Office on August 20, 1983 but Sun Insurance Office denied the claim on February 20,
1984. Tan sought reconsideration in April 1984 but the same was rejected. Tan kept on
seeking for reconsideration but Sun Insurance advised Tan’s counsel that the denial of
Tan’s claim remained unchanged (September 3, 1985.)

The Case

On November 20, 1985, Tan filed Civil Case No. 16817 with the Regional Trial Court of
Iloilo, Branch 27. Sun Insurance Company filed a motion to dismiss on the alleged ground
that the action had already prescribed. Said motion was denied in an order dated
November 3, 1987; and petitioner's motion for reconsideration was also denied in an order
dated January 14, 1988.
93

ISSUES:

1. Whether or not the filing of a motion for reconsideration interrupts the twelve (12)
months prescriptive period to contest the denial of the insurance claim; and

2. Whether or not the rejection of the claim shall be deemed final only if it contains words
to the effect that the denial is final

RULING:

1. No. The answer to the first issue is in the negative.

While it is a cardinal principle of insurance law that a policy or contract of insurance is to


be construed liberally in favor of the insured and strictly against the insurer company, yet,
contracts of insurance, like other contracts, are to be construed according to the sense
and meaning of the terms which the parties themselves have used. If such terms are clear
and unambiguous, they must be taken and understood in their plain, ordinary and popular
sense (Pacific Banking Corp. v. Court of Appeals, 168 SCRA 1 [1988]).

Condition 27 of the Insurance Policy, which is the subject of the conflicting contentions of
the parties, reads:

27. Action or suit clause — If a claim be made and rejected and an action or suit
be not commenced either in the Insurance Commission or in any court of
competent jurisdiction within twelve (12) months from receipt of notice of such
rejection, or in case of arbitration taking place as provided herein, within twelve
(12) months after due notice of the award made by the arbitrator or arbitrators or
umpire, then the claim shall for all purposes be deemed to have been abandoned
and shall not thereafter be recoverable hereunder.

As the terms are very clear and free from any doubt or ambiguity whatsoever, it must be
taken and understood in its plain, ordinary and popular sense pursuant to the above-cited
principle laid down by this Court.

It is apparent that Section 27 of the insurance policy was stipulated pursuant to Section
63 of the Insurance Code, which states that:

Sec. 63. A condition, stipulation or agreement in any policy of insurance, limiting


the time for commencing an action thereunder to a period of less than one year
from the time when the cause of action accrues, is void.
94

WHEN DOES CAUSE OF ACTION ACCRUE?

In support of private respondent's view, two rulings of this Court have been cited, namely,
the case of Eagle Star Insurance Co. vs. Chia Yu (96 Phil. 696 (1955]), where the Court
held:

The right of the insured to the payment of his loss accrues from the happening of
the loss. However, the cause of action in an insurance contract does not accrue
until the insured's claim is finally rejected by the insurer. This is because before
such final rejection there is no real necessity for bringing suit.

and the case of ACCFA vs. Alpha Insurance & Surety Co., Inc. (24 SCRA 151 [1968],
holding that:

Since "cause of action" requires as essential elements not only a legal right of the
plaintiff and a correlated obligation of the defendant in violation of the said legal
right, the cause of action does not accrue until the party obligated (surety) refuses,
expressly or impliedly, to comply with its duty (in this case to pay the amount of the
bond).

Indisputably, the above-cited pronouncements of this Court may be taken to mean


that the insured's cause of action or his right to file a claim either in the Insurance
Commission or in a court of competent jurisdiction commences from the time of
the denial of his claim by the Insurer, either expressly or impliedly.

2. But as pointed out by the petitioner insurance company, the rejection referred to should
be construed as the rejection, in the first instance, for if what is being referred to is a
reiterated rejection conveyed in a resolution of a petition for reconsideration, such should
have been expressly stipulated.

While in the Eagle Star case (96 Phil. 701), this Court uses the phrase "final rejection",
the same cannot be taken to mean the rejection of a petition for reconsideration as
insisted by respondents. Such was clearly not the meaning contemplated by this Court.
The Insurance policy in said case provides that the insured should file his claim, first, with
the carrier and then with the insurer. The "final rejection" being referred to in said case is
the rejection by the insurance company.
95

Case Title
Malayan Insurance Co., Inc. vs. Cruz-Arnaldo, G.R. No.
67835, October 12, 1987

Topic Fire Insurance

Digested By: CASANOVA, Jenny Mae I.

Characters and Their Roles

Petitioner: Malayan Insurance Co.

Respondent: Gregoria Cruz Arnaldo (Insurance Commissioner) and Coronacion Pinca

FACTS:

The petitioner, Malayan Insurance Co., Inc, (MICO) issued to the private respondent,
Coronacion Pinca, a Fire Insurance Policy on her property for the amount of P100,000.00,
effective July 22, 1981, until July 22, 1982. Subsequently, MICO allegedly cancelled the
policy for non-payment of the premium and sent the corresponding notice to Pinca. On
December 24, 1981, payment of the premium for Pinca was received by Domingo Adora,
agent of MICO.

Then on January 15, 1982, Adora remitted this payment to MICO, together with other
payments. However, three days later, Pinca's property was completely burned. Then on
the following month, Pinca's payment was returned by MICO to Adora on the ground that
her policy had been cancelled earlier. But the latter refused to accept it. As a result, Pinca
made the requisite demands for payment, which MICO rejected. She then went to the
Insurance Commission who ruled in her favour. Hence, this petition.

ISSUE:

Whether or not there was a valid insurance contract at the time of the loss.

RULING:

Yes. There was a valid insurance contract at the time of the loss.
96

In the instant case, MICO contended that the policy had due been cancelled to the non-
payment of premium before the loss occurred. However, the Court held that their
contention is untenable because payment of the premium was eventually made in this
case. In fact, the premium invoice issued to Pinca at the time of the delivery of the policy
on June 7, 1981 was stamped "Payment Received" of the amount of P930.60 on "12-24-
81" by Domingo Adora who then remitted the same to MICO.

In relation to the cancellation of the policy, one of the conditions for a valid cancellation
involves prior notice of cancellation to the insured. In the case at bar, MICO claims it
cancelled the policy in question on October 15. 1981, for non-payment of premium. To
support this assertion, it presented one of its employees, who testified that the alleged
cancellation—"was sent to the insured by mail through our mailing section." However,
there is no proof that the notice was actually mailed to and received by Pinca. All MICO
offers to show that the cancellation was communicated to the insured and its employee's
testimony that the said cancellation was sent "by mail through our mailing section".

On the other hand, Pinca denied receiving the claimed cancellation. If Pinca had really
received the said notice, she would not have made payment on the original policy on
December 24, 1981. Instead, she would have asked for a new insurance, effective on
that date and until one year later, and so taken advantage of the extended period. As it
has not been shown that there was a valid cancellation of the policy, there was
consequently no need to renew it but to pay the premium thereon. Payment was thus
legally made on the original transaction and it could be, and was, validly received on
behalf of the insurer by its agent Adora. The latter, incidentally, had not been informed of
the cancellation either and saw no reason not to accept the said payment.

Furthermore, the certification issued by the Integrated National Police, Lao-ang, Samar,
as to the extent of Pinca's loss should be considered sufficient. In contrast, MICO
submitted no evidence to the contrary nor did it even question the extent of the loss in its
answer before the Insurance Commission. Although Pinca's payment was remitted to
MICO by its agent on January 15, 1982, MICO sought to return it to Adora only on
February 5, 1982, after it presumably had learned of the occurrence of the loss insured
against on January 18,1982. These circumstances make the motives of the petitioner
highly suspect and cast serious doubts upon its candor and bona fides. Therefore, the
Court held that there was a valid insurance contract at the time of the loss.
97

Case Title
American Home Assurance Co. vs. Tantoco Enterprise
Inc. 366 SCRA 740 (2001)

Topic Policy; Breach of Warranty

Digested By: CONDA, Ana Ultima G.

Parties:
● American Home: Insurer
● Tantuco Enterprises: Insured

Facts:

1) Tantuco Enterprises, Inc. is engaged in the coconut oil milling and refining
industry. It owns two oil mills at Lucena City. The second [or in this case referred
to as “new”] oil mill only started operating in 1988.
2) The two oil mills were separately covered by fire insurance policies issued by
American Home in which the mills were insured for 3 million and 6 million pesos
respectively for the period of March 1, 1991 to 1992.
3) On Sept. 30, 1991, a fire broke out and consumed the NEW OIL MILL.
Consequently, Tantuco Enterprises notified its insurer American Home of the
incident.
4) However, American Home rejected Tantuco’s claim for insurance proceeds
arguing mainly that there was no policy issued by it covering the burned oil mill -
- citing misdescription [it was not the one described by the specific boundaries in
the policy]. According to American Home, said policy in question was issued to
cover/protect the old oil mill.
a) Barred by parole evidence rule
b) Tantuco is barred by estoppel from claiming that the description of the insured
oil mill in the policy was wrong
c) There was breach of the Fire Extinguishing Appliances Warranty as the
respondent failed to install internal fire hydrants inside the burned building

Issue:
1. WON the new oil mill is covered by the subject policy [YES]
2. WON there was a breach of warranty [NO]
98

Discussion:

1. YES.

The SC held that “in construing the words used descriptive of a building insured,
the greatest liberality is shown by the courts in giving effect to the insurance.”

In view of the custom of insurance agents to examine buildings before writing


policies upon them, and since a mistake as to the identity and character of the building is
extremely unlikely, the courts are inclined to consider that the policy of insurance covers
any building which the parties manifestly intended to insure, however inaccurate the
description may be.

In this case, the first oil mill is already covered under a certain insurance policy
issued by American Home, contrary to the contention of the latter, as it would be absurd
to assume that Tantuco would protect its first oil mill for different amounts and leave
uncovered its second one.

In determining what the parties intended, the courts will read and construe the
policy as a whole and if possible, give effect to all the parts of the contract, keeping in
mind always, however, the prime rule that in the event of doubt, this doubt is to be
resolved against the insurer. In determining the intent of the parties to the contract, the
courts will consider the purpose and object of the contract.

2. NO.

As for the contention of American Home regarding the alleged breach of fire
extinguishing appliances warranty, the SC held that the warranty clause stated in the
policy did not require Tantuco to provide for all the fire extinguishing appliances
enumerated therein [Portable Extinguishers, Internal Hydrant, External Hydrant, Fire
Pump and 24 hour security services]. The warranty mandates Tantuco to maintain fire
fighting equipments such as, but not limited to, those identified in the list, in efficient
working condition within the premises of the insured property, which will serve as the oil
mill’s first line of defense in case any part of it bursts into flame.

In this case, Tantuco was able to comply as records showed that new oil mill
contained numerous portable fire extinguishes, two fire hoses, fire hydrant, among others
which were all in efficient working order when the fire occurred.
99

Case Title
Gen. Insurance v. Ng Hua, G.R. No. L-14373, January
30, 1960

Topic Co-insurance

Digested By: DOLOR, Rhodilee Jean A.

Facts
General Insurance and Surety issued Insurance Policy No. 471, which for a period of
one year, insured against fire the stocks in trade of the Central Pomade factory owned by
Ng Hua. The policy covers damages up to PhP 10,000. A day after the policy was issued
the building and the insured goods were burned.

General Insurance refused to pay citing Ng Hua’s breach against a co-insurance


stipulation included in the Policy No. 471, requiring the insured to notify the company of
other insurances that may affect the property insured and that it is stated in and endorsed
by the company. Otherwise the benefits will be forfeited:

“The insured shall give notice to the company of any insurance or insurances
already affected, or which may subsequently be effected, covering any of the
property hereby insured, and unless such notice be given and the particulars of
such insurance or insurances be stated in or endorsed on this Policy by or on
behalf of the Company before the occurrence of any loss or damage, all benefits
under the policy shall be forfeited.”

The face of the policy bore the annotation: "Co-Insurance Declared — NIL"

It was not denied that Ng Hua also took out fire insurance worth PhP 20,000 over the
same goods for the same period of time from General Indemnity Corporation.

Issue:
Whether or not General Insurance is liable to pay Ng Hua
100

Ruling

No.

The annotation [then], must be deemed to be a warranty that the property was not insured
by any other policy. Violation thereof entitles the insurer to rescind. The materiality of non-
disclosure of other insurance policies is not open to doubt.

Furthermore, even if the annotations were overlooked, the defendant insurer would still
be free from liability because there is no question that the policy issued by General
Indemnity had not been stated in nor endorsed on Policy No. 471 of defendant. And as
stipulated in the above-quoted provisions of such policy "all benefit under this policy shall
be forfeited.

All the above considerations lead to the conclusion that the defendant insurer successfully
established its defense of warranty breach or concealment of the other insurance and/or
violation of the provision of the policy above-mentioned.
101

Case Title Prudential Guarantee and Assurance, Inc. vs. Trans-


Asia Shipping Lines, Inc. G.R. No. 151890, June 20,
2006

Topic Breach of warranty

Digested By: DOMINGO, Angel Jean N.

Characters and Their Roles

Petitioner: Prudential Guarantee and Assurance, Inc.

Respondent: Trans-Asia Shipping Lines, Inc.

FACTS:

Trans Asia is the owner of the vessel M/V Asia Korea. Prudential Guarantee and
Assurance Inc. insured said vessel for loss/damage of the hull and machinery arising from
perils of fire and explosion beginning from the period of July 1, 1993 until July 1, 1994.

While the policy was in force, a fire broke out. Trans Asia filed its notice of claim for
damages sustained by the vessel. It also reserved its right to subsequently notify
Prudential as to the full amount of the claim upon final survey and determination by the
average adjuster Richard Hogg International of the damage sustained by the cause of
fire.

Trans Asia executed a document denominated “Loan and Trust Receipt” amounting to
Php 3,000,000. Prudential Guarantee and Assurance Inc. denied the former’s claim
and requested for the return of the said amount. The insurance company contends
that there was a breach in the policy conditions, specifically, “Warranted Vessel
Classed and Class Maintained”.

Trans-Asia filed a complaint for a sum of money against Prudential for the balance of the
indemnity due upon the insurance policy.

The RTC dismissed the complaint for failure to state a cause of action. It interpreted the
provision to mean that Trans-Asia is required to maintain the vessel at a certain class at
all times pertinent during the life of the policy. According to the court a quo, Trans-Asia
102

failed to prove compliance of the terms of the warranty, the violation thereof entitled
Prudential, the insured party, to rescind the contract. On appeal, said decision was
reversed by the CA.

ISSUE:

Whether or not Trans-Asia breached a material warranty that the vessel is classed and
class maintained.

RULING:

No, the Supreme Court held that Trans-Asia did not breach a material warranty that
the vessel is classed and class maintained. Prudential Guaranty was not successful
in discharging the burden of evidence that Trans-Asia breached the subject policy
condition on CLASSED AND CLASS MAINTAINED. Foremost, Prudential, through the
Senior Manager of its Marine and Aviation Division, Lucio Fernandez, made a categorical
admission that at the time of the procurement of the insurance, Trans-Asia’s vessel, “M/V
Asia Korea” was properly classed by Bureau Veritas, a classification society recognized
in the marine industry.

As it is undisputed that Trans-Asia was properly classed at the time the contract of
insurance was entered into, thus, it becomes incumbent upon Prudential to show
evidence that the status of Trans-Asia being properly CLASSED by Bureau Veritas had
shifted in violation of the warranty. Unfortunately, Prudential failed to support the
allegation.

Sec. 74 of the Insurance Code provides that, “the violation of a material warranty,
or other material provision of a policy on the part of either party thereto, entitles
the other to rescind.” It is generally accepted that “[a] warranty is a statement or promise
set forth in the policy, or by reference incorporated therein, the untruth or non-fulfillment
of which in any respect, and without reference to whether the insurer was in fact
prejudiced by such untruth or non-fulfillment, renders the policy voidable by the insurer.”
However, it is similarly indubitable that for the breach of a warranty to avoid a
policy, the same must be duly shown by the party alleging the same. The Supreme
Court did not sustain an allegation that is unfounded.

Consequently, PRUDENTIAL, not having shown that TRANS-ASIA breached the


warranty condition, CLASSED AND CLASS MAINTAINED, it remains that
TRANSASIA must be allowed to recover its rightful claims on the policy.
103

Case Title
Delsan Transport Lines, Inc. v. The Hon. Court of
Appeals, G.R. No. 127897, November 15, 2001

Topic Seaworthiness (Marine Insurance)

Digested By: GALEA, Samantha Racel Jeanne R.

Characters and Their Roles

Petitioner: Delsan Transport Lines, Inc.

Respondent: Caltex Philippines

FACTS:

Caltex Philippines (Caltex for brevity) entered into a contract of affreightment with Delsan
Transport Lines Inc. whereby the said common carrier agreed to transport Caltex's
industrial fuel oil from the Batangas-Bataan Refinery to different parts of the country.
Under the contract, Delsan petitioner took on board its vessel, MT Maysun, industrial fuel
oil of Caltex to be delivered to the Caltex Oil Terminal in Zamboanga City. The shipment
was insured with the private respondent, American Home Assurance Corporation.
Unfortunately, the vessel sank near Panay Gulf in the Visayas taking with it the entire
cargo of fuel oil.

Subsequently, private respondent paid Caltex the sum of Five Million Ninety-Six
Thousand Six Hundred Thirty-Five Pesos and Fifty-Seven Centavos (P5,096,635.57)
representing the insured value of the lost cargo. Exercising its right of subrogation under
Article 2207 of the New Civil Code, American Home demanded Delsan the same amount
it paid to Caltex. But due to its failure to collect, American Home filed a complaint against
Delsan for collection of a sum of money.

Delsan Transport Lines, Inc. invoked the provision of Section 113 of the Insurance Code
of the Philippines, which states that in every marine insurance upon a ship or freight, or
freightage, or upon any thing which is the subject of marine insurance there is an implied
warranty by the shipper that the ship is seaworthy. Delsan also alleged that under Section
116 of the Insurance Code of the Philippines, the implied warranty of seaworthiness of
104

the vessel, which the private respondent admitted as having been fulfilled by its payment
of the insurance proceeds to Caltex of its lost cargo, extends to the vessel's complement.
Delsan further averred that American Home failed to present in evidence during the trial
the subject marine cargo insurance policy it entered into with Caltex. By virtue of the
doctrine laid down in the case of Home Insurance Corporation vs. CA, the failure of the
private respondent to present the insurance policy in evidence is allegedly fatal to its
claim.

After the trial and upon analyzing the evidence adduced, the trial court rendered
a decision on dismissing the complaint against Delsan as it found that the vessel, MT
Maysun, was seaworthy to undertake the voyage as determined by the Philippine Coast
Guard per Survey Certificate Report No. M5-016-MH upon inspection during its annual
dry-docking and that the incident was caused by unexpected inclement weather
conditions or force majeure, thus exempting the common carrier, Delsan, from liability
for the loss of its cargo. The CA reversed this decision, giving credence to the weather
report issued by PAGASA that the waves where the vessel sank remained at 0.7 to 2
meters in height, in contrast to Delsan’s allegation that the waves were 6 meters (20 feet)
high.

ISSUE:

1. (Procedural) WON payment made by the private respondent to Caltex for the
insured value of the lost cargo amounted to an admission that the vessel was
seaworthy, thus precluding any action for recovery against the petitioner.
2. (Substantive) WON the non-presentation of the marine insurance policy bars the
complaint for recovery of sum of money for lack of cause of action.

RULING:
1. No. The payment made by the private respondent for the insured value of the lost
cargo operates as a waiver of its (private respondent) right to enforce the term of
the implied warranty against Caltex under the marine insurance policy. However,
the same cannot be validly interpreted as an automatic admission of the vessel's
seaworthiness by the private respondent as to foreclose recourse against the
petitioner for any liability under its contractual obligation as a common carrier. The
fact of payment grants the private respondent subrogatory right which enables it
to exercise legal remedies that would otherwise be available to Caltex as owner of
the lost cargo against the petitioner common carrier.

The right of subrogation under Art. 2207 of the Civil Code has its roots in equity. It
is designed to promote and to accomplish justice and is the mode which equity
adopts to compel the ultimate payment of a debt by one who in justice and good
105

conscience ought to pay.[9] It is not dependent upon, nor does it grow out of, any
privity of contract or upon written assignment of claim. It accrues simply upon
payment by the insurance company of the insurance claim.[10] Consequently, the
payment made by the private respondent (insurer) to Caltex (assured) operates as
an equitable assignment to the former of all the remedies which the latter may have
against the petitioner.

Neither may Delsan escape liability by presenting in evidence certificates[16] that


tend to show that at the time of dry-docking and inspection by the Philippine Coast
Guard, the vessel MT Maysun, was fit for voyage. These pieces of evidence do
not necessarily take into account the actual condition of the vessel at the time of
the commencement of the voyage. At the time of dry-docking and inspection, the
ship may have appeared fit. The certificates issued, however, do not negate the
presumption of unseaworthiness triggered by an unexplained sinking. Of
certificates issued in this regard, authorities are likewise clear as to their probative
value. Remember that ‘seaworthiness’ relates to a vessel's actual condition.
Neither the granting of classification or the issuance of certificates establishes
seaworthiness. Authorities are clear that diligence in securing certificates of
seaworthiness does not satisfy the vessel owner's obligation. Also securing the
approval of the shipper of the cargo, or his surveyor, of the condition of the vessel
or her stowage does not establish due diligence if the vessel was in fact
unseaworthy, for the cargo owner has no obligation in relation to seaworthiness.

2. No. The presentation in evidence of the marine insurance policy is not


indispensable in this case before the insurer may recover from the common carrier
the insured value of the lost cargo in the exercise of its subrogatory right. The
subrogation receipt, by itself, is sufficient to establish not only the relationship of
herein private respondent as insurer and Caltex, as the assured shipper of the lost
cargo of industrial fuel oil, but also the amount paid to settle the insurance claim.
The right of subrogation accrues simply upon payment by the insurance company
of the insurance claim.

The presentation of the insurance policy was necessary in the case of Home
Insurance Corporation v. CA (a case cited by petitioner) because the shipment
therein (hydraulic engines) passed through several stages with different parties
involved in each stage. In the present case, however, there is no doubt that the
cargo of industrial fuel oil belonging to Caltex was lost while on board petitioner's
vessel, MT Maysun, which sank while in transit
106

Case Title
American Home Assurance v. Chua, 309 SCRA 250
(1999)

Topic Valid payment of premium

Digested By: IRAN, Jayson O.

Characters and Their Roles

Petitioner: American Home Assurance

Respondent: Antonio Chua

FACTS:
Petitioner is a domestic corporation engaged in the insurance business. Sometime in
1990, respondent obtained from the petitioner a fire insurance covering the stock-in-trade
of his business, Moonlight Enterprises, located at Valencia, Bukidnon. The insurance was
due to expire on 25 March 1990.

On 5 April 1990 respondent issued a check to petitioner’s agent, James Uy, as payment
for the renewal of the policy. In turn, the latter delivered a renewal certificate to the
respondent. Subsequently, a new insurance policy was issued, whereby petitioner
undertook to indemnify respondent for any damage or loss arising from fire up to
P200,000 for the period 25 March 1990 to 25 March 1991.

On 6 April 1990 Moonlight Enterprises was completely razed by fire. Respondent filed an
insurance claim with petitioner and four other co-insurers. Petitioner refused to honor the
claim notwithstanding several demands by respondent.

ISSUES:

1. Whether or not there was a valid payment of premium, considering that


respondent’s check was cashed after the occurrence of the fire?
2. Whether or not the respondent violated the policy by his submission of fraudulent
documents and non-disclosure of the other existing insurance contracts?

RULING:
107

1. YES. The general rule in insurance laws is that unless the premium is paid the
insurance policy is not valid and binding. The only exceptions are life and industrial life
insurance. Whether payment was indeed made is a question of fact which is best
determined by the trial court. Well-settled is the rule that the factual findings and
conclusions of the trial court and the Court of Appeals are entitled to great weight and
respect, and will not be disturbed on appeal in the absence of any clear showing that the
trial court overlooked certain facts or circumstances which would substantially affect the
disposition of the case. The SC found no reason to depart from this ruling.

In the instant case, the best evidence of such authority is the fact that the petitioner
accepted the check and issued the official receipt for the payment. It is, as well, bound by
its agent’s acknowledgment of receipt of payment. Section 306 of the Insurance Code
provides that any insurance company which delivers a policy or contract of insurance to
an insurance agent or insurance broker shall be deemed to have authorized such agent
or broker to receive on its behalf payment of any premium which is due on such policy or
contract of insurance at the time of its issuance or delivery or which becomes due thereon.

2. NO. Ordinarily, where the insurance policy specifies as a condition the disclosure of
existing co-insurers, non-disclosure thereof is a violation that entitles the insurer to avoid
the policy. This condition is common in fire insurance policies and is known as the “other
insurance clause.” The purpose for the inclusion of this clause is to prevent an increase
in the moral hazard.. However, SC found an exception in the instant case. To constitute
a violation the other existing insurance contracts must be upon the same subject matter
and with the same interest and risk. Indeed, respondent acquired several co-insurers and
he failed to disclose this information to petitioner. Nonetheless, petitioner is estopped from
invoking this argument. The trial court cited the testimony of petitioner’s loss adjuster who
admitted previous knowledge of the co-insurers. Indubitably, it cannot be said that
petitioner was deceived by respondent by the latter’s non-disclosure of the other
insurance contracts when petitioner actually had prior knowledge thereof. Petitioner’s loss
adjuster had known all along of the other existing insurance contracts, yet, he did not use
that as basis for his recommendation of denial. The loss adjuster, being an employee of
petitioner, is deemed a representative of the latter whose awareness of the other
insurance contracts binds the petitioner. SC therefore held that there was no violation of
the “other insurance clause” by respondent.

Case Title
Makati Tuscany Condominium vs. Court of Appeals,
G.R. No. 95546, Nov. 6, 1992
108

Topic Sec. 77 of P.D. 612

Digested By MACULBE, Tresha Hyacinth A.

Characters and Their Roles

Petitioner: Makati Tuscany Condominium

Respondent: American International Underwriters

FACTS:

American International Underwriters issued a policy in favor of Makati Tuscany


Condominium Corporation with a total premium of P466,103.05. The company issued a
replacement policy. Premium was again paid. In 1984, the policy was again renewed and
private respondent issued to petitioner another policy. The petitioner paid 152,000 pesos
then refused to furnish the balance.

The company filed an action to recover the unpaid balance of P314,103.05.

The condominium administration explained that it discontinued the payment of premiums


because the policy did not contain a credit clause in its favor and that the acceptance of
premiums didn’t waive any of the company rights to deny liability on any claim under the
policy arising before such payments or after the expiration of the credit clause of the policy
and prior to premium payment, loss wasn’t covered.

Petitioner sought for a refund. The trial court dismissed the complaint and counterclaim
owing to the argument that payment of the premiums of the policies were made during
the lifetime or term of said policies, so risk attached under the policies.

The Court of Appeals ordered the petitioner to pay the balance of the premiums owing to
the reason that it was part of an indivisible obligation.
109

Petitioner now asserts that its payment by installment of the premiums for the insurance
policies invalidated them because of the provisions of Sec. 77 of the Insurance Code
disclaiming liability for loss for occurring before payment of premiums.

ISSUE:

Whether payment by installment of the premiums due on an insurance policy invalidates


the contract of insurance, in view of Sec. 77 of P.D. 612

RULING:

No. Under Sec. 77. An insurer is entitled to the payment of the premium as soon as the
thing is exposed to the peril insured against. Notwithstanding any agreement to the
contrary, no policy or contract of insurance issued by an insurance company is valid and
binding unless and until the premium thereof has been paid, except in the case of a life
or an industrial life policy whenever the grace period provision applies.

The Supreme Court affirmed the decision of the CA that while the import of Section 77 is
that prepayment of premiums is strictly required as a condition to the validity of the
contract, it cannot be ruled that the request to make installment payments duly approved
by the insurer, would prevent the entire contract of insurance from going into effect despite
payment and acceptance of the initial premium or first installment.

Section 78 of the Insurance Code in effect allows waiver by the insurer of the
condition of prepayment by making an acknowledgment in the insurance policy of
receipt of premium as conclusive evidence of payment so far as to make the policy
binding despite the fact that premium is actually unpaid. Section 77 merely precludes
the parties from stipulating that the policy is valid even if premiums are not paid, but does
not expressly prohibit an agreement granting credit extension. So is an understanding to
allow insured to pay premiums in installments not so proscribed.

The reliance by the petitioner on Arce vs. Capital Surety and Insurance Co. is unavailing
because the facts therein are substantially different from those in the case at bar. In Arce,
no payment was made by the insured at all despite the grace period given. Here,
petitioner paid the initial installment and thereafter made staggered payments resulting in
full payment of the 1982 and 1983 insurance policies. For the 1984 policy, petitioner paid
two (2) installments although it refused to pay the balance.
110

It appears from the peculiar circumstances that the parties actually intended to make
three (3) insurance contracts valid, effective and binding, petitioner may not be allowed
to renege on its obligation to pay the balance of the premium after the expiration of the
whole term. Moreover, as correctly observed by the appellate court, where the risk is
entire and the contract is indivisible, the insured is not entitled to a refund of the premiums
paid if the insurer was exposed to the risk insured for any period, however brief or
momentary. Hence, payment by installment of the premiums due on an insurance policy
does not invalidate the contract of insurance.
111

Case Title
UCPB General Insurance vs. Masagana Telamart, G.R.
No. 137172, April 4, 2001 (decision on the M/R)

Topic Exception to Section 77 of the Insurance Code

Digested By: MAGUINDANAO, Da'ud A.

Characters and Their Roles

Petitioner: UCPB General Insurance Co.

Respondent: Masagana Telemart, Inc.

FACTS:

UCPB issued 5 insurance policies covering Masagana’s various property against fire for
the period of May 22 1991 – 1992. On March of 1992, UCPB decided not to renew
Masagana’s insurance policy and gave a written notice to Masagana for the non-renewal
of the policy. A fire razed Masagana’s building on June 13 of 1992. A month later,
Masagana paid their premiums for May 22 1992-93 at UCPB’s main office. Such payment
was accepted. Masagana filed for a claim for indemnification of insured property but
UCPB returned the payment of Masagana and denied their claim for the reason that the
policies had expired and the fire occurred on June 13 which is a month before Masagana
tendered their payment of premiums.

However, it must be noted in this case UCPB, for a number of years, has granted
Masagana a 60-90 day credit term for the renewal of the policies.

ISSUE:

Whether or not Masagana can still claim from the policies even after its expiration

RULING:

Yes. Masagana can still claim the proceeds of the policy. UCPB is estopped from raising
the defense of expiration of the policy when it had been of granting Masagana a credit
term of 60-90 days. The instant case has to rise or fall on the core issue of whether
112

Section 77 of the Insurance Code must be strictly applied to UCPB’s advantage despite
its practice of granting a 60- to 90-day credit term for the payment of premiums.

There have been 4 exceptions to the application of Section 77, namely:

1. From Section 77 itself, in case of a life or industrial life policy whenever the grace
period provision applies.

2. Provided by Section 78

3. In the case of Tuscany where parties may agree that the payment of premiums
may be made in installments and partial payment has been made at the time of the loss.

4. Also in the case of Tuscany where the insurer may grant a credit extension for the
payment of the premium.

As held by the court in this case, here is nothing in Section 77 which prohibits the parties
in an insurance contract to provide a credit term within which to pay the premiums. That
agreement is not against the law, morals, good customs, public order or public policy. The
agreement binds the parties.

It would be unjust and inequitable if recovery on the policy would not be permitted against
UCPB, which had consistently granted a 60- to 90-day credit term for the payment of
premiums despite its full awareness of Section 77. Estoppel bars it from taking refuge
under said Section, since Masagana relied in good faith on such practice. Estoppel then
is the fifth exception to Section 77.
113

Case Title
Phil. Phoenix vs Woodworks, 20 SCRA 1270 (1967)

Topic Perfected contract of insurance

Digested By: MAYANGAO, Ian Jermie O.

Characters and Their Roles

Plaintiff: Philippine Phoenix Surety & Insurance, Inc

Defendant: Woodworks, Inc.,

FACTS:

Woodworks obtained from Phoenix and was issued a fire insurance policy for the amount
of P300,000.00. The premiums amounted to P 6,051.95 but Woodworks only paid
P3,000.

Despite several demands made by Phoenix, Woodworks failed to pay.

Thus, Phoenix filed a case to recover from Woodworks the unpaid balance of the
premiums(P3,522.09).

CFI Manila decided in favor of Phoenix. Woodworks appealed, claiming that its non-
payment of the premium cancelled the policy and thus is not liable to pay for the premiums
due.

ISSUE:

Whether or not the non-payment of the premium by Woodworks cancelled the contract of
insurance.

RULING:

No, non-payment of premium due did not cancel the contract of insurance.

As the contract of insurance had already become perfected, the parties could demand
from each other the performance of whatever obligations they had assumed.
114

Where, between the insurer and the insured, there was not only a perfected contract of
insurance but a partially performed one as far ,as the payment of the agreed premium
was concerned, the obligation of the insurer to pay the insured the amount for which the
policy was issued in case the conditions therefor had ,been complied with, arose and
became binding upon it, while the obligation of the insured to pay the remainder of the
total amount of premium due became demandable. ,

Such theory would place exclusively in the hands of one of the contracting parties the
right to decide whether the contract should stand or not. Rather the correct view would
seem to be this: as the contract had become perfected, the parties. could demand from
each other the performance of whatever obligations they had assumed. In the case of the
insurer, it is obvious that it had the right to demand from the insured the completion of the
payment of the premium due or sue for the rescission of the contract. As it chose to
demand specific performance of the insured's obligation to pay the balance of the
premium, the latter's duty to pay is indubitable.

In the case of the insurer (Phoenix), it is obvious that it had the right to demand from the
insured (Woodworks) the completion of the payment of the premium due
OR sue for therescission of the contract.

Since Phoenix chose to demand specific performance of Woodwork’s obligation to pay


the balance of the premium, the latter's duty to pay is indeed indubitable. If the non-
payment of the premium would cancel the policy, it would place exclusively in the hands
of one of the contracting parties the right to decide whether the contract should stand or
not.
115

Case Title
Phil. Phoenix vs. Woodworks, 92 SCRA 419 (1979)

Topic Perfected Contract of Insurance; Premiums

Digested By: MENDOZA, Juan Antonio A.

Characters and Their Roles

Plaintiff-Appellee: Philippine Phoenix Surety & Insurance Company

Defendant-Appellant: Woodworks, Inc.

FACTS:

On July 21, 1960, Phoenix issued a Fire Insurance Policy to Woodworks for 500 thousand
pesos. Phoenix insured Woodwork’s building, machinery, and equipment for a term of
one (1) year from July 21, 1960 to July 21, 1961 against loss by fire.

The premium and other charges including the margin fee surcharge of P590.76 and the
documentary stamps worth P156.80 affixed on the policy, amounted to P10,593.36.

Woodworks did not pay the premiums stipulated in the policy when it was issued nor at
any time thereafter. On April 19, 1961, before the expiration of the one-year term, Phoenix
notified Woodworks through its Indorsement of the cancellation of the policy allegedly
upon the request of Woodworks. Woodworks denied having made such a request.

In said Indorsement, Phoenix credited Woodworks with the amount of P3,11.25 for the
unexpired period of 94 days, and claimed the balance of P7,483.11 representing “earned
premium from July 21, 1960 to 18th April 1961, or say 271 days.” On July 6, 1961,
Phoenix demanded in writing for the payment of said amount. Woodworks, through
counsel, disclaimed any liability in its Reply-Letter dated August 15, 1961, contending
that it need not pay the premium because the insurer did not stand liable for any indemnity
during the period the premiums were not paid.

On January 30, 1962, Phoenix commenced an action in the CFI of Manila to recover the
amount of P7,483.11 as earned premium. Woodworks countered that its failure to pay the
116

premium after the issuance of the policy put an end to the insurance contract and
rendered the policy unenforceable.

The CFI ruled in favor of Phoenix. It ordered:

1. That Woodworks should pay Phoenix for the earned premiums;


2. That Woodworks should pay the attorney’s fees and costs of the suit.

Upon appeal to the CA, the CA certified the case to the SC on a question of law.

ISSUE:

1. Whether or not the policy was binding even without the payment of premiums.
2. Whether or not the premium became a demandable obligation even after the
period in which the policy has expired.

RULING:

Insurance is a contract whereby one undertakes for a consideration to indemnify another


against loss, damage, or liability arising from an unknown or contingent event. The
consideration is the premium. The premium must be paid at the time and in the way and
manner specified in the policy. If not so paid, the policy will lapse and be forfeited by its
own terms.

The policy provided for the pre-payment of premium. Accordingly, when the policy is
tendered, the insured must pay the premium unless credit is given or there is a waiver or
some agreement obviating the necessity for pre-payment. To constitute an extension of
credit, there must be a clear and express agreement therefor.

From the policy provisions, there was a lack of showing that a credit extension was
accorded to Woodworks. And even if it were to be presumed that Phoenix had extended
credit from the circumstances of the unconditional delivery of the policy without pre-
payment of the premium, it is obvious that Woodworks had not accepted Phoenix’s offer
to extend the credit, which is essential for the validity of such agreement.

This case differs from that involving the same parties, where recovery of the balance of
the unpaid premium was allowed because in that case “there was not only a perfected,
contract of insurance but a partially performed one as far as the payment of the agreed,
117

premium was concerned. This is not the situation obtaining here where no partial payment
of premiums has been made whatsoever.

Since the premium had not been paid, the policy must be deemed to have lapsed.

“The non-payment of premiums does not merely suspend but puts an end to an
insurance contract, since the time of the payment is peculiarly of the essence of
the contract.”

“x x x the rule is that under policy provisions that upon the failure to make a
payment of a premium or assessment at the time provided for, the policy shall
become void or forfeited, or the obligation of the insurer shall cease, or words to
like effect, because the contract so prescribes and because such a stipulation is
material and essential part of the contract. This is true, for instance, in the case of
life, health and accident, fire and hail insurance policies.”

In fact, if the peril insured against had occurred, plaintiff, as insurer, would have had a
valid defense against recovery under the Policy it had issued. Explicit in the Policy itself
is plaintiff’s agreement to indemnify defendant for loss by fire only “after payment of
premium,” supra. Compliance by the insured with the terms of the contract is a condition
precedent to the right of recovery.

“The burden is on an insured to keep a policy in force by the payment of premiums,


rather than on the insurer to exert every effort to prevent the insured from allowing
a policy to elapse through a failure to make premium payments. The continuance
of the insurer’s obligation is conditional upon the payment of premiums, so that no
recovery can be had upon a lapsed policy, the contractual relation between the
parties having ceased.”

Moreover, “an insurer cannot treat a contract as valid for the purpose of collecting
premiums and invalid for the purpose of indemnity.”

The foregoing findings are buttressed by section 77 of the Insurance Code (Presidential
Decree No. 612, promulgated on December 18, 1974), which now provides that no
contract of insurance issued by an insurance company is valid and binding unless and
until the premium thereof has been paid, notwithstanding any agreement to the contrary.

WHEREFORE, the judgment appealed from is reversed, and plaintiff’s complaint hereby
dismissed.
118

Case Title Grepalife vs. CA, 184 SCRA 501 (1990)

Topic Premium

Digested By: MIRAFUENTES, Charlotte P.

Characters and Their Roles

Petitioner: Great Pacific Life Insurance Corporation (Insurance Company)

Respondent: Court of Appeals and Teodoro Cortez (Private Respondent)

FACTS:

Teodoro Cortez, upon the solicitation of Margarita Siega, Grepalife's underwriter, applied
for a 20-year endowment policy for P30,000. It was accepted and approved by the
Grepalife and subsequently an endowment policy was issued in his name. It was released
for delivery on January 24, 1973, and was actually delivered to him by the underwriter,
Mrs. Siega on January 25, 1973.

The effectivity date on the policy was December 25, 1972 and the annual premium was
1,416.60. Siega assured Cortez that the first premium may be paid within the grace period
of thirty (30) days from date of delivery of the policy.

Cortez then paid in three (3) installments which were evidenced by temporary receipts
issued by Siega then official receipts by the insurer confirming such payments were
subsequently issued on a later date.

On June 1, 1973, four months after the payments in installment, Grepalife advised Cortez
that the insurance policy was not in force, hence to make it effective Cortez was asked to
remit the balance of P1,015.60 to complete his initial annual premium due December 15,
1972, and to see a physician for another full medical examination at his own expense.

With this, Cortez informed the insurance company that he is cancelling the policy and he
is demanding the return of his premium.

ISSUE: Whether or not Cortez is entitled to the return of his premium

RULING: Yes, Teodoro Cortez is entitled to the return of his premium.

Sections 79, 81 and 82 of the Insurance Code of 1978 provide when the insured is entitled
to the return of premium paid.
119

SECTION 79. A person insured is entitled to a return of premium, as follows:

(a) To the whole premium, if no part of his interest in the thing insured be exposed
to any of the perils insured against.

(b) Where the insure is made for a definite period of time and the insured
surrenders his policy, to such portion of the premium as corresponds with the
unexpired time, at a pro rata rate, unless a short period rate has been agreed upon
and appears on the face of the policy, after deducting from the whole premium any
claim for loss or damage under the policy which has previously accrued: Provided,
That no holder of a life insurance policy may avail himself of the privileges of this
paragraph without sufficient causes as otherwise provided by law.

SECTION 81. A person insured is entitled to a return of the premium when the contract
is voidable on account of the fraud or misrepresentation of the insurer or of his agent or
on account of facts the existence of which the insured was ignorant without his fault; or
when, by any default of the insured other than actual fraud, the insurer never incurred any
liability under the policy.

SECTION 82. In case of an over-insurance by several insurers, the insured is entitled to


a ratable return of the premium, proportioned to the amount by which the aggregate sum
insured in all the policies exceeds the insurable value of the thing at risk.

In this case, the policy was issued on December 25, 1972 and was delivered on January
25, 1973. Cortez was given by Grepalife through its underwriter Mrs. Margarita Siega a
grace period of 30 days from said date within which the premium was to be paid. Record
shows that the premium was paid fully on February 21, 1973 or within the grace period.
This being so, the policy was already enforceable. The company had sufficient time to
examine the result of their medical examination on Cortez. They would not have delivered
the policy on January 24, 1973 if Cortez was unacceptable. Moreover, if premiums were
to be paid within 90 days then the reckoning period should be the date the policy was
delivered and not the date the Cortez was physically examined. The 90-day period from
the date of physical examination as provided for in the receipts of payment is of' no
moment, since said receipts are an integral part of the insurance policy (contract). The
official receipts issued by the company's agent can only mean that the company ratified
the act of Siega in giving Siega a grace period of 30 days from January 25, 1973 within
which to pay the annual premium.

Since Cortez’s policy was in fact inoperative or ineffectual from the beginning, Grepalife
was never at risk, hence, it is not entitled to keep the premium. It should be returned to
Cortez.
120

Case Title
Heirs of I. Coscolluela vs. Rico General Insurance
Corp. 179 SCRA 511 (1989)

Topic Insurance Policy Excepted Risk; Burden of Proof

Digested By: OLAER, Jezrel L.

Characters and Their Roles

Petitioner: Heirs of Ildefonso Coscolluela, Sr., Inc.,

Respondent: Rico General Insurance Corp.

FACTS:

Heirs of Ildefonso Coscoluella, Inc. is a domestic corporation and the registered owner of
an Isuzu KBD Pick-up truck. The vehicle was insured with the private respondent Rico
General Insurance Corporation for a consideration of P100,000.00 excluding third party
liability. Within the period covered by the insurance, the insured vehicle was severely
damaged and rendered unserviceable when fired upon by a group of unidentified armed
persons at Hacienda Puyas, Barangay Blumentritt, Murcia, Negros Occidental. Petitioner
filed its claim of P80,000.00 for the repair of the vehicle but private respondent refused to
grant it.

The insurer filed a motion to dismiss alleging that the complaint lacks a cause of action
because the firing by armed men is a risk excepted under the provisions in the insurance
policy. It alleged that the firing was "an indirect consequence of rebellion, insurrection or
civil commotion." which is a risk excepted in the policy. The petitioner opposed the motion,
saying that the quoted provision does not apply in the absence of an official governmental
proclamation of any of the above-enumerated conditions.

The trial court ordered the dismissal of the complaint for lack of cause of action stating
that the damage arose from a civil commotion or was a direct result thereof. The trial court
further noted that it can take effective cognizance of the general civil disturbance in the
country akin to civil war without any executive proclamation of the existence of such
unsettling condition.
121

Petitioner further maintained that the order of dismissal was erroneous in that: it
overlooked the principle that a motion to dismiss a complaint on the ground of failure to
state a cause of action hypothetically admits the allegations in the complaint; no trial was
held for the reception of proof that the firing incident was a direct or indirect result
of a civil commotion, mutiny, insurrection or rebellion; private respondent had the
burden of proof to show that the cause was really an excepted risk; and in any
case, the nature of the incident as a "civil disturbance" must first be officially
proclaimed by the executive branch of the government.

ISSUE:

Whether or not the claim by petitioner lacks cause of action

RULING:

NO. The facts as alleged clearly define the existence of a right of the petitioner to a just
claim against the insurer for the payment of the indemnity for a loss due to an event
against which the petitioner's vehicle was insured. The insurance contract mentioned
therein manifests a right to pursue a claim and a duty on the part of the insurer or private
respondent to compensate the insured in case of a risk insured against. The refusal of
the insurer to satisfy the claim and the consequent loss to the petitioner in incurring the
cost of acquiring legal assistance on the matter constitutes a violation or an injury brought
to the petitioner.

The private respondent's invocation of the exceptions clause in the insurance policy as
the basis for its non-liability and the consequent dismissal of the complaint is without
merit. The Court reiterated the established rule that when the terms of an insurance
contract contain limitations on liability, the court "should construe them in such a
way as to preclude the insurer from non-compliance with his obligations." A policy
of insurance with a narration of exceptions tending to work a forfeiture of the policy
shall be interpreted liberally in favor of the insured and strictly against the
insurance company or the party for whose benefit they are inserted.

The facts alleged in the complaint do not give a complete scenario of the real nature of
the firing incident. Hence, it was incumbent upon the trial judge to have made a deeper
scrutiny into the circumstances of the case by receiving evidence instead of summarily
disposing of the case. This case does not present a pure question of law but demands a
factual determination of whether the incident was a result of events falling under the
exceptions to the liability of private respondent contained in the policy of insurance.

The burden of proof to show that the insured is not liable because of an excepted
risk is on the insurer. Where the insurer denies liability for a loss alleged to be due
122

to a risk not insured against, but fails to establish the truth of such fact by concrete
proofs, the Court rules that the insurer is liable under the terms and conditions of
the policy by which it has bound itself. In this case, the dismissal order without hearing
and reception of evidence to prove that the firing incident was indeed a result of a civil
commotion, rebellion or insurrection constitutes reversible error on the part of the trial
court.

The Court stresses that it would be a grave and dangerous procedure for the courts to
permit insurance companies to escape liability through a motion to dismiss without the
benefit of hearing and evidence every time someone is killed, or as in this case,. property
is damaged in an ambush. The question on the nature of the firing incident for the purpose
of determining whether or not the insurer is liable must first be threshed out and resolved
in a full-blown trial.
123

Case Title
Country Bankers Insurance Corp. vs. Lianga Bay and
Multi-Purpose Cooperative, Inc. 374 SCRA 653

Topic Fire Insurance Policy

Digested By: Group 1

FACTS:

Respondent, Llanga Bay and Community Multi-Purpose Cooperative, and petitioner


entered into a contract of fire insurance to protect respondent’s stocks-in-trade against
fire loss, damage or liability for P200,000.00. The respondent’s building was gutted in a
fire resulting in the total loss of the respondent’s stocks-in-trade, pieces of furniture and
fixtures, equipment and records.

The petitioner denied the insurance claim on the ground that the building was set on fire
by two (2) NPA rebels who wanted to obtain canned goods, rice and medicines as
provisions for their comrades in the forest, and that such loss was an excepted risk under
paragraph No. 6 of the policy conditions of Fire Insurance Policy.

Paragraph 6 provides:

This insurance does not cover any loss or damage occasioned by or through or in
consequence, directly or indirectly, of any of the following occurrences, namely:

xxx - xxx - xxx

(d) Mutiny, riot, military or popular uprising, insurrection, rebellion, revolution, military or
usurped power.

Any loss or damage happening during the existence of abnormal conditions (whether
physical or otherwise) which are occasioned by or through or in consequence, directly or
indirectly, of any of said occurrences shall be deemed to be loss or damage which is not
covered by this insurance, except to the extent that the Insured shall prove that such loss
or damage happened independently of the existence of such abnormal conditions.
124

RTC ruled in favor of the respondent. CA affirmed RTC's decision. Hence, this petition.

ISSUE:

Whether or not the stocks-in-trade burned by the NPA rebels are considered excepted
risks under the fire insurance policy

RULING:

No. A party is bound by his own affirmative allegations. This is a well known postulate
echoed in Section 1 of Rule 131 of the Revised Rules of Court. Each party must prove
his own affirmative allegations by the amount of evidence required by law which in civil
cases, as in this case, is preponderance of evidence, to obtain a favorable judgment. In
the instant case, the petitioner does not dispute that the respondent’s stocks-in-trade
were insured against fire loss, damage or liability under Fire Insurance Policy No. F- 1397
and that the respondent lost its stock in- trade in a fire that occurred on July 1, 1989,
within the duration of said fire insurance.

The petitioner, however, posits the view that the cause of the loss was an excepted risk
under the terms of the fire insurance policy. Where a risk is excepted by the terms of a
policy which insures against other perils or hazards, loss from such a risk constitutes a
defense which the insurer may urge, since it has not assumed that risk, and from this it
follows that an insurer seeking to defeat a claim because of an exception or limitation in
the policy has the burden of proving that the loss comes within the purview of the
exception or limitation set up.

If a proof is made of a loss apparently within a contract of insurance, the burden is upon
the insurer to prove that the loss arose from a cause of loss which is excepted or for which
it is not liable, or from a cause which limits its liability. Stated elsewise, since the petitioner
in this case is defending on the ground of noncoverage and relying upon an exemption
or exception clause in the fire insurance policy, it has the burden of proving the facts upon
which such excepted risk is based, by a preponderance of evidence. But petitioner failed
to do so.

The Sworn Statements of Jose Lomocso and Ernesto Urbiztondo are inadmissible in
evidence, for being hearsay, inasmuch as they did not take the witness stand and could
not therefore be cross examined. The petitioner’s evidence to prove its defense is sadly
wanting and thus, gives rise to its liability to the respondent under Fire Insurance Policy
No. F-1397.
125

Case Title
DBP Pool of Accredited Insurance Companies vs.
Radio Mindanao Network, Inc. 480 SCRA 314 (2006)

Topic Excepted risk

Digested By: Olayan, Imma Coney P.

Characters and Their Roles

Petitioner: DBP Pool of Accredited Insurance Companies

Respondent: Radio Mindanao Network, Inc.

FACTS:

Radio Mindanao Network, Inc. owns several broadcasting stations all over the country.
Provident Insurance covered RMN's transmitter equipment and generating set under a
Fire Policy while DBP Pool of Accredited Insurance Companies covered RMN's
transmitter, furniture, fixture and other transmitter facilities under another fire insurance
policy.

On July 27, 1988 RMN's radio station in Bacolod City was razed by fire but the claims
were denied on the ground that the cause of loss was an excepted risk excluded under
condition no. 6(c) and (d):

6. This insurance does not cover any loss or damage occasioned by or through or
in consequence, directly or indirectly, of any of the following consequences,
namely:

(c) War, invasion, act of foreign enemy, hostilities, or warlike operations (whether
war be declared or not), civil war.

(d) Mutiny, riot, military or popular rising, insurrection, rebellion, revolution, military
or usurped power.

The insurance companies maintained that evidence showed that the fire was caused by
members of the Communist Party of the Philippines/New People's Army (CPP/NPA).
However, the trial court and the CA found that the evidence presented failed to support
126

the allegation that the loss was caused by an excepted risk, i.e. members of the CPP/NPA
caused the fire.

ISSUE:

Whether or not the insurance companies are liable for the fire

RULING:

YES, the insurance companies are liable. The court found that none of the evidence
presented (police blotter, fire investigation report) categorically stated that the twenty (20)
armed men which burned DYHB were members of the CPP/NPA. The said documents
simply stated that the said armed men were ‘ believed’ to be or ‘ suspected’ of being
members of the said group.

An insurance contract, being a contract of adhesion, should be so interpreted as to carry


out the purpose for which the parties entered into the contract which is to insure against
risks of loss or damage to the goods. Limitations of liability should be regarded with
extreme jealousy and must be construed in such a way as to preclude the insurer from
noncompliance with its obligations.

The burden of proof refers to the duty of the insured to show that the loss or damage is
covered by the policy.; The burden of proof still rests upon the insurer to prove that the
damage or loss was caused by an excepted risk in order to escape any liability under the
contract.

It is sufficient for RMN to prove the fact of damage or loss. Once respondent makes out
a prima facie case in its favor, the duty or burden of evidence shifts to petitioner to
controvert respondent’s prima facie case.

In this case, since the insurer alleged an excepted risk, then the burden of evidence
shifted to the insurer to prove such exception. It is only when the insurer has sufficiently
proven that the damage or loss was caused by an excepted risk does the burden of
evidence shift back to respondent who is then under a duty of producing evidence to show
why such excepted risk does not release petitioner from any liability. Unfortunately for the
insurer, it failed to discharge its primordial burden of proving that the damage or loss was
caused by an excepted risk.

WHEREFORE, the insurance companies were ordered to pay RMN the value of the
destroyed property covered under their respective fire insurance policies.
127

Case Title
FGU Insurance Corporation vs. Court of Appeals, 454
SCRA 337 (2005)

Topic Section 89 - Negligence of the Insured

Digested By: San Tiago

FACTS:

Anco Enterprises Company (ANCO), a partnership between Ang Gui and Co To, was
engaged in the shipping business. It owned the M/T ANCO tugboat and the D/B Lucio
barge that were operated as common carriers. Since the D/B Lucio had no engine of its
own, it could not maneuver by itself and had to be towed by a tugboat for it to move from
one place to another.

On 23 September 1979, San Miguel Corporation (SMC) shipped from Mandaue City,
Cebu, on board the D/B Lucio, for towage by M/T ANCO, when the barge and tugboat
arrived at San Jose, Antique, in the afternoon of 30 September 1979, the clouds over the
area were dark and the waves were already big. The arrastre workers unloading the
cargoes of SMC on board the D/B Lucio began to complain about their difficulty in
unloading the cargoes. SMC’s District Sales Supervisor, Fernando Macabuag, requested
ANCO’s representative to transfer the barge to a safer place because the vessel might
not be able to withstand the big waves.

ANCO’s representative did not heed the request because he was confident that the barge
could withstand the waves. At around midnight, the barge run aground and was broken
and the cargoes of beer in the barge were swept away.

As a consequence of the incident, SMC filed a complaint for Breach of Contract of


Carriage and Damages against ANCO. ANCO admitted that the cases of beer Pale Pilsen
and Cerveza Negra mentioned in the complaint were indeed loaded on the vessel
belonging to ANCO. It claimed however that it had an agreement with SMC that ANCO
would not be liable for any losses or damages resulting to the cargoes by reason of
fortuitous event. Third-party defendant FGU admitted the existence of the Insurance
Policy under Marine Cover Note No. 29591 but maintained that the alleged loss of the
cargoes covered by the said insurance policy cannot be attributed directly or indirectly to
any of the risks insured against in the said insurance policy.
128

ISSUE:

1. Whether ANCO’s representatives were able to exercise the extraordinary degree


of diligence required by the law to exculpate them from liability for the loss of the
cargoes.
2. Whether or not FGU be held liable under the insurance policy to reimburse ANCO
for the loss of the cargoes despite the findings of the CA that such loss was
occasioned by the blatant negligence of the latter’s employees?

RULING:

1. No. In the case at bar, the calamity that caused the loss of the cargoes was not
unforeseen nor was it unavoidable. In fact, the other vessels in the port of San
Jose, Antique, managed to transfer to another place, a circumstance which
prompted SMC’s District Sales Supervisor to request that the D/B Lucio be likewise
transferred, but to no avail. The D/B Lucio had no engine and could not maneuver
by itself. Even if ANCO’s representatives wanted to transfer it, they no longer had
any means to do so as the tugboat M/T ANCO had already departed, leaving the
barge to its own devices. The captain of the tugboat should have had the foresight
not to leave the barge alone considering the pending storm.

2. No. While, it is a basic rule in insurance that the carelessness and negligence of
the insured or his agents constitute no defense on the part of the insurer (as
ordinary negligence of the insured and his agents has long been held as a part of
the risk which the insurer takes upon himself). This rule presupposes that the loss
has occurred due to causes which could not have been prevented by the insured,
despite the exercise of due diligence. When evidence shows that the insured’s
negligence or recklessness is so gross as to be sufficient to constitute a willful act,
the insurer must be exonerated.

In this case, the Court agreed with the findings of the lower courts that under the
circumstances, the employees and crewmembers of both the D/B Lucio and the
M/T ANCO were blatantly negligent.
129

Case Title
Bonifacio Bros. vs. Mora, 20 SCRA 261 (1969)

Topic

Digested By: RIZON, Justine M.

Characters and Their Roles

Petitioner: Bonifacio Bros. Inc. – labor and materials (some materials supplied by Ayala
Auto Parts Co.)

Respondent, Mortgagor: Enrique Mora

Mortgagee – H.S. Reyes, Inc.

Insurer - State Bonding & Insurance Co

Investigation and appraisal – H.H. Bayne Adjustment Co.

FACTS:

Enrique Mora, owner of Oldsmobile sedan model 1956 mortgaged the same to the H.S.
Reyes, Inc., with the condition that the former would insure the automobile with the latter
as beneficiary. The automobile was thereafter insured on June 23, 1959 with the State
Bonding & Insurance Co., Inc., and a motor car insurance policy was issued to Enrique
Mora.

During the effectivity of the insurance contract, the car met with an accident. The
insurance company then assigned the accident to the H.H, Bayne Adjustment Co. for
investigation and appraisal of the damage.

Enrique Mora, without the knowledge and consent of the H.S. Reyes, Inc., authorized the
Bonifacio Bros. Inc. to furnish the labor and materials, some of which were supplied by
the Ayala Auto Parts Co.
130

For the cost of labor and materials, Enrique Mora was billed at P2,102.73 through the
H.H. Bayne Adjustment Co. The insurance company after claiming a franchise in the
amount of P100, drew a check in the amount of P2,002.73, as proceeds of the insurance
policy, payable to the order of Enrique Mora or H.S. Reyes,. Inc., and entrusted the check
to the H.H. Bayne Adjustment Co. for disposition and delivery to the proper party. In the
meantime, the car was delivered to Enrique Mora without the consent of the H.S. Reyes,
Inc., and without payment to the Bonifacio Bros. Inc. and the Ayala Auto Parts Co. of the
cost of repairs and materials.

Arguing that the insurance proceeds should be paid directly to them, the Bonifacio Bros.
Inc. and the Ayala Auto Parts Co. filed against Enrique Mora and the State Bonding &
Insurance Co., Inc. for the collection of the sum of P2,002.73. It based its claim in
paragraph 4 of the insurance which provides that “the insured may authorize the repair of
the Motor Vehicle necessitated by damage for which the company may be liable under
the policy provided that (a) the estimated cost of such repair does not exceed the
Authorized Repair Limit, and (b) a detailed estimate of the cost is forwarded to the
company without delay." They stressed that the H.H. Bayne Adjustment Company's
recommendation of payment of the appellants' bill for materials and repairs for which the
latter drew a check for P2,002.73 indicates that Mora and the H.H. Bayne Adjustment Co.
acted for and in representation of the insurance company.

ISSUE:

Whether or not Bonifacio Bros. Inc. and the Ayala Auto Parts have right of action to claim
the insurance proceeds

RULING:

The appellants' alleged cause of action rests exclusively upon the terms of the insurance
contract. However, they are not mentioned in the contract as parties thereto nor is
there any clause or provision thereof from which we can infer that there is an
obligation on the part of the insurance company to pay the cost of repairs directly
to them.

Contracts take effect only between the parties thereto, except in some specific instances
provided by law where the contract contains some stipulation in favor of a third person.
Such stipulation is known as stipulation pour autrui or a provision in favor of a third person
not a party to the contract. Under this doctrine, a third person is allowed to avail himself
131

of a benefit granted to him by the terms of the contract, provided that the contracting
parties have clearly and deliberately conferred a favor upon such person.

In the instant case the insurance contract does not contain any words or clauses
to disclose an intent to give any benefit to any repairmen or materialmen in case of
repair of the car in question. The parties to the insurance contract omitted such
stipulation, which is a circumstance that supports the said conclusion. On the other hand,
the "loss payable" clause of the insurance policy stipulates that "Loss, if any, is
payable to H.S. Reyes, Inc." indicating that it was only the H.S. Reyes, Inc. which
they intended to benefit.

"a policy of insurance is a distinct and independent contract between the insured
and insurer, and third persons have no right either in a court of equity, or in a court
of law, to the proceeds of it, unless there be some contract of trust, expressed or
implied between the insured and third person."

In this case, no contract of trust, expressed or implied exists. The appellants' claim, if at
all, is merely equitable in nature and must be made effective through Enrique Mora who
entered into a contract with the Bonifacio Bros. Inc. This is covered by the express
provisions of section 50 of the Insurance Act which read:

The insurance shall be applied exclusively to the proper interests of the person in
whose name it is made unless otherwise specified in the policy.

In this case, the policy states that "Loss, if any, is payable to H.S. Reyes, Inc.," which
unmistakably shows the intention of the parties.
132

Case Title
Allied Banking Corp. vs. Lim Sio Wan, 549 SCRA 504
(2008)

Topic Section 84; Proximate Cause

Digested By: Sumamban, Geraldine B.

Characters and Their Roles

Petitioner: Allied Banking Corporation

Respondent: Lim Sio Wan

DOCTRINE:
The doctrine of proximate cause has been defined as follows: "Was there an unbroken
connection between the wrongful act and the injury, a continuous operation? Did the facts
constitute a continuous succession of events, so linked together as to make a natural
whole, or was there some new and independent cause intervening between the wrong
and the injury?"

The question that needs to be asked is: If the event did not happen, could the injury have
resulted? If the answer is NO, then the event is the proximate cause.

FACTS:
On November 14, 1983, Lim Sio Wan deposited with petitioner Allied Banking Corporation
a money market placement of PhP 1,152,597.35 for a term of 31 days to mature on
December 15, 1983.

On December 5, 1983, a person claiming to be Lim Sio Wan called up Cristina So, an
officer of Allied, and instructed the latter to pre-terminate Lim Sio Wan’s money market
placement, to issue a manager’s check representing the proceeds of the placement, and
to give the check to one Deborah Dee Santos who would pick up the check.

Santos arrived at the bank, which issued a Manager's Check for PhP 1,158,648.49,
representing the proceeds of Lim Sio Wan’s money market placement. Said manager’s
check was deposited in the account of Filipinas Cement Corporation at Metropolitan Bank
and Trust Co. with the forged signature of Lim Sio Wan as indorser.
133

Upon the maturity date of the money market placement, Lim Sio Wan went to Allied to
withdraw it. She was then informed that the placement had been pre-terminated upon her
instructions. She denied giving any instructions and receiving the proceeds thereof. She
desisted from further complaints when she was assured by the bank’s manager that her
money would be recovered.

On January 24, 1984, Lim Sio Wan, realizing that the promise that her money would be
recovered would not materialize, sent a demand letter to Allied asking for the payment of
the first placement. Allied refused to pay Lim Sio Wan, claiming that the latter had
authorized the pre-termination of the placement and its subsequent release to Santos.

Consequently, Lim Sio Wan filed with the RTC a Complaint dated February 13, 1984
against Allied to recover the proceeds of her first money market placement. Allied claims
that Metrobank is the proximate cause of the loss of Lim Sio Wan’s money. It points
out that Metrobank guaranteed all prior indorsements inscribed on the manager’s check,
and without Metrobank’s guarantee, the present controversy would never have occurred.

ISSUE:

Is Allied Bank liable along with Metrobank for the conversion of Lim Sio Wan’s funds?

RULING:
Yes. Proximate cause is "that cause, which, in natural and continuous sequence,
unbroken by any efficient intervening cause, produces the injury and without which the
result would not have occurred." Thus, there is an efficient supervening event if the event
breaks the sequence leading from the cause to the ultimate result. To determine the
proximate cause of a controversy, the question that needs to be asked is: If the event did
not happen, would the injury have resulted? If the answer is NO, then the event is the
proximate cause.

In the instant case, the trial court correctly found Allied negligent in issuing the manager’s
check and in transmitting it to Santos without even a written authorization. In fact, Allied
did not even ask for the certificate evidencing the money market placement or call up Lim
Sio Wan at her residence or office to confirm her instructions. Both actions could have
prevented the whole fraudulent transaction from unfolding. Allied’s negligence must be
considered as the proximate cause of the resulting loss.

Petition PARTLY GRANTED. CA and RTC Decisions AFFIRMED with MODIFICATION


for both Banks to equitably bear the injury in proportion to their degree of negligence.
134

Case Title
Stokes vs. Malayan Insurance Co., Inc. 127 SCRA 766
(1984)

Topic Conditions before loss. As a condition precedent to the


right of recovery, there must be compliance on the part of
the insured with the terms of the policy.

Digested By: TEMBLOR, Haidee A.

Characters and Their Roles

Petitioner: Stokes- Irish Citizen, without a Philippine Driver’s License, drove the car of
Adolfson who had a car insurance policy under Malayan Insurance.

Respondent: Malayan Insurance- Insurer

FACTS:

Adolfson had a subsisting MALAYAN car insurance policy with the above coverage on
November 23, 1969 when his car collided with a car owned by Cesar Poblete, resulting
in damage to both vehicles.
At the time of the accident, Adolfson’s car was being driven by James Stokes, who was
authorized to do so by Adolfson. Stokes, an Irish citizen who had been in the Philippines
as a tourist for more than ninety days, had a valid and subsisting Irish driver’s license but
without a Philippine driver’s license.
After the collision, Adolfson filed a claim with MALAYAN but the latter refused to pay,
contending that Stokes was not an authorized driver under the “Authorized Driver” clause
of the insurance policy in relation to Section 21 of the Land Transportation and Traffic
Code.

Under the insurance policy, “authorized driver” refers to—

“(a)The insured; and


135

“(b)Any person driving on the insured’s order or with his permission.

“PROVIDED that the person driving is permitted in accordance with the licensing or other
laws or regulations to drive the motor vehicle and is net disqualified from driving such
motor vehicle by order of a court of law or by reason of any enactment or regulation in
that behalf.”

The cited Section 21 of the Land Transportation and Traffic Code provides:

“Operation of motor vehicles by tourists.—Bona fide tourists and similar transients who
are duly licensed to operate motor vehicles in their respective countries may be allowed
to operate motor vehicles during but not after ninety days of their sojourn in the
Philippines.

xxx xxx xxx

“After ninety days, any tourist or transient desiring to operate motor vehicles shall pay
fees and obtain and carry a license as hereinafter provided.” (Italics supplied.)

Unable to convince MALAYAN to pay, Stokes and Adolfson brought suit and succeeded
in getting a favorable judgment, although Stokes had ceased to be authorized to drive a
motor vehicle in the Philippines at the time of the accident, he having stayed therein as a
tourist for over 90 days without having obtained a Philippine driver’s license.

ISSUE/S:

1. Whether or not the failure on the part of Stokes to have a Philippine driver’s license
is not such a defect that can be considered as fatal to the contract of insurance, due
to the fact that Stokes still had a valid and unexpired Irish license.

2. Whether or not the defendant is in estoppel because it allowed the plaintiff to pay
the insurance premium even after the accident occurred.

RULING:

1. No, it is fatal.
136

A contract of insurance is a contract of indemnity upon the terms and conditions specified
therein. When the insurer is called upon to pay in case of loss or damage, he has the right
to insist upon compliance with the terms of the contract. If the insured cannot bring himself
within the terms and conditions of the contract, he is not entitled as a rule to recover for
the loss or damage suffered. For the terms of the contract constitute the measure of the
insurer’s liability, and compliance therewith is a condition precedent to the right of
recovery. (Young vs. Midland Textile Insurance Co., 30 Phil. 617.)

Under the “authorized driver” clause, an authorized driver must not only be permitted to
drive by the insured. It is also essential that he is permitted under the law and regulations
to drive the motor vehicle and is not disqualified from so doing under any enactment or
regulation.

At the time of the accident, Stokes had been in the Philippines for more than 90 days.
Hence, under the law, he could not drive a motor vehicle without a Philippine driver’s
license. He was therefore not an “authorized driver” under the terms of the insurance
policy in question, and MALAYAN was right in denying the claim of the insured.

2. There is no estoppel.

Acceptance of premium within the stipulated period for payment thereof, including the
agreed period of grace, merely assures continued effectivity of the insurance policy in
accordance with its terms. Such acceptance does not estop the insurer from interposing
any valid defense under the terms of the insurance policy.

The principle of estoppel is an equitable principle rooted upon natural justice which
prevents a person from going back on his own acts and representations to the prejudice
of another whom he has led to rely upon them. The principle does not apply to the instant
case. In accepting the premium payment of the insured, MALAYAN was not guilty of any
inequitable act or representation. There is nothing inconsistent between acceptance of
premium due under an insurance policy and the enforcement of its terms Stokes vs.
Malayan Insurance Co., Inc., 127 SCRA 766, No. L-34768 February 24, 1984

Reveresed. Judgment in favor of Malayan.


137

Case Title
United Merchants Corporation vs. Country Bankers
Insurance Corporation. 676 SCRA 382 (2012)

Topic Fire Insurance

Digested By: Group 1

FACTS:

Petitioner United Merchants Corporation (UMC) is engaged in the business of buying,


selling, and manufacturing Christmas lights. UMC leased a warehouse at 19-B Dagot
Street, San Jose Subdivision, Barrio Manresa, Quezon City, where UMC assembled and
stored its products.

On 6 September 1995, UMCs General Manager Alfredo Tan insured UMCs stocks in
trade of Christmas lights against fire with defendant Country Bankers Insurance
Corporation (CBIC) for P 15,000,000.00. The Fire Insurance Policy No. F-HO/95- 576
(Insurance Policy) and Fire Invoice No. 12959A, valid until 6 September 1996.

On 7 May 1996, UMC and CBIC executed Endorsement F/96-154 and Fire Invoice No.
16583A to form part of the Insurance Policy. Endorsement F/96-154 provides that UMCs
stocks in trade were insured against additional perils, to wit: typhoon, flood, ext. cover,
and full earthquake. The sum insured was also increased to P50,000,000.00 effective 7
May 1996 to 10 January 1997.

On 9 May 1996, CBIC issued Endorsement F/96-157 where the name of the assured was
changed from Alfredo Tan to UMC.

On 3 July 1996, a fire gutted the warehouse rented by UMC. CBIC designated CRM
Adjustment Corporation (CRM) to investigate and evaluate UMCs loss by

reason of the fire. CBICs reinsurer, Central Surety, likewise requested the National
Bureau of Investigation (NBI) toconduct a parallel investigation.

On 6 July 1996, UMC, through CRM, submitted to CBIC its Sworn Statement of Formal
Claim, with proofs of its loss.
138

ISSUE:

Whether or not UMC is entitled to claim from CBIC the full coverage of its fire insurance
policy

RULING:

No, Burden of proof is the duty of any party to present evidence to establish his claim or
defense by the amount of evidence required by law, which is preponderance of evidence
in civil cases. The party, whether plaintiff or defendant, who asserts the affirmative of the
issue has the burden of proof to obtain a favorable judgment. Particularly, in insurance
cases, once an insured makes out a prima facie case in its favor, the burden of evidence
shifts to the insurer to controvert the insureds prima facie case.

In the present case, UMC established a prima facie case against CBIC. CBIC does not
dispute that UMCs stocks in trade were insured against fire under the Insurance Policy
and that the warehouse, where UMCs stocks in trade were stored, was gutted by fire on
3 July 1996, within the duration of the fire insurance. However, since CBIC alleged an
excepted risk, then the burden of evidence shifted to CBIC to prove such exception.

An insurer who seeks to defeat a claim because of an exception or limitation in the policy
has the burden of establishing that the loss comes within the purview of the exception or
limitation. If loss is proved apparently within a contract of insurance, the burden is upon
the insurer to establish that the loss arose from a cause of loss which is excepted or for
which it is not liable, or from a cause which limits its liability.

In Uy Hu & Co. v. The Prudential Assurance Co., Ltd., the Court held that where a fire
insurance policy provides that if the claim be in any respect fraudulent, or if any false
declaration be made or used in support thereof, or if any fraudulent means or devices are
used by the Insured or anyone acting on his behalf to obtain any benefit under this Policy,
and the evidence is conclusive that the proof of claim which the insured submitted was
false and fraudulent both as to the kind, quality and amount of the goods and their value
destroyed by the fire, such a proof of claim is a bar against the insured from recovering
on the policy even for the amount of his actual loss.

In the present case, as proof of its loss of stocks in trade amounting to P 50,000,000.00,
UMC submitted its Sworn Statement of Formal Claim together with

the following documents: (1) letters of credit and invoices for raw materials, Christmas
lights and cartons purchased; (2) charges for assembling the Christmas lights; and (3)
139

delivery receipts of the raw materials. However, the charges for assembling the Christmas
lights and delivery receipts could not support its insurance claim.

The Insurance Policy provides that CBIC agreed to insure UMCs stocks in trade. UMC
defined stock in trade as tangible personal property kept for sale or traffic. Applying UMCs
definition, only the letters of credit and invoices for raw materials, Christmas lights and
cartons may be considered.

It has long been settled that a false and material statement made with an intent to deceive
or defraud voids an insurance policy. The most liberal human judgment cannot attribute
such difference to mere innocent error in estimating or counting but to a deliberate intent
to demand from insurance company’s payment for indemnity of goods not existing at the
time of the fire.

This constitutes the so-called fraudulent claim which, by express agreement between the
insurers and the insured, is a ground for the exemption of insurers from civil liability.
140

Case Title
Santa Ana vs. Commercial Union Assurance Company,
Ltd., 55 Phil. 329.

Topic Section 93

Digested By: TUDIO, Gabriel L.

Characters and Their Roles

Petitioner: Ulpiano Santa Ana

Respondent: Phoenix Assurance Company, Guardian Assurance Company, Globe


and Rutgers Fire Insurance Company of New York, Commercial Union Assurance
Company

FACTS:
Ulpiano Santa Ana built a house of strong materials with a galvanized iron roof and took
out the following fire insurance policy on the house:
1. On October 1, 2925, Phoenix Assurance Company and a six-thousand-peso policy
in the Guardian Assurance Company for a period of one year until 4:00PM paying the
respective premiums of P97.50 and P195 to said companies through their duly authorized
Philippine agent, Kerr & Company.
2. On December 16, 1925, Globe and Rutgers Fire Insurance Company of New York,
and the Commercial Union Assurance Company, Limited of London, through their
common agent duly authorized to represent them in the Philippine Islands, the Pacific
Commercial Company to be effective for one year.
3. On September 20, 1926, "Filipinas, Compañía de Seguros," which issued the one-
year policy.

It should be noted that on November 19, 1925 before the second set of insurance
company, Ulpiano Santa Ana mortgaged said house to the plaintiff Rafael Garcia for
P5,000, for a period of two years, and the policies issued by the Phoenix Assurance
Company and the Guardian Assurance Company, Limited, were endorsed to the
mortgagee, Rafael Garcia.

Now, about 3 o'clock in the morning of October 1, 1926, that is, twelve hours before the
expiration of the policies issued by the Phoenix Assurance Company and the Guardian
141

Assurance Company fire broke out in the insured house, where Ulpiano Santa Ana and
his family lived, starting in the ceiling of the living room where the plaintiff and his family
were at that time sleeping, consuming the dwelling and every combustible object within.

Ulpiano Santa Ana gave notice in due time of the loss to each and every one of the
companies in which he had insured the house and demanded payment of the respective
policies but the assurance companies refused payment on the ground that the claim of
P21,000 filed by him was fraudulent, being in excess of the real value of the insured
property; that none of said companies had been informed of the existence of the other
policies in the other companies, and that the fire was intentional.

Sta. Ana filed civil cases in RTC against The Commercial Union Assurance Company,
Limited in case No. 31263; the Globe and Rutgers Fire Insurance Company of New York
in case No. 31264; and the Phoenix Assurance Company, Limited, the Guardian
Assurance Company, Limited, and the "Filipinas, Compania de Seguros", incase No.
31322. All the defendants are absolved in their alleged liabilities by the RTC. Hence this
petition.

ISSUE:
Whether or not the insured can claim against the insurance companies.

RULING:
NO.

Without deciding whether notice of other insurance upon the same property must be given
in writing, or whether a verbal notice is sufficient to render an insurance valid which
requires such notice, whether oral or written, the SC held that in the absolute absence of
such notice when it is one of the conditions specified in the fire insurance policy, the policy
is null and void. Since the policy is null and void, plaintiff cannot recover from the
defendant’s insurance companies.

Moreover, the Honorable Supreme Court upheld the finding of the trial court that the
policies provide that no other insurance should be admitted upon the property thereby
assured without the consent of said companies duly given by endorsement.

In this case, it has been provided in English and Spanish in notices attached to the
insurance policies issued by the Phoenix Assurance Co., Ltd., and the Guardian
Assurance Company upon the first of October, 1925 , and by the Globe and Rutgers Fire
Insurance Company of New York, and the Commercial Union, Company, upon the 19th
of December of the same year, that no other insurance should be admitted upon the
142

property thereby assured without the consent of said companies duly given by
endorsement. It is clear that there is an express provision in policies that no other
insurance should be admitted upon the property thereby assured without the consent of
said companies duly given by endorsement.

Thus, insured can claim against the insurance companies.


143

Case Title
Guingon vs. Del Monte, 20 SCRA 1043

Topic Stipulation Pour Autrui

Digested By: ABONAL, Maria Lou Isabel P.

Characters and Their Roles

Petitioner: Heirs of Gervacio Guingon

Respondent: Iluminado del Monte and Julio Aguilar

FACTS:
■ Julio Aguilar owner and operator of several jeepneys insured them
with Capital Insurance & Surety Co., Inc.
■ February 20, 1961: Along the intersection of Juan Luna and Moro
streets, City of Manila, the jeepneys operated by Aguilar driven by
Iluminado del Monte and Gervacio Guingon bumped and Guingon
died some days after
■ Iluminado del Monte was charged with homicide thru reckless
imprudence and was penalized 4 months imprisonment
■ The heirs of Gervacio Guingon filed an action for damages praying
that P82,771.80 be paid to them jointly and severally by the driver
del Monte, owner and operator Aguilar, and the Capital Insurance &
Surety Co., Inc.
■ CFI: Iluminado del Monte and Julio Aguilar jointly and severally to
pay plaintiffs the sum of P8,572.95 as damages for the death of
their father, plus P1,000.00 for attorney's fees plus costs.
■ Capital Insurance and Surety Co., Inc. is hereby sentenced to pay
P5,000 plus P500 as attorney's fees and costs to be applied in
partial satisfaction of the judgment rendered against Iluminado del
Monte and Julio Aguilar in this case
144

ISSUE:
1. Whether or not there was a stipulation pour autriu that will enable the heirs to sue
against Capital Insurance and Surety Co., Inc.?

RULING:
YES.

The right of a person injured to sue the insurer of the party at fault depends on
whether the contract of insurance was intended to benefit third persons. The test applied
here is: Where the contract provides for indemnity against liability to third persons, then
third persons to whom the insured is liable, can sue the insurer. On the other hand, where
the contract is for indemnity against actual loss or payment, then third persons cannot
proceed against the insurer, the contract being solely to reimburse the insured for liability
actually discharged by him through payment to third persons, said third persons' recourse
being thus limited to the insured alone. The policy in the present case, is one whereby
the insurer agreed to indemnify the insured "against all sums . which the Insured shall
become legally liable to pay in respect of: a. death of or bodily injury to any person . . ."
Clearly, therefore, it is one for indemnity against liability from the fact that the insured is
liable to the third person, such third person is entitled to sue the insurer.
145

Case Title
Go Tiaco vs. Union Insurance, 40 Phil. 40 (1919)

Topic Marine Insurance; Perils of the Sea vs Peril of the Ship;


Liability of Insurer vs Shipowner

Digested By: BARROSO, Jon Carlow L.

Characters and Their Roles

Petitioner: La Razon Social “Go Tiaco y Hermanos”

Respondent:Union Insurance Society of Canton LTD.

FACTS:

This is an action on a policy of marine insurance issued by the Union Insurance Society
of Canton, Ltd., upon a cargo of rice belonging to Go Tiaoco brothers. The rice was
transported on board the steamship Hondagua from Saigon to Cebu. When the rice was
being discharged in Cebu, it was discovered that 1,473 sacks was damaged by sea water.
The loss to the owners of the rice was valued at Php 3875.25. The trial court found that
the inflow of the sea water during the voyage was due to a defect in one of the drain pipes
of the ship and concluded that the loss was not covered by the policy of insurance. The
court found in effect that the opening above described had resulted in course of time from
ordinary wear and tear and not from the straining of the ship in rough weather on that
voyage.

The drain pipe which served as a discharge from the water closet passed down through
the compartment where the rice in question was stowed and thence out to sea through
the wall of the compartment, which was a part of the wall of the ship. The joint or elbow
where the pipe changed its direction was of cast iron; and in course of time it had become
corroded and abraded until a longitudinal opening had appeared in the pipe about one
inch in length. This hole had been in existence before the voyage was begun, and an
attempt had been made to repair it by filling with cement and bolting over it a strip of iron.

The policy of insurance was signed upon a form long in use among companies engaged
in maritime insurance. It purports to insure the cargo from the following among other risks:
"Perils . . . of the seas, men of war, fire, enemies, pirates, rovers, thieves, jettisons, . . .
146

barratry of the master and mariners, and of all other perils, losses, and misfortunes that
have or shall come to the hurt, detriment, or damage of the said goods and merchandise
or any part thereof."

ISSUE:

1. Whether or not perils of the sea includes "entrance of water into the ships hold through
a defective pipe"?

2. Whether or not there is an implied warranty on the seaworthy of the vessel in every
marine insurance contract"?

RULING:

(1) No.

Perils of the Sea vs Peril of the Ship

Ø In the first place it is determined that the words "all other perils, losses, and
misfortunes" are to be interpreted as covering risks which are of like kind (ejusdem
generis) with the particular risks which are enumerated in the preceding part of the same
clause of the contract. Ever since the case of Cullen vs. Butler, when they first became
the subject of judicial construction, they have always been held or assumed to be
restricted to cases 'akin to' or resembling' or 'of the same kind as' those specially
mentioned. I see no reason for departing from this settled rule. In marine insurance it is
above all things necessary to abide by settled rules and to avoid anything like novel
refinements or a new departure."

Ø It is settled that a loss which, in the ordinary course of events, results from the natural
and inevitable action of the sea, from the ordinary wear and tear of the ship, or from the
negligent failure of the ship's owner to provide the vessel with proper equipment to convey
the cargo under ordinary conditions, is not a peril of the sea. Such a loss is rather due to
what has been aptly called the "peril of the ship."

Ø The insurer undertakes to insure against perils of the sea and similar perils, not against
perils of the ship.

Ø In the present case the entrance of the sea water into the ship's hold through the
defective pipe already described was not due to any accident which happened during the
147

voyage, but to the failure of the ship's owner properly to repair a defect of the existence
of which he was apprised. The loss was therefore more analogous to that which directly
results from simple unseaworthiness than to that which results from perils of the sea.

(2)

Liability of Insurer vs Shipowner

Ø There is no room to doubt the liability of the shipowner for such a loss as occurred in
this case. By parity of reasoning the insurer is not liable; for, generally speaking, the
shipowner excepts the perils of the sea from his engagement under the bill of lading, while
this is the very peril against which the insurer intends to give protection. As applied to the
present case it results that the owners of the damaged rice must look to the shipowner
for redress and not to the insurer.
148

Case Title
White Gold Marine Services vs. Pioneer Insurance, 464
SCRA 448 (2005)

Topic Insurance Business

Digested By: BOISER, Virgilio IV A.

FACTS:

White Gold, petitioner, procured a protection and indemnity for its vessel from the
Steamship Mutual Underwriting Association which is a protection and indemnity club
(P&I) through Pioneer Insurance and Security Corporation. Subsequently, White Gold
was issued a Certificate of Entry and Acceptance. When petitioner failed to fully pay its
account, Steamship Mutual refused to renew the coverage.

Steamship thereafter filed a case of collection of sum of money for the unpaid balance of
the petitioner while the latter filed before the Insurance Commissioner a case against
Steamship for violating Sections 186 and 187 (192 and 193) of the Insurance Code, while
Pioneer violated Sections 299, 300 and 301 (307, 308, 309) in relation to Sections 302
and 303 (310, 311), thereof.

The Insurance Commissioner dismissed the complaint and said that there is no need for
the Steamship Mutual to procure license because it was not engage in insurance
business and was only a protection and indemnity club. Likewise, it ruled that Pioneer
need not secure another license as an insurance agent and/or a broker of Steamship
Mutual because it was not engaged in insurance business and Pioneer already had a
license hence procurement of separate license as an insurance agent would only be
superfluous. CA affirmed the decision of Insurance Commissioner.

ISSUE:

(1) Whether or not Steamship Mutual, a Protection and Indemnity Club (P & I), is
engaged in the insurance business in the Philippines.
149

(2) Does Pioneer need a license as an insurance agent/broker for Steamship


Mutual?

RULING:

(1) For the first issue, the court said YES.

Section 2 of the Insurance Code enumerates what constitutes doing an insurance


business or transacting an insurance business. These are:

(a) making or proposing to make, as insurer, any insurance contract;

(b) making, or proposing to make, as surety, any contract of suretyship as a vocation and
not as merely incidental to any other legitimate business or activity of the surety.

(c) doing any kind of business, including a reinsurance business, specifically recognized
as constituting the doing of an insurance business within the meaning of this Code;

(d) doing or proposing to do any business in substance equivalent to any of the foregoing
in a manner designed to evade the provisions of this Code.

The test to determine if a contract is an insurance contract or not, depends on the nature
of the promise, the act required to be performed, and the exact nature of the agreement
in the light of the occurrence, contingency, or circumstances under which the performance
becomes requisite. It is not by what it is called. In particular, a marine insurance
undertakes to indemnify the assured against marine losses, such as the losses incident
to a marine adventure. Section 99 (101) of the Insurance Code enumerates the coverage
of marine insurance.

Relatedly, a mutual insurance company is a cooperative enterprise where the members


are both the insurer and insured. In it, the members all contribute, by a system of
premiums or assessments, to the creation of a fund from which all losses and liabilities
are paid, and where the profits are divided among themselves, in proportion to their
interest. Additionally, mutual insurance associations, or clubs, provide three types of
coverage, namely, protection and indemnity, war risks, and defense costs.

A P & I Club is a form of insurance against third party liability, where the third party is
anyone other than the P & I Club and the members. By definition then, Steamship Mutual
as a P & I Club is a mutual insurance association engaged in the marine insurance
business.

The records reveal Steamship Mutual is doing business in the country albeit without the
requisite certificate of authority mandated by Section 187 (193) of the Insurance Code. It
150

maintains a resident agent in the Philippines to solicit insurance and to collect payments
in its behalf. We note that Steamship Mutual even renewed its P & I Club cover until it
was cancelled due to non-payment of the calls. Thus, to continue doing business here,
Steamship Mutual or through its agent Pioneer, must secure a license from the Insurance
Commission.

Since a contract of insurance involves public interest, regulation by the State is


necessary. Thus, no insurer or insurance company is allowed to engage in the insurance
business without a license or a certificate of authority from the Insurance Commission.

(2) For the second issue, the court ruled that, YES, although Pioneer is already
licensed as an insurance company, it needs a separate license to act as insurance agent
for Steamship Mutual. Section 299 (307) of the Insurance Code clearly states:

SEC. 307

No person shall act as an insurance agent or as an insurance broker in the solicitation or


procurement of applications for insurance, or receive for services in obtaining insurance,
any commission or other compensation from any insurance company doing business in
the Philippines or any agent thereof, without first procuring a license so to act from the
Commissioner, which must be renewed annually on the first day of January, or within six
months thereafter. . .
151

Case Title
Cathay Insurance Co. vs. Court of Appeals, 151 SCRA
710 (1987)

Topic Construction/ Interpretation

Digested By: BUSTAMANTE, Anne Georgette A.

Characters and Their Roles

Petitioner: Remington Industrial Sales Corporation

Respondent: Cathay Insurance Co

Doctrine : A cardinal rule in the interpretation of contracts, namely, that any ambiguity
therein should be construed against the maker/issuer/drafter thereof, namely, the insurer.

FACTS:

A complaint was filed by Remington Industrial Sales Corporation against Cathay


Insurance Co seeking collection of the sum of P868,339.15 representing Remington
Industrial Sales Corporations’ losses and damages incurred in a shipment of seamless
steel pipes under an insurance contract in favor of Remington Industrial Sales
Corporation as the insured, consignee or importer of aforesaid merchandise while in
transit from Japan to the Philippines on board vessel SS "Eastern Mariner." The total
value of the shipment was P2,894,463.83 at the prevailing rate of P7.95 to a dollar in
June and July 1984, when the shipment was made.

Remington Industrial Sales Corporation’s Contention:

1. Rust is not an inherent vice of the seamless steel pipes without interference of external
factors.

2. Shipment is covered by an implied coverage from the phrase “perils of the sea”
mentioned in the opening sentence of the policy
152

Cathay Insurance’s Contention

1. Rusting is not a risk insured against, since a risk to be insured against should be a
casualty or some casualty, something which could not be foreseen as one of the
necessary incidents of adventure.

2.A loss occasioned by an inherent defect or vice in the insured article is not within the
terms of the policy

3. The questioned shipment is not covered by a “square provision of the contract

Issue :

Whether the rusting of steel pipes in the course of a voyage is a "peril of the sea" in view
of the toll on the cargo of wind, water, and salt conditions.

RULING

Yes, there is no question that the rusting of steel pipes in the course of a voyage is a
"peril of the sea" in view of the toll on the cargo of wind, water, and salt conditions. At any
rate if the insurer cannot be held accountable therefor, the Court would fail to observe a
cardinal rule in the interpretation of contracts, namely, that any ambiguity therein should
be construed against the maker/issuer/drafter thereof, namely, the insurer. Besides, the
precise purpose of insuring cargo during a voyage would be rendered fruitless.
153

Case Title
Choa Tiek Seng vs. Court of Appeals, 183 SCRA 223
[1990]

Topic Marine Insurance

Digested By: CAGAPE, Joanna Paula M.

Characters and Their Roles

Parties

Choa Tiek Seng-businessman, petitioner

Filipino Merchant’s Insurance Company, Inc.-insurer

FACTS:

Choa Tiek Seng imported some lactose crystals from Holland on November 4, 1976 .
The importation involved fifteen (15) metric tons packed in 600 6-ply paper bags with
polythelene inner bags, each bag at 25 kilos net.

The goods were loaded at the port at Rotterdam in sea vans on board the vessel "MS
Benalder' as the mother vessel, and thereafter aboard the feeder vessel "Wesser Broker
V-25" of respondent Ben Lines Container, Ltd. (Ben Lines for short).

The goods were insured by the respondent Filipino Merchants' Insurance Co., Inc.
(insurance company for short) for the sum of P98,882.35, the equivalent of US$8,765.00
plus 50% mark-up or US$13,147.50, against all risks under the terms of the insurance
cargo policy.

Upon arrival at the port of Manila, the cargo was discharged into the custody of the
arrastre operator respondent E. Razon, Inc. (broker for short), prior to the delivery to
petitioner through his broker. It was found out the 600 bags delivered to petitioner, 403
154

were in bad order. The surveys showed that the bad order bags suffered spillage and loss
later valued at P33,117.63.

The case

Petitioner filed a claim for said loss dated February 16, 1977 against respondent
insurance company in the amount of P33,117.63 as the insured value of the loss.

Contention of Filipino Merchant’s Insurance Company:

Respondent insurance company rejected the claim alleging that:

1. Assuming that spillage took place while the goods were in transit, petitioner
and his agent failed to avert or minimize the loss by failing to recover spillage from
the sea van, thus violating the terms of the insurance policy sued upon; and

2. Assuming that the spillage did not occur while the cargo was in transit, the
said 400 bags were loaded in bad order, and that in any case, the van did not carry
any evidence of spillage.

Choa Tiek Seng filed the complaint dated August 2, 1977 in the Regional Trial Court of
Manila against respondent insurance company seeking payment of the sum of
P33,117.63 as damages plus attorney's fees and expenses of litigation.

Issues:

1. Whether or not the insured shipment sustained any damage/loss

2. Whether or not the “all risks” coverage covers only losses occasioned by or
resulting from “extra and fortuitous events” despite the clear and unequivocal definition of
the term made and contained in the policy sued upon

Ruling:

1. Yes.

In the first place it was respondent insurance company which undertook the protective
survey aforestated relating to the goods from the time of discharge up to the time of
155

delivery thereof to the consignee's warehouse, so that it is bound by the report of its
surveyor which is the Adjustment Corporation of the Philippines. 5 The Worldwide Marine
Cargo Survey Corporation of the Philippines was the vessel's surveyor. The survey report
of the said Adjustment Corporation of the Philippines reads as follows:

During the turn-over of the contents delivery from the cargo sea van by the representative
of the shipping agent to consignee's representative/ Broker (Saint Rose Forwarders), 403
bags were bursted and/or torn, opened on one end contents partly spilled.

The Honorable Supreme Court held that “the authenticity of the said survey report need
not be established in evidence as it is binding on respondent insurance company who
caused said protective survey.

There is no question, therefore, that there were 403 bags in damaged condition delivered
and received by petitioner.

2. Yes.

In Gloren Inc. vs. Filipinas Cia. de Seguros, it was held that an all risk insurance
policy insures against all causes of conceivable loss or damage, except as otherwise
excluded in the policy or due to fraud or intentional misconduct on the part of the insured.
It covers all losses during the voyage whether arising from a marine peril or not, including
pilferage losses during the war.

In the present case, the "all risks" clause of the policy sued upon reads as follows:

This insurance is against all risks of loss or damage to the subject matter
insured but shall in no case be deemed to extend to cover loss, damage, or
expense proximately caused by delay or inherent vice or nature of the
subject matter insured. Claims recoverable hereunder shall be payable
irrespective of percentage.

The terms of the policy are so clear and require no interpretation. The insurance policy
covers all loss or damage to the cargo except those caused by delay or inherent vice or
nature of the cargo insured. It is the duty of the respondent insurance company to
156

establish that said loss or damage falls within the exceptions provided for by law,
otherwise it is liable therefor.

An "all risks" provision of a marine policy creates a special type of insurance which
extends coverage to risks not usually contemplated and avoids putting upon the insured
the burden of establishing that the loss was due to peril falling within the policy's coverage.
The insurer can avoid coverage upon demonstrating that a specific provision expressly
excludes the loss from coverage.

In this case, the damage caused to the cargo has not been attributed to any of the
exceptions provided for nor is there any pretension to this effect. Thus, the liability of
respondent insurance company is clear.
157

Case Title
Filipino Merchants Insurance Co., vs. Court of Appeals,
179 SCRA 638 (1989)

Topic Insurable Interest

Digested By: CASANOVA, Jenny Mae I.

Characters and Their Roles

Petitioner: Filipino Merchants Insurance Co.

Respondent: Court of Appeals and Choa Tiek Seng

FACTS:

The plaintiff insured the shipment of 600 metric tons of fishmeal in new gunny bags of 90
kilos each with defendant insurance company Filipino Merchants Insurance Co. against
all risks under warehouse to warehouse terms. However, the actual imported shipment
was only 59.940 metric tons not 600 tons at $395.42 a ton CNF Manila. The fishmeal in
666 new gunny bags were unloaded from the ship at Manila unto the arrastre contractor
E. Razon, Inc. and defendant’s surveyor ascertained and certified that in such discharge
105 bags were in bad order condition as jointly surveyed by the ship’s agent and the
arrastre contractor. The condition of the bad order was reflected in the turn over survey
report of Bad Order cargoes Nos. 120320 to 120322. Similarly, when the arrastre
contractor surveyed the cargo before its delivery to the consignee, the said cargo was
also in bad condition and it was reflected in the certificates as well. Accordingly, the extent
of the loss on the bad order cargo totalled 227 bags amounting to 12, 148 kilos.

As a result, the plaintiff made a formal claim against the defendant Filipino Merchants
Insurance Company for P51,568.62. However, the defendant Filipino Merchants
Insurance Company refused to pay the claim. Consequently, the plaintiff brought an
action against the defendant while the latter presented a third party complaint against the
vessel and the arrastre contractor.
158

After the trial, the lower court rendered judgment in favor of plaintiff and ordered the
defendant Filipino Merchants Insurance to pay the former. On appeal, the Court of
Appeals affirmed the decision of the lower court insofar as the award on the complaint is
concerned and modified the same with regard to the adjudication of the third-party
complaint. Hence, this petition.

ISSUES:

1. Whether or not the Court of Appeals erred in its interpretation and application of the ‘all
risks’ clause of the marine insurance policy when it held the petitioner liable to the private
respondent for the partial loss of the cargo.

2. Whether or not the private respondent had no insurable interest in the subject cargo,
resulting in the nullification of the marine insurance policy taken out by private respondent.

RULING:

1. No. The Court of Appeals did not err in its interpretation and application of the ‘all risks’
clause of the marine insurance policy.

An “all risks policy” should be read literally as meaning all risks whatsoever and covering
all losses by an accidental cause of any kind. An “all risks” policy has been evolved to
grant greater protection than that afforded by the “perils clause,” in order to assure that
no loss can happen through the incidence of a cause neither insured against nor creating
liability in the ship; it is written against all losses, that is, attributable to external causes.

Under an “all risks” policy, the burden is not on the insured to prove the precise cause of
loss or damage for which it seeks compensation. The insured under an “all risks insurance
policy” has the initial burden of proving that the cargo was in good condition when the
policy attached and that the cargo was damaged when unloaded from the vessel;
thereafter, the burden then shifts to the insurer to show the exception to the coverage. As
a general rule, the insurance company has the burden of proving that the loss is caused
by the risks excepted and for want of such proof, the company is liable. A marine
insurance policy providing that the insurance was to be “against all risks” must be
construed as creating a special insurance and extending to other risks than are usually
contemplated, and covers all losses except such as arise from the fraud of the insured.
In the present case, there being no showing that the loss was caused by any of the
excepted perils, the insurer is liable under the policy.

2. No, private respondent, as consignee of the goods in transit, has insurable interest in
said goods. In principle, anyone has an insurable interest in property who derives a benefit
159

from its existence or would suffer loss from its destruction whether he has or has not any
title in, or lien upon or possession of the property.

In the present case, the private respondent, as vendee/consignee of the goods in transit
has such existing interest therein as may be the subject of a valid contract of insurance.
His interest over the goods is based on the perfected contract of sale. The perfected
contract of sale between him and the shipper of the goods operates to vest in him an
equitable title even before delivery or before he performed the conditions of the sale. The
perfected contract of sale even without delivery vests in the vendee an equitable title, an
existing interest over the goods sufficient to be the subject of insurance.
160

Case Title
Caltex [Phil.] Inc. vs. Sulpicio Lines, Inc., 315 SCRA 709
(1999)

Topic Implied Warranty of Seaworthiness

Digested By: CONDA, Ana Ultima G.

Facts:

[This is Petition seeking to reverse the CA’s decision holding Caltex jointly liable with the
operator of MT Vector for damages]

1) [Dec. 19, 1987] During a particular voyage, motor tanker MT vector carried on
board 8,800 barrels of petroleum products shipped by Caltex enroute to Masbate
from Bataan.
2) On Dec. 20, 1987, MV Dona Paz [a passenger and cargo vessel operated by
Sulpicio Lines] left the port of Tacloban which was bound for Manila with 1,493
passengers and 59 crew members [but allegedly there was an estimate of 4,000
total passengers in which some were not in the passenger manifest].
3) At about 10:30 pm of the same day, the two vessels collided in the open sea
within the vicinity of Dumali Point bet. Marinduque and Oriental Mindoro. Only 24
survived.
4) On [March] 1988, the Board of Marine Inquiry found that the MT Vector’s
registered operator Soriano and actual operator [Vector Shipping Corp] were at
fault and responsible for its collision with MV Dona Paz.
5) In the RTC level, a complaint for Damages arising from Breach of Contract of
Carriage was filed against Sulpicio; Sulpicio, in turn filed a third-party complaint
against Vector Shipping Corporation and Caltex alleging that Caltex chartered
MT Vector with gross and evident bad faith knowing fully well that MT Vector was
improperly manned, ill-equipped, unseaworthy and a hazard to safe navigation
a) RTC dismissed the third party complaint for want of substantiation.
6) CA, however, reversed RTC’s ruling and included petitioner Caltex as one of
those liable for damages. [Both Vector Shipping and Caltex are held equally liable
a) Vector Shipping [(being the vessel at fault for the collision]
161

b) Caltex [being the charterer that negligently caused the shipping of


combustible cargo aboard an unseaworthy vessel]

Issue:

WON the charterer of a sea vessel liable for damages resulting from a collision between
the chartered vessel and a passenger ship

Ruling:

NO. The charterer has no liability for damages under Philippine Maritime laws.

The Supreme Court held that the carriers are deemed to warrant impliedly the
seaworthiness of the ship. For a vessel to be seaworthy, it must be adequately equipped
for the voyage and manned with a sufficient number of competent officers and crew.
[Section 3 [1] of the Carriage of Goods by Sea Act].

Further, the charterer of a vessel has no obligation before transporting its cargo to
ensure that the vessel it chartered complied with all legal requirements. The duty rests
upon the common carrier simply for being engaged in “public service.”

Because of the implied warranty of seaworthiness, shippers of goods, when


transacting with common carriers, are not expected to inquire into the vessel’s
seaworthiness, genuineness of its licenses and compliance with all maritime laws. To
demand more from shippers and hold them liable in case of failure exhibits nothing but
the futility of our maritime laws insofar as the protection of the public in general is
concerned.

Petitioner Caltex and Vector Shipping entered into a contract of affreightment, also known
as a voyage charter. The SC cited the case of Coastwise vs CA which states that
“Although a charter party may transform a common carrier into a private one, the same
however is not true in a contract of affreightment. Here it was held that MT Vector was
still a common carrier which is deemed to warrant impliedly the seaworthiness of the ship.
The provisions owed their conception to the nature of the business of common carriers.
162

Case Title
San Miguel Corporation vs. Heirs of S. Inguito, 384
SCRA 87 [2002]

Topic Marine Insurance

Digested By: DOLOR, Rhodilee Jean A.

FACTS:

San Miguel Corporation and Julius Ouano, owner of Ouano marine services, entered into
a time charter party agreement in which SMC will charter MV Dona Roberta to transports
beverages from Mandaue City to various points in Mindanao for two years.

In November 11, 1990 SMC issued a sail order to Captain Sabiniano Inquito to sail to
Cagayan and then to Mandaue.

At 4 am on November 12, typhoon Ruping was spotted. Dona Roberta set sail at 6 AM
on that day. At 7 AM, SMC radio operator Moreno called Captain Inquito advising him to
take shelter because of the storm, but the captain said they can manage.

Moreno called and advised Inquito to take shelter three times but the captain said they
can manage. By 1:15 AM on November 13, the captain radioed informing that the boat
was sinking and they need to be rescued by a helicopter. At 2:30 AM, the boat sank. Only
5 of the 25 crew members survived.

ISSUE: Whether or not SMC is liable for the loss and damage sustained during the
voyage

RULING: No.

In deciding the cases at bar, the Court of Appeals correctly resolved the issues with an
initial discussion of the definition and kinds of charter parties.
163

A charter party is a contract by virtue of which the owner or the agent of a vessel binds
himself to transport merchandise or persons for a fixed price. It has also been defined as
a contract by virtue of which the owner or the agent of the vessel leases for a certain price
the whole or a portion of the vessel for the transportation of goods or persons from one
port to another

A charter party may either be a


(1) bareboat or demise charter or
(2) contract of affreightment.

Under a demise or bareboat charter, the charterer mans the vessel with his own people
and becomes, in effect, the owner of the ship for the voyage or service stipulated, subject
to liability for damages caused by negligence.

In a contract of affreightment, on the other hand, the owner of the vessel leases part or
all of its space to haul goods for others. It is a contract for special service to be rendered
by the owner of the vessel. Under such contract the ship owner retains the possession,
command and navigation of the ship, the charterer or freighter merely having use of the
space in the vessel in return for his payment of the charter hire.

Otherwise put, a contract of affreightment is one by which the owner of a ship or other
vessel lets the whole or part of her to a merchant or other person for the conveyance of
goods, on a particular voyage, in consideration of the payment of freight.

A contract of affreightment may be either time charter, wherein the leased vessel is
leased to the charterer for a fixed period of time, or voyage charter, wherein the ship is
leased for a single voyage.

In both cases, the charterer provides for the hire of the vessel only, either for a
determinate period of time or for a single or consecutive voyage, the ship owner to supply
the ship's store, pay for the wages of the master of the crew, and defray the expenses for
the maintenance of the ship.

If the charter is a contract of affreightment, which leaves the general owner in


possession of the ship as owner for the voyage, the rights and the responsibilities
of ownership rest on the owner. The charterer is free from liability to third persons
in respect of the ship.
164

We concur with the findings of the Court of Appeals that the charter party in these cases
was a contract of affreightment, contrary to petitioner Ouano's protestation that it was a
demise charter, as shown by the stipulations in the Time Charter Party Agreement:

It appearing that Ouano was the employer of the captain and crew of the M/V Doña
Roberta during the term of the charter, he therefore had command and control over the
vessel. His son, Rico Ouano, even testified that during the period that the vessel was
under charter to SMC, the Captain thereof had control of the navigation of all voyages.

Under the foregoing definitions, as well as the clear terms of the Charter Party Agreement
between the parties, the charterer, SMC, should be free from liability for any loss or
damage sustained during the voyage unless it be shown that the same was due to
its fault or negligence.
165

Case Title
Roque vs. Intermediate Appellate Court 139 SCRA 596
(1985)

Topic Marine Insurance

Digested By: GALEA, Samantha Racel Jeanne R.

Characters and Their Roles

Petitioner: Isabela Roque

Respondent: Pioneer Insurance and Surety Corporation

FACTS:
Manila Bay Lighterage Corporation (Manila Bay) , a common carrier, entered into a
contract with Isabela Roque whereby the former would load and carry on board its barge
Mable 10 about 422.18 cubic meters of logs from Malampaya Sound, Palawan to North
Harbor, Manila. Roque insured the logs against loss for P100,000.00 with respondent
Pioneer Insurance and Surety Corporation.

On February 29, 1972, the petitioners loaded on the barge, logs at Malampaya Sound,
Palawan for carriage and delivery to North Harbor, Port of Manila, but the shipment never
reached its destination because Mable 10 sank with the 811 pieces of logs somewhere
off Cabuli Point in Palawan on its way to Manila. Later on, Roque wrote a letter to Manila
Bay demanding payment of P150,000.00 for the loss of the shipment plus P100,000.00
as unrealized profits but the latter ignored the demand. Another letter was sent to
respondent Pioneer claiming the full amount of P100,000.00 under the insurance policy
but respondent refused to pay on the ground that its liability depended upon the "Total
loss by Total Loss of Vessel only".

Now, Roque contends that the implied warranty of seaworthiness provided for in the
Insurance Code refers only to the responsibility of the shipowner who must see to it that
his ship is reasonably fit to make in safety the contemplated voyage. Roque states that a
mere shipper of cargo, having no control over the ship, has nothing to do with its
166

seaworthiness. They argue that a cargo owner has no control over the structure of the
ship, its cables, anchors, fuel and provisions, the manner of loading his cargo and the
cargo of other shippers, and the hiring of a sufficient number of competent officers and
seamen.

The barge where the logs were loaded was later found both by the lower court and
appellate court as not seaworthy such that it developed a leak. As such, Pioneer was
absolved from liability due to finding that there was a breach of implied warranty of
seaworthiness on the part of Roque and that the loss of the insured cargo was caused
by the "perils of the ship" and not by the "perils of the sea". It ruled that the loss is not
covered by the marine insurance policy.

ISSUE:
WON the insurer should be absolved from liability on the grounds that the vessel carrying
the insured cargo was unseaworthy and the loss of said cargo was caused not by the
perils of the sea but by the perils of the ship.

RULING:
Yes. The liability of the insurance company is governed by law. Section 113 of the
Insurance Code provides:

"In every marine insurance upon a ship or freight, or freightage, or upon any thing which
is the subject of marine insurance, a warranty is implied that the ship is seaworthy."

Section 99 of the same Code also provides in part:

'Marine insurance includes:


"(1) Insurance against loss of or damage to:
(a) Vessels, craft, aircraft, vehicles, goods, freights, cargoes, merchandise, . . . ."

From the above-quoted provisions, there can be no mistaking the fact that the term
"cargo" can be the subject of marine insurance and that once it is so made, the implied
warranty of seaworthiness immediately attaches to whoever is insuring the cargo whether
he be the shipowner or not.

Moreover, the fact that the unseaworthiness of the ship was unknown to the insured is
immaterial in ordinary marine insurance and may not be used by him as a defense in
order to recover on the marine insurance policy. Since the law provides for an implied
warranty of seaworthiness in every contract of ordinary marine insurance, it becomes
the obligation of a cargo owner to look for a reliable common carrier which keeps its
vessels in seaworthy condition. The shipper of cargo may have no control over the vessel
167

but he has full control in the choice of the common carrier that will transport his goods. Or
the cargo owner may enter into a contract of insurance which specifically provides that
the insurer answers not only for the perils of the sea but also provides for coverage of
perils of the ship.

In Roque’s complaint, it is alleged that "the barge Mable 10 of defendant carrier developed
a leak which allowed water to come in and that one of the hatches of said barge was
negligently left open by the person in charge thereof causing more water to come in", and
that "the loss of said plaintiffs' cargo was due to the fault, negligence, and/or lack of skill
of defendant carrier and/or defendant carrier's representatives on barge Mable 10." It is
quite unmistakable that the loss of the cargo was due to the perils of the ship rather than
the perils of the sea. The facts clearly negate the petitioners' claim under the insurance
policy.
168

Case Title
Phil. American General Insurance Co. vs. Court of
Appeals, 273 SCRA 262 (1997)

Topic Right of Subrogation

Digested By: IRAN, Jayson O.

Characters and Their Roles

Petitioner: Philippine American General Insurance Co.

Respondent: Court of Appeals

FACTS:

On July 6, 1983 Coca-cola loaded on board MV Asilda, owned and operated by Felman
Shipping Lines, 7,500 cases of 1-liter Coca-Cola soft drink bottles to be transported from
Zamboanga City to Cebu. The shipment was insured with petitioner Philamgen (for
brevity). On July 7, the vessel sank in Zamboanga del Norte. On July 15, Coca-Cola filed
a claim with respondent Felman for recovery of damages. Felman denied thus prompted
cocacola to file an insurance claim with Philamgen. Philamgen later on claimed its right
of subrogation against Felman which disclaimed any liability for the loss. Philamgen
alleged that the sinking and loss were due to the vessel's unseaworthiness, that the
vessel was improperly manned and its officers were grossly negligent. Felman filed a
motion to dismiss saying that there is no right of subrogation in favor of Philamgen was
transmitted by the shipper.

RTC rendered judgment in favor of Felman. It ruled that the vessel was seaworthy when
it left the port of Zamboanga as evidenced by the certificate issued by the Phil. Coast
Guard and the ship owner’s surveyor. On appeal, CA rendered judgment finding the
vessel unseaworthy for the cargo for being top-heavy and the cocacola bottles were also
improperly stored on deck. Nonetheless, the CA denied the claim of Philamgen, saying
that Philamgen was not properly subrogated to the rights and interests of the shipper plus
the filing of notice of abandonment had absolved the ship owner from liability under the
limited liability rule.
169

ISSUE:

Whether or not Philamgen was properly subrogated to the rights against Felman?

RULING:

YES. Article 2207 of the New Civil Code provides that, If the plaintiff's property has been
insured, and he has received indemnity from the insurance company for the injury or loss
arising out of the wrong or breach of contract complained of, the insurance company shall
be subrogated to the rights of the insured against the wrongdoer or the person who has
violated the contract. If the amount paid by the insurance company does not fully cover
the injury or loss, the aggrieved party shall be entitled to recover the deficiency from the
person causing the loss or injury. Therefore, the payment made by PHILAMGEN to Coca-
Cola Bottlers Philippines, Inc., gave the former the right to bring an action as subrogee
against FELMAN. Having failed to rebut the presumption of fault, the liability of FELMAN
for the loss of the 7,500 cases of 1-liter Coca-Cola soft drink bottles is inevitable.
170

Case Title
Pan Malayan Insurance Co. vs. Court of Appeals, 201
SCRA 382 (1991)

Topic Right of Subrogation

Digested By: MACULBE, Tresha Hyacinth A.

Characters and Their Roles

Petitioner: PANMALAY

Respondent:

FACTS:

In 1985, PANMALAY filed a complaint for damages with the RTC of Makati against private
respondents Erlinda Fabie and her driver. PANMALAY averred that: it is an insurer of a
Mitsubishi Colt Lancer car registered in the name of Canlubang Automotive Resources
Corporation; on May 26, 1985, due to the "carelessness, recklessness, and imprudence"
of the unknown driver of a pick-up with plate no. PCR-220, the insured car was hit and
suffered damages in the amount of P42,052.00; PANMALAY defrayed the cost of repair
of the insured car and, therefore, was subrogated to the rights of Canlubang against the
driver of the pick-up and his employer, Erlinda Fabie; and, despite repeated demands,
defendants, failed and refused to pay the claim of PANMALAY.

Private respondents filed a Motion for Bill of Particulars. In compliance therewith,


PANMALAY clarified, among others, that the damage caused to the insured car was
settled under the "own damage", coverage of the insurance policy, and that the driver of
the insured car was, at the time of the accident, an authorized driver duly licensed to drive
the vehicle. PANMALAY also submitted a copy of the insurance policy and the Release
of Claim and Subrogation Receipt executed by CANLUBANG in favor of PANMALAY.
171

Private respondents filed a Motion to Dismiss alleging that PANMALAY had no cause of
action against them. They argued that payment under the "own damage" clause of the
insurance policy precluded subrogation under Article 2207 of the Civil Code, since
indemnification thereunder was made on the assumption that there was no wrongdoer or
no third party at fault.

After hearing, the RTC issued an order dismissing PANMALAY's complaint for no cause
of action. The RTC ruled that that payment by PANMALAY of Canlubang's claim under
the "own damage" clause of the insurance policy was an admission by the insurer that
the damage was caused by the assured and/or its representatives.

The Court of Appeals in upholding the RTC decision, held that Section III-1 of the policy,
which was the basis for settlement of Canlubang's claim, did not cover damage arising
from collision or overturning due to the negligence of third parties as one of the insurable
risks.

ISSUE:

Whether or not PANMALAY was legally subrogated to the rights of Canlubang.

RULING:

Yes. Article 2207 of the Civil Code provides: "If the plaintiffs property has been insured,
and he has received indemnity from the insurance company for the injury or loss arising
out of the wrong or breach of contract complained of, the insurance company shall be
subrogated to the rights of the insured against the wrongdoer or the person who has
violated the contract."

Article 2207 of the Civil Code is founded on the well-settled principle of subrogation. If the
insured property is destroyed or damaged through the fault or negligence of a party other
172

than the assured, then the insurer, upon payment to the assured, will be subrogated to
the rights of the assured to recover from the wrongdoer to the extent that the insurer has
been obligated to pay. Payment by the insurer to the assured operates as an equitable
assignment to the former of all remedies which the latter may have against the third party
whose negligence or wrongful act caused the loss. The right of subrogation is not
dependent upon, nor does it grow out of, any privity of contract or upon written assignment
of claim. It accrues simply upon payment of the insurance claim by the insurer.

There are a few recognized exceptions to this rule. For instance, if the assured by his
own act releases the wrongdoer or third party liable for the loss or damage, from liability,
the insurer's right of subrogation is defeated. Similarly, where the insurer pays the
assured the value of the lost goods without notifying the carrier who has in good faith
settled the assured's claim for loss, the settlement is binding on both the assured and the
insurer, and the latter cannot bring an action against the carrier on his right of subrogation.
And where the insurer pays the assured for a loss which is not a risk covered by the
policy, thereby effecting "voluntary payment", the former has no right of subrogation
against the third party liable for the loss.

None of the exceptions are availing in the present case.

It must be emphasized that the lower court's ruling that the "own damage" coverage under
the policy implies damage to the insured car caused by the assured itself, instead of third
parties, proceeds from an incorrect comprehension of the phrase "own damage" as used
by the insurer. When PANMALAY utilized the phrase "own damage" — a phrase which,
incidentally, is not found in the insurance policy — to define the basis for its settlement of
CANLUBANG's claim under the policy, it simply meant that it had assumed to reimburse
the costs for repairing the damage to the insured vehicle. It is in this sense that the so-
called "own damage" coverage under Section III of the insurance policy is differentiated
from Sections I and IV-1 which refer to "Third Party Liability" coverage (liabilities arising
from the death of, or bodily injuries suffered by, third parties) and from Section IV-2 which
refer to "Property Damage" coverage (liabilities arising from damage caused by the
insured vehicle to the properties of third parties).
173

Neither is there merit in the Court of Appeals' ruling that the coverage of insured risks
under Section III-1 of the policy does not include to the insured vehicle arising from
collision or overturning due to the negligent acts of the third party. Not only does it stem
from an erroneous interpretation of the provisions of the section, but it also violates a
fundamental rule on the interpretation of property insurance contracts.

PANMALAY contends that the coverage of insured risks under the above section,
specifically Section III-1(a), is comprehensive enough to include damage to the insured
vehicle arising from collision or overturning due to the fault or negligence of a third party.
CANLUBANG is apparently of the same understanding. Based on a police report wherein
the driver of the insured car reported that after the vehicle was sideswiped by a pick-up,
the driver thereof fled the scene [Record, p. 20], CANLUBANG filed its claim with
PANMALAY for indemnification of the damage caused to its car. It then accepted payment
from PANMALAY, and executed a Release of Claim and Subrogation Receipt in favor of
latter.

Considering that the very parties to the policy were not shown to be in disagreement
regarding the meaning and coverage of Section III-1, specifically sub-paragraph (a)
thereof, it was improper for the appellate court to indulge in contract construction, to apply
the ejusdem generis rule, and to ascribe meaning contrary to the clear intention and
understanding of these parties.

For even if under the above circumstances PANMALAY could not be deemed subrogated
to the rights of its assured under Article 2207 of the Civil Code, PANMALAY would still
have a cause of action against private respondents. In the pertinent case of Sveriges
Angfartygs Assurans Forening v. Qua Chee Gan, the Court ruled that the insurer who
may have no rights of subrogation due to "voluntary" payment may nevertheless recover
from the third party responsible for the damage to the insured property under Article 1236
of the Civil Code.
174

Case Title
Aboitiz Shipping Corp. vs. Court of Appeals, 569 SCRA
294 (2008)

Topic Exception to the application of the doctrine of limited


liability of maritime law

Digested By: MAGUINDANAO, Da'ud A.

Characters and Their Roles

Petitioner: Aboitiz Shipping Corp

Respondent: Malayan Insurance Company, Traders Insurance Corporation, and


Equitable Insurance Corporation

FACTS:

This case is a consolidation of three cases involving three different respondents. All of
the cases involved are founded on the same factual antecedents. They are suing Aboitiz
company due to its negligence in ensuring that M/V P. Aboitiz is seaworthy. Aboitiz on
the other hand is raising the defense that the sinking of the ship was due to force majeure
and that the limited liability doctrine must be applied in this case.

ISSUE:

Whether or not Aboitiz can avail limited liability based on real and hypothecary doctrine
of maritime law.

RULING:

No. The doctrine of limited liability based on real and hypothecary doctrine of maritime
law cannot be applied in this case.
175

As held in the GAFLAC case, the court applied the doctrine of limited liability in favor of
Aboitiz due to the fact that there was no finding of Aboitiz’s negligence in the sinking of
the ship. However, in this case, there is a categorical finding that Aboitiz was negligent in
failing to take course of action that would prevent the sailing of the vessel into the typhoon.

As a general rule, a ship owner’s liability is merely co-extensive with his interest in the
vessel, except where actual fault is attributable to the shipowner. Thus, as an exception
to the limited liability doctrine, a shipowner or ship agent may be held liable for damages
when the sinking of the vessel is attributable to the actual fault or negligence of the
shipowner or its failure to ensure the seaworthiness of the vessel. The instant petitions
cannot be spared from the application of the exception to the doctrine of limited liability in
view of the unanimous findings of the courts below that both Aboitiz and the crew failed
to ensure the seaworthiness of the M/V P. Aboitiz.
176

Case Title
Oriental Assurance Corporation, vs. Court of Appeals
and Panama Saw Mill Co., Inc., G.R. No. 94052, August
09, 1991

Topic Constructive total loss

Digested By: MAYANGAO, Ian Jermie O.

Characters and Their Roles

Petitioner: Oriental Assurance Corporation

Respondent: Panama Saw Mill Co., Inc.,

FACTS:

An action to recover on a marine insurance policy, issued by petitioner in favor of private


respondent, arising from the loss of a shipment of apitong logs from Palawan to Manila.

Sometime in January 1986, private respondent Panama Sawmill Co., Inc. (Panama)
bought, in Palawan, 1,208 pieces of Apitong logs, with a total volume of 2,000 cubic
meters. It hired Transpacific Towage, Inc., to transport the logs by sea to Manila and
insured it against loss for P1-M with petitioner Oriental Assurance Corporation (Oriental
Assurance). There is a claim by Panama, however, that the insurance coverage should
have been for P3-M were it not for the fraudulent act of one Benito Sy Yee Long to whom
it had entrusted the amount of P6,000.00 for the payment of the premium for a P3-M
policy.

The logs were loaded on two (2) barges: (1) on barge PCT-7000,610 pieces of logs with
a volume of 1,000 cubicmeters; and (2) on Barge TPAC-1000, 598 pieces of logs, also
with a volume of 1,000 cubic meters.

On 28 January 1986, the two barges were towed by one tug-boat. During the voyage,
rough seas and strong winds caused damage to Barge TPAC-1000 resulting in the loss
of 497 pieces of logs out of the 598 pieces loaded. Panama demanded payment for the
loss but Oriental Assurance refuse on the ground that its contracted liability was for
"TOTAL LOSS ONLY." RTC and CA rendered a decision ordering petitioner to pay
Panama.
177

Both Courts shared the view that the insurance contract should be liberally construed in
order to avoid a denial of substantial justice; and that the logs loaded in the two barges
should be treated separately such that the loss sustained by the shipment in one of them
may be considered as "constructive total loss" and correspondingly compensable.

ISSUE:

Whether or not Oriental Assurance can be held liable under its marine insurance policy
based on the theory of a divisible contract of insurance and, consequently, a constructive
total loss.

RULING:

No liability attaches.

The terms of the contract constitute the measure of the insurer liability and compliance
therewith is a condition precedent to the insured's right to recovery from the insurer.
Whether a contract is entire or severable is a question of intention to be determined by
the language employed by the parties. The policy in question shows that the subject
matter insured was the entire shipment of 2,000 cubic meters of apitong logs. The fact
that the logs were loaded on two different barges did not make the contract several and
divisible as to the items insured. The logs on the two barges were not separately valued
or separately insured. Only one premium was paid for the entire shipment, making for
only one cause or consideration. The insurance contract must, therefore, be considered
indivisible.

More importantly, the insurer's liability was for "total loss only." A total loss may be either
actual or constructive (Sec. 129, Insurance Code). An actual total loss is caused by:

(a) A total destruction of the thing insured;

(b) The irretrievable loss of the thing by sinking, or by being broken up;

(c) Any damage to the thing which renders it valueless to the owner for the
purpose for which he held it; or

(d) Any other event which effectively deprives the owner of the possession, at
the port of destination, of the thing insured. (Section 130, Insurance Code).

A constructive total loss is one which gives to a person insured a right to abandon, under
Section 139 of the Insurance Code. This provision reads:

SECTION 139. A person insured by a contract of marine insurance may abandon the
thing insured, or any particular portion thereof separately valued by the policy, or
otherwise separately insured, and recover for a total loss thereof, when the cause of the
loss is a peril injured against,
178

(a) If more than three-fourths thereof in value is actually lost, or would have to
be expended to recover it from the peril;

(b) If it is injured to such an extent as to reduce its value more than three-fourths;

CA treated the loss a constructive loss. The requirements for the application of Section
139 of the Insurance Code, quoted above, have not been met. The logs involved,
although placed in two barges, were not separately valued by the policy, nor separately
insured. The logs having been insured as one inseparable unit, the correct basis for
determining the existence of constructive total loss is the totality of the shipment of logs.
In the absence of either actual or constructive total loss, there can be no recovery by the
insured Panama against the insurer, Oriental Assurance. The judgment under review is
hereby SET ASIDE and petitioner, Oriental Assurance Corporation, is hereby
ABSOLVED from liability under its marine insurance policy
179

Case Title
Vector Shipping Corporation v. American Home
Assurance Company (G.R. No. 159213)

Topic Prescription

Digested By: PUNO, Jensen Marie O.

FACTS:

1. Vector was the operator of the motor tanker M/T Vector, while Soriano was the
registered owner of the M/T Vector. Respondent is a domestic insurance
corporation.
2. On September 30, 1987, Caltex entered into a contract of affreightment with Vector
for the transport of Caltex's petroleum cargo through the M/T Vector. Caltex
insured the petroleum cargo with respondent for P7,455,421.08.
3. In the evening of December 20, 1987, the M/T Vector and the M/V Doña Paz, the
latter a vessel owned and operated by Sulpicio Lines, Inc., collided in the open sea
near Dumali Point in Tablas Strait, located between the Provinces of Marinduque
and Oriental Mindoro. The collision led to the sinking of both vessels. The entire
petroleum cargo of Caltex on board the M/T Vector perished.
4. On July 12, 1988, respondent indemnified Caltex for the loss of the petroleum
cargo in the full amount of P7,455,421.08.
5. On March 5, 1992, respondent filed a complaint against Vector, Soriano, and
Sulpicio Lines, Inc. to recover the full amount of P7,455,421.08 it paid to Caltex.
On December 10, 1997, the RTC issued a resolution dismissing the case on the
ground that when the case was filed against defendants Vector Shipping and
Francisco Soriano on 5 March 1992, the action not having been interrupted, had
already prescribed.

ISSUE: Whether this action of respondent was already barred by prescription for bringing
it only on March 5, 1992.
180

RULING:
No. The petition lacks merit. The Court concurs with the CA's ruling that respondent's
action did not yet prescribe. The legal provision governing this case was not Article 1146 of the
Civil Code, 16 but Article 1144 of the Civil Code, which states: “The following actions must be
brought within ten years from the time the cause of action accrues: (1) Upon a written contract;
(2) Upon an obligation created by law; (3) Upon a judgment.” The Court finds and holds that that
the present action was not upon a written contract, but upon an obligation created by law.
Hence, it came under Article 1144 (2). This is because the subrogation of respondent to
the rights of Caltex as the insured was by virtue of the express provision of law embodied in
Article 2207, to wit: “If the plaintiff's property has been insured, and he has received indemnity
from the insurance company for the injury or loss arising out of the wrong or breach of contract
complained of, the insurance company shall be subrogated to the rights of the insured against
the wrongdoer or the person who has violated the contract. If the amount paid by the insurance
company does not fully cover the injury or loss, the aggrieved party shall be entitled to recover
the deficiency from the person causing the loss or injury.”
Verily, the contract of affreightment that Caltex and Vector entered into did not give rise
to the legal obligation of Vector and Soriano to pay the demand for reimbursement by
respondent because it concerned only the agreement for the transport of Caltex's petroleum
cargo. As the Court has aptly put it in Pan Malayan Insurance Corporation v. Court of Appeals,
supra, respondent's right of subrogation pursuant to Article 2207, was "not dependent upon, nor
d[id] it grow out of, any privity of contract or upon written assignment of claim [but] accrue[d]
simply upon payment of the insurance claim by the insurer." Considering that the cause of
action accrued as of the time respondent actually indemnified Caltex in the amount of
P7,455,421.08 on July 12, 1988, 19 the action was not yet barred by the time of the filing of its
complaint on March 5, 1992, 20 which was well within the 10-year period prescribed by Article
1144 of the Civil Code.

You might also like